You are on page 1of 89

RECURSOS DIDÁCTICOS.

2
ORIENTACIONES y solucionario

Lengua y Literatura
ÍNDICE
1. BACHILLERATO edebé . . . . . . . . . . . . . . . . . . . . . . . . . . . . . . . . . . . 4

2. CÓMO ES EL LIBRO DEL ALUMNO . . . . . . . . . . . . . . . . . . . . . . . . . . 6

3. LAS CLAVES DEL NUEVO PROYECTO . . . . . . . . . . . . . . . . . . . . . . . . 8

4. SOLUCIONARIO

Unidad   0. . . . . . . . . . . . . . . . . . . . . . . . . . . . . . . . . . . . . . . . . . . . . . . . . 14

Unidad   1.  La palabra . . . . . . . . . . . . . . . . . . . . . . . . . . . . . . . . . . . . . . . 17

Unidad   2.  De la palabra a la oración . . . . . . . . . . . . . . . . . . . . . . . . . . . . 21

Unidad   3.  La oración compuesta. La coordinación . . . . . . . . . . . . . . . . . . 26

Unidad   4.  La subordinación sustantiva . . . . . . . . . . . . . . . . . . . . . . . . . . 30

Unidad   5.  La subordinación adjetiva . . . . . . . . . . . . . . . . . . . . . . . . . . . . 36

Unidad   6.  La subordinación adverbial.


Estructuras oracionales bimembres . . . . . . . . . . . . . . . . . . . . . 41

Unidad   7.  El discurso . . . . . . . . . . . . . . . . . . . . . . . . . . . . . . . . . . . . . . . 47

Unidad   8.  El español actual . . . . . . . . . . . . . . . . . . . . . . . . . . . . . . . . . . . 52

Unidad   0. . . . . . . . . . . . . . . . . . . . . . . . . . . . . . . . . . . . . . . . . . . . . . . . . 58

Unidad   9. El modernismo y la generación del 98 . . . . . . . . . . . . . . . . . . . 59

Unidad 10.  El novecentismo y el vanguardismo . . . . . . . . . . . . . . . . . . . . . 63

Unidad 11.  La lírica del 27 . . . . . . . . . . . . . . . . . . . . . . . . . . . . . . . . . . . . 67

Unidad 12.  El teatro de principios del siglo xx . . . . . . . . . . . . . . . . . . . . . . 71

Unidad 13.  La narrativa. De la posguerra a la actualidad . . . . . . . . . . . . . . 74

Unidad 14.  La lírica. De la posguerra a la actualidad . . . . . . . . . . . . . . . . . 78

Unidad 15.  El teatro. De la posguerra a la actualidad . . . . . . . . . . . . . . . . . 81

Unidad 16.  La literatura hispanoamericana desde el siglo xx . . . . . . . . . . . 84


BACHILLERATO edebé

COMPROMISO CON LOS VALORES PROPIOS DEL BACHILLERATO

Rigor y CUltura
actualización del esfuerzo
científica —— Trabajo eficiente mediante actividades,
—— Uso preciso y eficaz del conocimiento documentos... que exigen una actitud
científico. proactiva por parte del alumnado.
—— Actualización y contextualización del
conocimiento.

Curiosidad Autonomía y
intelectual responsabilidad
—— Visión 360º: el conocimiento más allá —— Capacidad para gestionar el propio
del aula. aprendizaje por medio de retos
—— Descubrimiento del gusto por saber. abordables.

Seis habilidades para una sociedad global

Cooperación Compromiso con valores


—— Propuestas para un trabajo cooperativo. —— Compromiso ético para convivir en una socie-
—— PBL (Problem-based learning / Aprendi- dad cambiante, para crecer como persona…
zaje basado en problemas).

Pensamiento crítico Creatividad


—— Actividades de razonamiento para —— Actitud creativa y superación de retos.
hacer frente a la toxicidad de la —— Actitudes abiertas y flexibles para abordar
información. retos aportando soluciones nuevas y creativas.

Comunicación Iniciativa
—— Gestión de la información y la comuni- —— Toma de decisiones e iniciativa emprendedo-
cación de manera efectiva. ra mediante actividades y proyectos para la
—— Las TIC como herramienta de comuni- creación de miniempresas.
cación y fuente de aprendizaje.
edebé n
proyecto global interactivo

LIBRO DIGITAL INTERACTIVO


Incluye los recursos digitales necesarios (presentaciones, vídeos y
otros recursos como antologías, monográficos...) para que el profe-
sorado gestione de forma eficaz el aprendizaje en el aula digital.

presentaciones
Presentación multimedia de contenidos.

VÍDEOS
Contenidos audiovisuales de apoyo en el aula.

OTROS RECURSOS
Amplia oferta de recursos educativos.

biblioteca de recursos digitales


Un espacio fácilmente accesible donde encontrar recursos para
consultar, descubrir y explorar el conocimiento.

PARA EL PROFESOR
—— Programaciones didácticas, según los requisitos y especificaciones estableci-
dos en la normativa vigente.
—— Orientaciones y solucionario (en formato PDF), para facilitar la labor del profe-
sor.
—— Generador de evaluaciones, una importante base de datos con ítems de diver-
sa tipología para enriquecer las propuestas de evaluación en el aula.

Disponible en «tu espacio personal»: www.edebe.com


Multidispositivo
LAS CLAVES DEL NUEVO PROYECTO

COMPROMISO CON LOS VALORES PROPIOS


DEL BACHILLERATO
El Bachillerato aporta la cultura personal para toda la vida.
Ha de incentivar el gusto por el conocimiento, el aprendizaje y el estudio personal motivador y exigente.

1.  Rigor y actualización científica


El nuevo proyecto de edebé se asienta sobre unas bases sólidas. La editorial edebé ofrece un método consolidado para una
educación integral en la sociedad del conocimiento.
El rigor científico es la capacidad para utilizar la información y el conocimiento científico, las normas y los procedimientos
propios de cada disciplina con precisión y eficacia. El rigor científico hace despertar sentimientos de insatisfacción ante la
incertidumbre, las respuestas inexactas, las mediciones poco precisas, la amplitud del más y del menos... El rigor también es
metódico: es fiel y mantiene apego al procedimiento experimental, reclama exigencia en el control de todos los parámetros que
pueden incidir en una situación o en un proyecto, aportando así franqueza y credibilidad técnica.
Por ello, en el nuevo Bachillerato, edebé ofrece:
—— Contenidos actualizados y contrastados. Incorpora los últimos avances científicos y los enfoques más actuales.
—— Valor del método propio de cada disciplina científica y del conocimiento científico ante la provisionalidad del conocimiento.
—— Textos explicativos estructurados. El orden y una estructura coherente en el desarrollo de los contenidos facilitan los
aprendizajes por parte de los alumnos.
—— Libros claros, en la exposición del contenido, en la selección de imágenes y gráficos.
—— Actividades inteligentes, que obligan a pensar.

8
BACHILLERATO edebé

2.  Curiosidad intelectual y cultura del esfuerzo


Se afirma que algunos de los grandes pensadores de la historia (aquellos que con sus aportaciones han provocado cambios
en el mundo: Leonardo da Vinci, Einstein, Steve Jobs...) comparten, tienen en común, una curiosidad insaciable a lo largo de
toda su vida.

«No tengo un talento especial, solo soy «Muchas de las cosas con que me topé por seguir mi curiosidad y mi intui-
apasionadamente curioso». ción resultaron más tarde ser de un valor incalculable».
Einstein Steve Jobs

La curiosidad es la capacidad que nos lleva a profundizar en determinados temas y superar los propios límites. Es el deseo de
comprender el significado de lo que nos rodea y disfrutar de experiencias más enriquecedoras y plenas. En estas situaciones
las personas dedican tiempo y esfuerzo, ya que la finalidad merece la pena.
Recientes estudios han demostrado que la curiosidad (la inquietud intelectual) y el esfuerzo (el trabajo duro) influyen más
directamente en el rendimiento académico que la propia capacidad intelectual.
La curiosidad es el hambre por descubrir cosas nuevas. Las personas con mentes curiosas pueden adaptarse de manera
más exitosa a los entornos cambiantes que caracterizan nuestra sociedad actual, ampliar sus horizontes y evolucionar como
personas.
Por ello, en el nuevo Bachillerato, edebé introduce el aprendizaje 360º. En él:
—— Se sugieren temas que despiertan el interés y mueven a indagar y ampliar el conocimiento.
—— Se despierta la curiosidad intelectual, el gusto por aprender, e invita a descubrir curiosidades, hechos sorprendentes...
y todo aquello que puede llevar al alumno a aprender fuera del aula.

De este modo, en el nuevo proyecto, se abre el libro al mundo y se integran los aprendizajes no formales e informales, y se
recupera el espíritu de trabajo e indagación tan necesario en el Bachillerato.

3. Autonomía y responsabilidad.
El alumno, arquitecto de su propio aprendizaje
Están muy relacionadas con la curiosidad intelectual y el esfuerzo personal.
El alumno autónomo regula su conducta con normas que surgen de su propia conciencia; es capaz de hacer lo que tiene
que hacer por sí mismo, siguiendo su conciencia moral.
Además, el alumno autónomo se da cuenta de las consecuencias de sus actos, toma conciencia de ello y se hace respon-
sable. La responsabilidad es un valor que hace reflexionar a la persona, le permite gestionar su vida y valorar las conse-
cuencias de sus actos.
Las fuentes de información son hoy muy diversas, pero no aportan un conocimiento diverso y consolidado por sí mismas.
Es necesario enseñar a los alumnos a transformar la información en conocimiento. Para lograrlo el alumno ha de mantener
una actitud activa, comparar diferentes informaciones, realizar inferencias, buscar nuevas soluciones a los problemas...
Por ello, en el nuevo Bachillerato, edebé:
—— Propone retos abordables para que los alumnos se responsabilicen de su propio aprendizaje y obtengan una respuesta
positiva de su esfuerzo, muestren una actitud activa que les guíe a descubrir el gusto por saber y progresen en su auto­
nomía como personas.
—— Ha añadido valor a su propuesta, consciente de que la nueva sociedad reclama una formación más sólida y una base
cultural más amplia, y propone el establecimiento de filtros científicos para hacer frente a la toxicidad de la información.

9
BACHILLERATO edebé

SEIS HABILIDADES PARA UNA SOCIEDAD GLOBAL

El Bachillerato viene a representar un hito en el proceso de adquisición de la cultura personal.


Es el último escalón de la educación formal para la consecución de la llamada
«cultura general». Pero, además, los alumnos deben asumir la necesidad de desarrollar
unas habi­lidades básicas para poder afrontar con éxito los requerimientos
de la nueva sociedad global. Son las siguientes:

Cooperación

Comunicación

Pensamiento crítico

Creatividad

Iniciativa

Compromiso con valores

1010
BACHILLERATO edebé

COOPERACIÓN COMUNICACIÓN

El nuevo Bachillerato de edebé aspira a formar a jóvenes cuyo La comunicación es una necesidad humana básica, indispensable
objetivo sea aportar valor a las personas y a la sociedad. Y para para la organización de las sociedades. Las habilidades comunica­
hacerlo, deben colaborar tanto desde el centro escolar como des­ tivas se consideran básicas tanto en el mundo del trabajo como en
de fuera. las relaciones sociales. Gestionar la información y comunicarla de
manera efectiva es uno de los retos importantes de los alumnos
Se trata simplemente de que cada equipo encuentre interseccio­ del Bachillerato y una de las claves para la construcción de las so­
nes con otros, de una manera no accidental, sino sistemática. ciedades de la información y la comunicación.
Todo ello desde la convicción de que el modo de generar energía
positiva en la sociedad actual consistirá en mezclar equipos, com­ En la sociedad de la información y el conocimiento, las TIC se han
binando sus activos invisibles, sus capacidades y sus conocimien­ afianzado como herramientas básicas para la comunicación. La edi­
tos, para explorar y explotar nuevas formas de generar valor para torial edebé se ha orientado hacia el humanismo tecnológico de
las personas y para la sociedad. nueva generación (la tecnología al servicio de las personas) y hacia la
proximidad ecológica a la realidad de la escuela de hoy, ofreciendo
En la sociedad actual, por mucho que un individuo aislado se es­ recursos asumibles por el profesorado y el centro:
fuerce, hay más conocimiento fuera que dentro; el conocimiento
disponible en el mundo es superior al del individuo. —— El libro digital interactivo, incluye los recursos digitales nece­
sarios (simuladores, presentaciones, problemas interactivos,
Además, la respuesta a los problemas de hoy requiere de una vi­ practicas de laboratorio, vídeos, y otros recursos como audios,
sión más periférica: hay que combinar maneras de ver, maneras galerías de imágenes, enlaces, documentos... ) para que el profe­
de resolver, de convencer... sorado gestione de forma eficaz el aprendizaje en el aula digital.

Parafraseando a A. Cor­ —— El generador de actividades, para poner a disposición del profe­


nella, el mundo es cada sorado toda una serie de propuestas para el trabajo en el aula.
vez más «co»: colaborativo, —— La biblioteca de recursos digitales, un espacio fácilmente acce­
cooperativo, cocreativo, sible donde encontrar recursos para consultar, descubrir y explo­
codiseñado, corresponsa­ rar el conocimiento.
ble...
La oferta digital de edebé se sitúa en un marco de convivencia papel/
Por ello, edebé incorpora
digital para aprovechar al máximo las posibilidades formativas de
propuestas para el trabajo
cada soporte y para promover el uso estratégico de cada formato por
cooperativo, PBL (Apren-
parte de los alumnos (anfibios analógico-digitales).
dizaje Basado en Proble-
mas)... Todo ello bajo la premisa de compatibilidad y entorno amigable.

PENSAMIENTO CRÍTICO CREATIVIDAD

En la sociedad de la información y el conocimiento, las fuentes La escuela (y la sociedad en general) está inmersa en un nuevo pa­
de información están más accesibles que nunca; ahora bien, re­ radigma educativo. Los avances de las neurociencias, la cadu­cidad
sulta imprescindible capacitar a la juventud para acceder a infor­ del conocimiento, la globalización, la revolución tecnológica... sitúan
mación de calidad. El desarrollo del pensamiento crítico permitirá el foco de la acción educativa en unas nuevas coordenadas.
a los alumnos establecer los filtros científicos necesarios para
hace frente a la toxicidad de la información y a los mensajes ses­ Necesitamos formar personas competentes, capaces de abordar
gados o manipuladores. problemas desde diferentes ámbitos en los que aporten soluciones
nuevas y creativas; que puedan enfrentarse a la vida en un entorno
En esta sociedad cambiante el conocimiento se caracteriza por su cambiante.
provisionalidad y su caducidad. Es importante enseñar al alum­
nado a aprender a aprender (aprender-desaprender-reaprender) Si queremos formar mentes flexibles (con múltiples y flexibles co­
y a reflexionar sobre los procesos y el resultado del aprendizaje. nexiones cerebrales), debemos abandonar las actitudes pasivas,
rígidas o repetitivas en las aulas y promover procedimientos de com­
Por ello, edebé ofrece en el nuevo Bachillerato: paración/contraste de informaciones, llevar a cabo inferencias o
deducciones, buscar nuevas soluciones... en nuestras clases.
—— Variedad de actividades de análisis, síntesis y ejercicios de
razonamiento.
Por ello, edebé:
—— Contraste de opiniones y puntos de vista al presentar conte­
nidos complejos o susceptibles de enfoques ideológicos di­ —— Incorpora PBL en sus nuevos materiales para el Bachillerato.
versos.
—— Propone actividades inteligentes que obligan al alumno a pensar,
—— Situaciones y propuestas de trabajo en grupos para hacer a relacionar, a inferir, a encontrar soluciones creativas e innova­
converger diferentes puntos de vista sobre un mismo tema... doras.

11
BACHILLERATO edebé

INICIATIVA COMPROMISO CON VALORES

En el nuevo proyecto de Bachillerato, edebé ha destinado una Las personas con alta competencia moral muestran sensibilidad por
atención especial a la iniciativa emprendedora. el mundo que nos rodea y contribuyen con aportaciones personales
a la mejora de la sociedad. Las personas con valores se muestran
1.  ¿Por qué el emprendimiento? honestas, íntegras y con un claro compromiso social.

—— Desarrolla la autonomía, la iniciativa personal y la capacidad La formación en valores es necesaria:


de liderazgo. —— Para convivir en una sociedad cambiante.
—— Potencia la creatividad y la capacidad de innovación. —— Para crecer como persona.

—— Prepara para la resolución de problemas y la toma de deci­ —— Para transformar el mundo.


siones. —— Para dar respuesta a los valores de la nueva sociedad.
—— Implica un componente activo (capacidad de uno mismo para —— Para abrir espacios de interioridad.
provocar cambios) y uno pasivo (aceptar y apoyar cambios pro-
ducidos por factores externos), y permite asumir la responsa­ Por ello, edebé impregna de valores el desarrollo de los contenidos
bilidad de las propias acciones. del Bachillerato de manera natural, sin forzar ni desnaturalizar, cuan­
do encajan con el contenido que se trabaja.
2.  ¿Qué capacidades se desarrollan? No añade contenido nuevo, sino que aporta un punto de vista posi­
tivo al contenido.
—— Cualidades personales: la iniciativa personal, la confianza en
uno mismo, la creatividad, el dinamismo... que hacen a las
personas activas ante las circunstancias que las rodean.

—— Habilidades sociales: actitudes de cooperación y de trabajo en


equipo, hábito de asumir nuevos roles en una sociedad en con­
tinuo cambio. También supone capacidad de la relación con el
entorno y sensibilidad ante las necesidades de los otros.

—— Habilidades para la dirección y el liderazgo: planificar, dirigir


equipos, tomar decisiones y aceptar responsabilidades. Tam­
bién significa poder de comunicación.

—— Espíritu innovador, necesidad de ensayar nuevas experien­


cias o hacer las cosas de manera diferente, simplemente por
la existencia de posibilidades de cambio.

Por ello, el nuevo proyecto de Bachillerato de edebé ofrece para


cada asignatura un proyecto emprendedor (Proyecto miniempre-
sa) a través del cual los alumnos crearán, planificarán, tomarán
de­cisiones... en torno a un proyecto práctico y motivador. Todos
los proyectos de miniempresa que ofrece edebé, además de su
carácter técnico y profesional, tienen un trasfondo social y aspiran
a conseguir un mundo un poco mejor cada día.

12
SOLUCIONARIO
0# Lengua

 1. Escribe otro título… Respuesta abierta. Sintetizamos a conti-  6. Localiza en el texto...: Modificadores oracionales: Sin duda,
nuación el tema que ofrece el escrito: se ofrece al lector un un artículo para aprovechar mejor la moto – Seguro que si
abanico de palabras y expresiones que han caído en desuso se esfuerzan aparecerán muchas. / Marcadores de función
con el paso del tiempo. También se invita al lector a recordar textual: Hacía, en cambio, una tipología del calavera (refor-
y aportar algunos ejemplos a la lista. mulador). / Por ejemplo, pollo pera (argumentativo). / Lue-
go, una especie de elogio (estructurador de información). /
 2. Con qué intención escribe…: El propósito del autor con el artícu- No he conseguido descubrir el origen de la expresión pollo
lo es buscar la complicidad de los lectores a partir de la re- pera. Pero eso no es extraño (conector de oposición y con-
flexión sobre expresiones y términos que se utilizaban en otra traste).
época y que, ahora, han caído en desuso.
 7. Redacta un artículo Respuesta abierta.
Si el lenguaje y el registro son adecuados: El lenguaje y el tono
utilizados son amenos, cotidianos y muy fáciles de asimilar  8. Calavera: Persona disipada, juerguista e irresponsable. /
po-
teniendo en cuenta el medio en el que se difunde la informa- llo pera:
persona muy presumida, demasiado elegante y refi-
ción: una revista para personas de la tercera edad. nada. / P ijo : persona que manifiesta en su actitud y
Qué procedimientos utiliza…: El autor utiliza preguntas directas vestimenta las maneras propias de la sociedad adinerada.
dirigidas al lector y ejemplos que puedan conocer para ani- Explica las acepciones de las palabras… juerga: fiesta, jolgo-
marlo a participar y a buscar más ejemplos. También busca la rio. Proviene de la aspiración de la palabra huelga. / Retre-
empatía explicando la investigación para encontrar sentido a te : aposento destinado a la evacuación. Inicialmente, se
algunas expresiones. utilizaba para referirse a la cámara donde se encontraba el
recipiente. / Ratón: roedor muy común. Actualmente, se
 3. Identifica la idea que desarrolla… El primer párrafo empieza utiliza la misma palabra para designar el dispositivo utiliza-
con la declaración del tema: hay palabras que dejan de do para hacer funcionar el cursor de los ordenadores. /
usarse, a partir del titular de un artículo en el que aparecía la Fanático: Persona que defiende una creencia con una pa-
palabra sidecar. En el segundo párrafo se pregunta directa- sión exagerada, a veces sin atender al uso de la razón. En
mente al receptor qué palabras conocen que ya no se utili- la antigüedad, se utilizaba esta misma palabra para definir
zan y se pone de ejemplo el término «calavera» en su a los esclavos que servían en los templos (latín: fanum). /
acepción de ‘persona ociosa’. En el tercer párrafo se expo- Azafata: persona que atiende a los pasajeros de un avión o
nen más ejemplos, como «pollo pera» y lo extraño de su tren o en los congresos. Antiguamente, designaba a las
origen para la acepción de ‘pijo’. El cuarto párrafo profundi- criadas de la reina, cuyo nombre proviene de las bandejas
za en la idea del «pollo pera» como cercano al «dandi» y la donde se colocaban las joyas. / Villano: persona que es
evolución de la forma para definir el mismo concepto en capaz de actuar de manera ruin o cruel. Antes se designa-
otras, como «metrosexual». ba así a las personas que vivían en las villas.

 4. Justifica que el texto es coherente…: El autor ofrece casos a  9. Averigua cuál es el origen… parranda: de origen bastante dis-
partir de su propia experiencia. Cada uno de ellos refuerza el cutido. Algunos apuntan a los grupos festivos que iban de
tema (la lengua cambia, pierde términos y aparecen otros) y parra en parra. Otros a la palabra farra, del portugués y este, a
abre nuevos puntos. Utiliza su propia experiencia para ir aña- su vez, del árabe ‘far’she’ (fiesta) (superestrato). / Viajeros:
diendo nuevos casos y remite a la experiencia del lector. Ade- proviene de viaje y este, a su vez, del catalán viatge (neologis-
más, hace referencia a textos relacionados con el tema mo, préstamo). / Silvestre: del latín silvestris, ‘procedente del
principal y a los ejemplos aportados. bosque’(neologismo). / Rico: procedente de la palabra gótica,
‘reiks’ (superestrato). / Ropas: palabra procedente del gótico
 5. Localiza procedimientos de…: Reiteración léxica: palabras / raupa (superestrato).
Sinónimo contextual: adminículo – receptáculo / Pronomi-
nalización: Recientemente lo sustituyó la palabra metro- 10. Localiza dos extranjerismos del texto…: Metrosexual y side-
sexual - el esnob se muere porque le inviten / Anáfora: El car, ambas del inglés. Metrosexual es una palabra com-
calavera puede ser “silvestre” o “doméstico”. El calavera puesta por metropolitan y sexual. Sidecar, de side (lado) y
“silvestre”… El calavera “doméstico”… - su cuna, su edad, car (coche).
su educación, su dinero… / Elipsis: Desconozco también la
Localiza todos los extranjerismos del texto… Inglés: Trending
[expresión] precedente de este uso - En el siglo XIX abunda-
topic, reality, reality show, The walking dead, zombies.
ron [los dandis] / Deíxis temporal y personal: Hoy, en el pe-
riódico, he leído un artículo… - su cuna… rico por sus Indica si alguno ha sido adaptado al castellano: Ninguna palabra
papás. ha sido adaptada a nuestra lengua.
Localiza palabras del mismo campo semántico…: moto: autobuses, ¿En algún caso podría sustituirse por una palabra en castellano?
sidecar / palabra: acepción / Apariencia: moda, aspecto. La palabra ranking podría sustituirse por ‘listado’, y trending
Recursos léxicos: reiteración léxica, sinónimos contextuales, topic por ‘tema de actualidad’.
pronominalización, campo semántico. / Recursos gramaticales:
anáfora, deíxis, elipsis.
11. Señala al menos diez países donde el español sea lengua oficial:

14
Unidad 0 >  Lengua

Uruguay, Panamá, Guinea Ecuatorial, Nicaragua, Honduras, Derecho: (sustantivo) conjunto de normas que regulan las
Guatemala, Chile, Bolivia, Ecuador, Costa Rica. relaciones humanas en la sociedad, justicia; (adjetivo) recto,
cierto, razonable / sentido: (sustantivo)c apacidad para sentir
México. Cuate: del náhuatl coatl (‘mellizo’), compañero, cama-
estímulos externos; (adjetivo) que expresa un sentimiento.
rada, mellizo. / Jején: del arahuaco xixén, un tipo de insecto. /
Chamaco: niño, del náhuatl chamahua (‘crecer’). Señala los sintagmas de la oración anterior:
Venezuela. Arepa: del cumanagoto, erepa (‘maíz’), especie de Ahora (SAv) al mundo (SPrep) se va (núcleo SV) en cercanías
pan de forma circular hecho de maíz. / Catire: del cumanagoto, (SPrep/CCM): oración impersonal
‘persona rubia’. / Chamo: etimológicamente puede venir del in-
glés chum (‘amigo’) y este del náhuatl chamahuac (‘rollizo’), o de Señala a qué término se refiere la t del anuncio: La T se refiere a
la lengua chibcha, chamab (‘nuestra sangre’), significa joven. la palabra Terminal, referente al Aeropuerto de Barajas. Se
abrevia para que sea más fácil de recordar.
Perú. Guachimán: del inglés watchmen (vigilante), designa a
los guardas costeros. / Arranchar: del francés, ranger (‘hilera’ ¿Qué tipo de palabra es …?: Renfe: Red Nacional de Ferro-
o ‘corro’), utilizado con el significado de ‘quitar algo a alguien carriles Españoles. Por su formación, se trata de un acróni-
con violencia’. mo.
Colombia. Chacho: ‘protagonista’, origen incierto. Yuyo: del
quechua yuyu (‘mala hierba’), condimento. 14. Indicael sujeto…: observará: usted. / Oímos: nosotros; canta:
alguien; tiene: nota / se puede definir: el tono (pasiva refleja).
Argentina. Achirlar: hacer chirle o más líquida una mezcla.
Semipiso: apartamento que ocupa con otro la totalidad de una Oración compuesta por yuxtaposición: Cuando se avanza hacia la
planta, de semi- y piso. derecha en el teclado, el tono es cada vez más agudo; en di-
Estados unidos. Guachar: del inglés, watch (‘mirar’) mirar, ver. rección contraria, el tono es más grave.
Yardero: del inglés yard (‘jardín’), el que se encarga de cortar Dos oraciones compuestas por subordinación: Si interpreta When
el césped en el jardín. the Saints Go Marching In en el teclado de un piano, observa-
Indica a qué registro y nivel de la lengua pertenecen…: La ma- rá que cada nota que toque se encuentra a la derecha de la
yoría de los términos anteriores pertenecen a un registro anterior. / Cada nota que oímos cuando alguien canta o toca
medio y vulgar. un instrumento musical tiene su tono .

Señala los rasgos referidos al léxico del español de américa…: Los Dos oraciones con verbos copulativos: el tono es cada vez más
rasgos del español de América que se refieren al léxico son: agudo / el tono es más grave.
arcaísmos, empleo de neologismos, uso de extranjerismos.
Dos oraciones con verbos semicopulativos: al mover la mano del
12. Explica quién es el remitente…: El remitente de la carta es Cris- mástil hacia el puente el tono se hace más agudo / el tono se
tóbal Colón y va dirigida a Sus Altezas, los Reyes Católicos. El vuelve más grave.
propósito es informar de la expedición de la nueva ruta hacia Dos oraciones impersonales: Cuando se avanza hacia la dere-
las Indias. Está escrito en un registro formal en un nivel culto cha en el teclado /al moverla hacia las clavijas.
de la lengua.
Transforma la yuxtapuesta en coordinada: Cuando se avanza
Reescribe la carta… Respuesta abierta. hacia la derecha en el teclado, el tono es cada vez más agudo
y, en dirección contraria, el tono es más grave.
13. Uso del adverbio ahora: para indicar inmediatez y moderni-
dad. Transforma las copulativas en predicativas Las notas adquieren
un tono cada vez más agudo. Las notas pasan a un tono más
Frases nominales, sin verbos: causan mayor impacto y son más
grave.
fáciles de recordar. Verbos en presente y segunda persona del
singular: permiten dirigirse más directamente al lector como si Transforma las impersonales en personales: Cuando alguien
se tratara de una conversación. avanza hacia la derecha en el teclado. Cuando la mueves ha-
Sintagmas nominales sin determinar: son ideas clave formuladas cia las clavijas.
para que se recuerde con mayor facilidad. Analiza morfosintácticamente…:
Analiza morfológicamente…: En (Prep) música (N) se dice (V) que (Conj) cada (Det)
tono (N) es (V) un (Det) sonido (N) fijo (A). Oración com-
Tiempo, puesta por subordinación sustantiva introducida por la
Palabra Categoría Género Número
modo, Pers. conjunción que. La proposición subordinada (que cada
Ahora Adverbio tono es un sonido fijo) realiza la función de sujeto. Es una
Al Prep + Det Masc Sing oración pasiva refleja.
Mundo Sustantivo Mas Sing Todas (Det) las (Det) teclas (N) blancas (A) de (Prep) un
Marcador de (Det) teclado (N) de (Prep) piano (N) estándar (A) tienen (V)
Se - - - nombres (N) de (Prep) nota, en (Prep) la (Det) mayor (A)
impersonalidad
parte (N) de (Prep) Europa (N) estas (Pron) van (V) del
Presente de
Va Verbo - Sing (Prep+Det) do (N) al (Prep+Det) si (N). Oración compuesta
indic. 3.ª pers
por yuxtaposición, ya que las proposiciones van unidas sin
En Preposición
nexo explícito.
cercanías Sustantivo Fem Plural -

15
Unidad 0 >  Lengua

En  música  se dice  que  cada  tono  es  un  sonido  fijo
Prep N V Conj Det N V Det N A
D núcleo CN
D núcleo núcleo SN / Atr
R SN/Sujeto SV/Predicado
Sprep \ CCL núcleo Pr. Sub. Sustantiva
SV/Predicado SN/Sujeto
Oración compuesta por subordinación sustantiva

Todas las teclas blancas de un teclado de piano estándar tienen nombres de nota, en la mayor parte de Europa estas van del do al si.
Det Det N A  Prp Det N Prp N A V N Prp N Prp Det Adj N Prp N Pron V     P/De  N P/De N
r nú SA/CN r núcleo r núcleo r/D nú r/D nú
r D núcleo SPrep./CN núcleo SPrep. /CN D CN núcl SPrep./CN núcl núcl SPrep/CC.
D D núcleo SA/CN SPrep./CN núcleo SN/CD r SN
SN/Sujeto SV/Predicado SPrep. /CCL SN/S SV/Predicado
Proposición Proposición
Oración compuesta por yuxtaposición

16
1# La palabra

introdujo primero porque no existía una palabra que repre-


En contexto (pág. 15)
sentase la realidad en castellano y, posteriormente se acuñó
a) Los seis recursos para crear palabras en inglés son los présta- una alternativa calcada del término inglés: «balompié». Por
mos (caramelo, del francés), la creación de compuestos (heart- contra, los otros anglicismos se introdujeron pese a que en
breaker, rompecorazones), la aglutinación (motel o electro- castellano ya había otras palabras con significados parale-
cutar), el cambio de categoría gramatical (de verbo a sustantivo, los. Por lo tanto, la necesidad de unas y otras es diferente.
por ejemplo), la simplificación por derivación (excavadoras-ex-
cavar) y las siglas (NASA).  6. Actividad personal.
b) Respuesta personal.
 7. Indicamos si las afirmaciones propuestas son verdaderas o
falsas. Según el autor, los anglicismos no suponen ningún proble-
ma para el idioma: verdadera. / El verbo ignorar ha adoptado un
PRACTICA CON LOS TEXTOS (págs. 16-19)
nuevo significado tomado del inglés: verdadera. / Balompié es más
correcto que fútbol: falsa. / Banear es un verbo castellano co-
rrecto que posee el mismo lexema que contrabando y bandido: ver-
Comprensión dadera. / Los anglicismos empobrecen el idioma, porque desplazan
palabras propias: falsa.
  1. ¿Qué son los extranjerismos...? Respuesta orientativa: Un
extranjerismo es una palabra prestada de otra lengua que A continuación, reformulamos las oraciones falsas: Balompié,
se incorpora al léxico común de la lengua propia. Los an- respecto de fútbol, es un término que se emplea como recur-
glicismos son extranjerismos que proceden, concretamen- so estilístico y que remite a otras épocas. / Los anglicismos no
te, del inglés. empobrecen el idioma; son los hablantes que, al usarlos,
desplazan y olvidan palabras propias.
¿Qué significa que...? Respuesta orientativa. El autor aboga por
emplear anglicismos pero siempre y cuando no se olviden ni
se dejen de usar otras palabras propias patrimoniales de la Cuestión de lengua
lengua. El problema aparece cuando el uso de un anglicismo
provoca que otras palabras dejen de usarse (mueren, por  8. En este texto predominan el presente de indicativo (causan,
tanto, son cadáveres). hacen, viene, constituyen, ocupan, acaban, llamamos, nece-
En el texto se menciona...? Concretamente, se alude a la deriva- sitamos...) y el pretérito indefinido (llegó, abandonó, supo,
ción cuando explica la adaptación de la palabra «fútbol»: fut- progresó, venció, estableció...). Ambos tiempos son del modo
bolero, futbolista, futbolístico. Y también a la adaptación indicativo y son propios de textos expositivos y argumentativos
ortográfica y fonética de préstamos. en los que se expone un tema y se defiende a partir de he-
chos pasados o argumentos anteriores.
 2. Respuesta personal. En este ejercicio, se espera que los estu-
diantes argumenten de forma coherente el título aludiendo a  9. Nada y nadie son pronombres indefinidos. Pueden parafra-
la parte final del texto: incorporar anglicismos es lícito siempre searse por ninguna cosa, ninguna persona
y cuando no se empobrezca la lengua olvidándonos de otras
palabras de la propia lengua con significado similar. 10. En cuanto al significado, todas se refieren a una persona cuya
profesión es preparar o ayudar a otra a alcanzar un objetivo
A modo de ejemplo proporcionamos otro título: «Enriquece la específico. En cuanto a la forma, todas se forman mediante el
lengua con anglicismos, no la simplifiques». sufijo -or, que añade a la raíz el significado de “persona que
se dedica a”. Otros ejemplos son pescador, corredor, cobra-
 3. Respuesta personal. Proporcionamos una justificación a dor, estafador, escultor...
modo de ejemplo: al hablar o escribir transmitimos muchos
valores y una imagen que se mide por lo que se dice y cómo
se dice. Por lo tanto, no solo es importante qué expresamos, Cuestión de léxico
sino también qué omitimos; qué palabras usamos y cuáles
dejamos de usar. 11. Las definiciones que proporcionamos son las que aparecen
en el Diccionario de la Real Academia. acuñadas: Dar forma a
 4. Respuesta posible: la diferencia radica en que «fútbol» es un expresiones o conceptos, especialmente cuando logran difu-
préstamo del inglés y «balompié», una palabra creada a pos- sión o permanencia. / evocación: Dicho de una cosa: Traer algo
teriori con los recursos propios de la lengua castellana (como a la imaginación por asociación de ideas. / acaecer: suceder;
calco del inglés). Otros ejemplos: del inglés, baloncesto (bas- hacerse realidad. / consagrada: conferir a alguien o algo fama
ketball), comida rápida, (fast food) bicicleta de montaña o preeminencia en determinado ámbito o actividad. / descalifi-
(mountain bike)...; del francés, golpe de estado (coup d’État), car: desacreditar, desautorizar o incapacitar a alguien o algo.
puesta en escena (mise en scène)...
12. Completamos las oraciones con el parónimo correspondiente.
 5. Respuesta posible: la diferencia fundamental radica en el or- No se debe prescribir medicamentos a pacientes cuyo historial se
den de incorporación de las palabras: por un lado, «fútbol» se desconoce. / Los fugitivos eran unos proscritos de la ley. / Las

17
Unidad 1 >  La palabra

personas racistas se basan en prejuicios culturales. / El uso de cefálicas también y sudoríficas./ En esto era gran práctico y teóri-
los fluorocarbonos resulta un perjuicio para el medioambiente. co un gato, pedantismo retórico, / que hablaba en un estilo tan
/ Al sospechoso le han infligido una herida profunda con arma enfático / como el más estirado catedrático.
blanca. / Los investigaron por infringir la ley electoral.
 4. Completamos las oraciones propuestas: Cuando la sílaba tóni-
13. Sustituimos las metáforas por expresiones sinónimas. ca es un diptongo, debemos acentuar la palabra según las reglas
generales de acentuación. Ejemplo: camión, cuidado / En los
La palabra fútbol pagó los aranceles, se adaptó a la ortografía y a diptongos formados por una vocal fuerte y una débil, la tilde se
la morfología española. / Irrumpen ahora anglicismos que sí tienen
coloca en la vocal fuerte. Ejemplo: miráis, periódico. / En los
equivalente en español. / Los anglicismos ocupan parcelas de signi-
diptongos formados por dos vocales débiles, la tilde se coloca en
ficado que ya tenían huéspedes/significantes. / Ha entrado/penetra-
la segunda vocal. Ejemplo: cuídate, construí. / En los tripton-
do una nueva acepción de ignorar. / Olvidamos los orígenes de
gos, la tilde siempre se pone en la vocal central, si lo necesita
evento y por clonación de event se distancian acto, actuación. / El
según las reglas generales de acentuación. Ejemplo: despre-
problema no es que lleguen anglicismos, sino que se sacrifiquen/
ciéis, guau. / Los hiatos formados por dos vocales fuertes siguen
supriman otras palabras.
las reglas generales de acentuación. Ejemplo: teatro, caoba,
saeta. / En los hiatos formados por una vocal débil y una fuerte,
la tilde se coloca sobre la débil aunque no cumpla las reglas ge-
Análisis del texto nerales de acentuación. Ejemplo: raíz, maría, país, río...
14. Respuesta posible: Tesis: se halla al principio: el uso de angli-  5. Contienen diptongo: evocación, también, conferencia, alergias,
cismos no es perjudicial para la lengua, el problema radica en contaminación, prohibir . Contienen hiato: raíz, ideas, desoír, día,
si, por usarlos, se olvidan otras palabras de la propia lengua había, ortografía, morfología, grafía. Contienen triptongo: nin-
con significado similar y, por tanto, se pierden. Cuerpo argu- guna.
mentativo: se extiende desde «Ahí está “fútbol”...» hasta «es
alguien a quien se proscribe». Ejemplos: las palabras pas-  6. Acentuamos las palabras propuestas e indicamos si contie-
sword, knowhow, fake... constituyen ejemplos de anglicismos nen hiato (H), diptongo (D) o triptongo (T): cuatro (D), seis (D),
que pueden desplazar a palabras patrimoniales. Conclusión: benjuí (D), historia (D), oído (H), espontáneo (H), poesía (H), in-
retoma la tesis inicial, reformulando que el problema no radi- dependencia (D), ruido (D), alfeizar (D), náufrago (D), contribuís
ca en la incorporación sino en usar o no usar las palabras. (D), enviéis (T), eólico (H), transeúnte (H), vehículo (H), heroico
(D), huida (D), despreciéis (T), averiguáis (T), corría (H),
15. Copiamos las oraciones en las que aparecen los conectores tuétano(D), dúo (H), aéreo (H).
indicados y los sustituimos por otros equivalentes: «Fútbol»,
después de todo, llegó a donde no había nada. / Igualmente, abonó  7. Actividad individual.
su peaje. / No obstante, nos invaden ahora anglicismos que tenían
palabras equivalentes en español. / Por tanto, una persona «ba-
 8. La desinencia del pretérito imperfecto y del condicional de los
neada» en Internet (porque insulta, calumnia, miente, altera el
verbos de la segunda y tercera conjugaciones es -ía (comía y
diálogo o usa palabras soeces) es alguien a quien se proscribe.
comería, por ejemplo) Y la segunda persona del plural del
presente (queréis) se acentúa porque es un diptongo en el
que la vocal tónica es la vocal fuerte y se deben seguir las re-
Expresión glas generales de acentuación; en este caso es una palabra
llana acabada en vocal + s). En cambio, quisisteis no se acen-
16. Actividad personal. túa porque es una palabra llana acabada en vocal + s.
17. Actividad personal.  9. El diecisiete de diciembre llegamos a las islas Canarias. El diecinue-
ve recalamos en Santa Cruz de Tenerife y, finalmente, el veintiuno
18. Actividad personal y en grupo. dejamos atrás el último puerto español de la travesía y pusimos
rumbo a San Juan de Puerto Rico, nuestra primera escala en el
continente americano. Palabras con hiato: travesía. lleva tilde
La norma ortográfica (pág. 19) porque las vocales que están juntas son una vocal débil tónica
y una vocal fuerte, de modo que no se siguen las reglas de
 1. Marcamos en negrita la sílaba tónica y entre paréntesis si son acentuación. / Palabras con diptongo: diecisiete, diciembre, Ca-
agudas (A), llanas (LL) o esdrújulas (E): Fonética (E), anglicismo narias, diecinueve, veintiuno, puerto, Juan, nuestra. En este caso,
(LL), depredador (A), quizás (A), ambiental (A), estilístico (E), ba- no se acentúan porque la mayoría son palabras llanas acaba-
lompié (A), trabajo (LL), diálogo (E), verdad (A), víctima (E), nadie das en vocal o vocal + s. Juan es monosílaba.
(LL), cadáveres (E), rodeen (LL), coloquio (LL). Faltan palabras
sobreesdrújulas, por ejemplo: dígamelo, tráigaselos, próxima-
mente, pintándoselos, quédatelo... ACTIVIDADES (págs. 30-31)

 2. Césped, bidé, cómics, catedral, bíceps, alhelí, helicóptero, sutil,


Héctor, robots, súbito, entrégaselo, virrey, parís, fértil. La palabra
 3. Ello es que hay animales muy científicos / en curarse con varios  1. Respuesta libre. A modo de orientación, se adjuntan las defi-
específicos / y en conservar su construcción orgánica / como hábi- niciones de la página 20. monema: los monemas son las unida-
les que son la botánica, / pues conocen las hierbas diuréticas, ca- des mínimas con significado. Se clasifican en dos tipos:
tárticas, narcóticas, eméticas, / febrífugas, estípticas, prolíficas, / lexemas y morfemas. / lexema: Es la base léxica o parte fija que

18
Unidad 1 >  La palabra

aporta el significado léxico de la palabra. es común a las pala-  7. Respuesta libre. Aportamos un ejemplo de cada tipo de for-
bras de la misma familia léxica, pero no es necesariamente mación: derivados nominales: aparcamiento, existencia, blancu-
invariable (estudiar, estudiantil, estudio, estudioso, desestu- ra, arbitraje, seriedad, lealtad, modernidad. / derivados
diar...). / morfema flexivo: permite que las palabras variables de adjetivales: aceitoso, caluroso, orgulloso, barbudo, bebido,
la lengua (sustantivos, adjetivos...) se flexionen, es decir, creído, pianístico. / derivados verbales: aterrar, marear, aboto-
aportan información gramatical (género, número, persona, nar, ablandar, endulzar, humedecer.
tiempo y modo). Estudiosas, estudiantes, estudiamos, estu-
diaré, estudiase. / morfema derivativo: modifica el significado de  8. Siglas: fm (Frecuencia Modulada) / ccaa (Comunidades Autó-
los lexemas a los que se unen y permiten formar nuevas pala- nomas) / isbn (International Standard Book Number) / dvd
bras. Según la posición que ocupan respecto el lexema se (Digital Versatile Disc) / acrónimos: boe (Boletín Oficial del Esta-
clasifican en: prefijos, sufijos, interfijos. do) / cif (Código de Identificación Fiscal) / opa (Oferta Pública
de Adquisición) / módem (Modulator demodulator) / láser (Light
 2. cotiz- (lex) acion- (m. deriv. sufijo) -es (m. flex. plural) / deb- Amplification by Stimulated Emision of Radiation) / pib (Pro-
(lex) -er (m. flex. infinitivo) / ángel- (lex) -es (m. flex. plural) / ducto Interior Bruto) / talgo (Tren Articulado Ligero Goi-
entr- (lex) -e (m. flex. 3ª pers. sing presente subj.) / sol (lex) / coechea Oriol) / uci (Unidad de Cuidados Intensivos) / sónar
ansi- (lex) -os- (m. deriv. sufijo) -as (m. flex. femenino, plural) / (Sound Navigation and Ranging) / uvi (Unidad de Vigilancia
pie (lex) / contra (lex) / vol- (lex) -aba (m. flex. 1ª y 3ª pers. sing Intensiva) / vao (Vehículos de alta ocupación) / iva (Impuesto
pretérito imperfecto indic.) / sencill- (lex) -ez (m. deriv. sufijo) / sobre el valor añadido) / mir (Médico Interno Residente) / oms
sin- (lex) -fín (lex) / re- (m. deriv. prefijo) -coloc- (lex) -ad- (m. (Organización Mundial de la Salud).
deriv. sufijo) -o (m. flex. gen. masc).
 9. http: Protocolo de transferencia de hipertexto (HyperText
Transfer Protocol) / usb: Bus Universal en Serie (Universal
Formación de palabras Serial Bus) / gps: Sistema de Posicionamiento Global (Global
Positioning System) / www: Red informática mundial (World
 3. pre- (morfema) -historia (lexema) / des- (morfema) -confiar
Wide Web).
(lexema) / acelera- (lexema) -ción (morfema) / bich- (lexema)
-ejo (morfema) / i- (morfema) -legal (lexema) / re- (morfema)
-admitir (lexema) / joven- (lexema) -zuelo (morfema) / sub- 10. Respuesta libre. Aportamos algunos ejemplos: Impuestos: irpf,
iva, ibi... / Leyes: loe, lomce, lou...
/ Instituciones: Unicef, acnur,
(morfema) -suelo (lexema) / malet- (lexema) -ón (morfema) /
once, ocde, fmi, onu, oms.../ Partidos Políticos: pp, psoe, psc, icv,
anti- (morfema) -aéreo (lexema) / pos- (morfema) -grado (lexe-
cup... / programas de televisión: gh, qmd (qué me dices), csi (Crime
ma) / mallorqu- (lexema) -ín (morfema).
Scene investigacion), mhyv (mujeres, hombres y viceversa), cqc
prefijos: pre- (Indica ‘anterioridad local o temporal’, prioridad o (caiga quien caiga)...
‘encarecimiento’), des- (Denota ‘negación’ o ‘inversión del signi-
ficado’ de la palabra simple a la que va antepuesto), i- (Indica 11. Ped - (del griego paidós ‘niño’) + iatra (del griego, iatría, ‘mé-
‘negación’ o ‘privación’), re- (Significa ‘repetición’), sub- (Signifi- dico’) / lacrimó- (del latín lacrima, ‘lágrima) + geno (del griego,
ca ‘bajo’ o ‘debajo de’), anti- (Significa ‘opuesto’ o ‘con propie- ‘producir’) / a- (del latín, ‘sin’) + céfalo (del griego, ‘cabeza’) /
dades contrarias’), pos- (Significa ‘detrás de’ o ‘después de’) / homi- (del latín homo, ‘hombre’) + cida (del griego, ‘el que
sufijos: -ción (Forma sustantivos deverbales que expresan ac- mata’) / retro- (del latín, ‘hacia atrás’) + activo (del latín, ‘lleva-
ción y efecto), -ejo (Tiene valor diminutivo y despectivo, en do a cabo’) / en- (del griego, en, ‘dentro’) + -ciclo- (del griego,
sustantivos y adjetivos), -zuelo (Tiene valor diminutivo o despec- ‘círculo’) + ped- (del griego, pais ‘niño’) + -ia (del griego, ‘cua-
tivo), -ón (Forma sustantivos y adjetivos, derivados de sustanti- lidad) / orto- (del griego orthos, ‘correcto’) + odon- (del griego,
vos, adjetivos y verbos, de valor aumentativo, intensivo o ‘diente’) + cia (del griego, ‘cualidad’) / ptero- (del griego, ‘ala’)
expresivo), -ín (formador de gentilicios). + dáctilo (del griego, ‘dedo’) / filmo- (del griego film, ‘película’)
+ teca (del griego, ‘colección’) / agrí- (del latín ager, ‘campo de
 4. traspié (compuesta) / retomar (derivada) / confesionario (deri-
cultivo’) + col- (del latín colere, ‘referente al cultivo) + a (del
vada) / indomable (parasintética) / radioaficionado (compuesta) latín, ‘agente que realiza la ‘acción’) / carní- (del latín caro,
/ alunizar (parasintética) / epicolírico (compuesta) / replegar carnis, ‘carne’) + voro (del latín vorus, ‘el que se alimenta de’)
(derivada) / aconsejable (parasintética) / hemoglobina (com- / piró- (del griego, ‘fuego’) + mano (del griego manía, ‘locura’) /
puesta) / descafeinado (parasintética) / histórico (derivada) / peri (del griego, ‘alrededor de’) + cardio (del griego kardia,
porque (compuesta) / alumbrar (derivada) / asimismo (com-
‘corazón’) / helió- (del griego helios, ‘sol’) + grafo (del griego
puesta). graphein, ‘escribir’) rino- (del griego, ‘nariz’) + plastia (del
griego, ‘modelado’).
 5. Matasanos (médico) / pintamonas (pintor) / picapica (revisor)
/ picapleitos (abogado) / sacamuelas (dentista) / rapavelas
12. Actividad personal.
(sacristán) / tocapuertas (vendedor) / comecocos (psicólogo) /
chupatintas (oficinista) / tiracueros (zapatero) / sacacasillas
(‘hacer enfadar a uno’, aunque no lo contempla el dicciona- Origen del léxico
rio) / sacapotras (cirujano).
13. Emparejamos las palabras españolas y las latinas e indicamos
 6. despuntar (parasintética) / incómodo (derivada) / reverdecer
los cultismos (C) y las palabras patrimoniales (P):
(parasintética) / desmotivado (derivada) / alargado (parasintéti-
ca) / recibido (derivada) / rejuvenecer (parasintética) / aterrizar delicatus: delicado (C), delgado (P) / frigidus: frígido (C), frío
(parasintética) / anaranjado (parasintética) / ilógico (parasinté- (P) / clamare: clamar (C), llamar (P) / articulus: artículo (C),
tica) / antinacionalista (parasintética) / engrasar (parasintética). artejo (P) / strictum: estricto (C), estrecho (P) / operari: ope-
rar (c), obrar (P) / cathedra: cátedra (C), cadera (P) / collocare:

19
Unidad 1 >  La palabra

colocar (C), colgar (P) / legalis: legal (C), leal (P) / calidus: 21. Sinónimas: ceñir/sujetar, perfil/contorno, obstinado/tenaz, hus-
cálido(C), caldo (P) / litigare: litigar (C), lidiar (P). Todas las mear/fisgar, inclinar/desnivelar / homónimas: basar/vasar, vino/
palabras propuestas son dobletes. vino, barón/varón, vaya/valla, bienes/vienes / parónimas: peso/beso,
efecto/afecto, prever/proveer, recabar/recaudar.
14. Derivación / neologismo / préstamos / cultismo / compuesto /
acrónimo. 22. ateo/creyente: complementarias; caliente/frío: antónimas; abue-
lo/nieto: recíprocas; aceptar/rechazar: complementarias; caro/
15. Calzoncillo, pausa o descanso, negocio, efectivo, charlar, bo- barato: antónimas; aprobado/suspendido: complementarias.
cina, correo electrónico, de moda o tendencia, carpeta, gim-
nasio, éxito, superventas, estilo o aire, vínculo, clasificación, 23. Justificación personal. Se espera que el alumno identifique
de compras, correo no deseado. que todas son homónimas: traje/traje: homónima, porque un
término se refiere a la prenda de ropa y el otro, al verbo traer;
16. Según la lengua: anglicismos: container, baloncesto, show, ho- llama/llama: homónima porque uno se refiere al animal y el
bby, spray, ratón, rascacielos, spot, chutar / galicismos: parking otro, al verbo llamar; ay/hay: homónimas; un término se refiere
(esta del inglés), cruasán / náhuatl: coyote. Según el grado de a la exclamación y la otra, al verbo haber.
adaptación: prestamos adaptados: baloncesto, rascacielos, chu-
tar, cruasán, coyote, ratón / xenismos: container, show, hobby,
spray, spot, parking / Barbarismos: container, show, hobby, Evaluación (pág. 33)
parking y spot / Según el modo de incorporación: préstamos léxi-
co: container, show, hobby, spray, coyote, parking, spot, crua-  1. Respuesta posible: el autor defiende que siempre ha habido
sán, chutar, préstamos semántico : ratón, calco semántico : préstamos entre lenguas y seguirá habiendo mientras haya
baloncesto, rascacielos. lenguas vivas y dinámicas. Respuesta personal.

17. Actividad personal.  2. ¿Qué significa...? Respuesta posible: que la expresión ha sido
copiada de la lengua en la que penetra: el castellano la calca
del inglés, pero este del chino.
El significado de las palabras
¿En qué consiste...? Respuesta posible: adaptarla a las reglas
18. Respuesta libre. A modo de ejemplo definimos ambos térmi- morfológicas y ortográficas de la propia lengua.
nos. polisemia: las palabras polisémicas son aquellas que po-
seen varios significados. Por ejemplo, banco (para depositar  3. profusamente: excesivamente, sobremanera. / forjó: acuñó,
dinero, para sentarse...), cabo (título militar, extremidad, len- concibió. / perplejo: atónito, sorprendido. / designa: señala,
gua de tierra que penetra en el mar...). homonimia: son pala- identifica. /
bras homónimas aquellas cuyo significante es muy similar
pero su significado dista porque su origen etimológico es dife-  4. El fragmento se refiere a los préstamos léxicos.
rentes. Por ejemplo, aya/haya; echo/hecho; valla/vaya.
 5. Respuesta abierta y personal. Indicamos algunos ejemplos
19. Respuesta personal. Proporcionamos algunas de oraciones a para cada formación. Lexema: da, sin, general, total, lugar... /
modo de ejemplo: Este pájaro se ha roto una ala. / El ala dere- Lex. + morfema flexivo: últimas, décadas, lenguas, partes, idio-
cha del edificio está totalmente en llamas; La boca de incen- mas... / Lex. + morfema derivativo: especialidades, especificida-
dios está en medio de la calle / Por favor, come con la boca des, creación, naturalizando... / Lex. + morf. flex. + morf.
cerrada; Me duele muchísimo la cabeza. / Sin querer nos pu- deriv.: especialidades, / Morf. deriv. + lex.: internacional, ex-

simos en la cabeza de la manifestación; Me ha entrado algo portando, intercambiando... / Morf. deriv. + lex. + morf. deriv.:
en el ojo derecho y me escuece. / El agujero de la cerradura, enriquecimiento / Lex. + lex.: grecolatina.
donde se pone la llave, se llama ojo; ¿Qué número de pie tie-
nes? / Hemos alquilado una apartamento a pie de playa; Aún  6. Respuesta posible. Préstamos adaptados: escopeta, carnet, es-
no me han salido las muelas del juicio. / Para que un molino cáner / xenismos: walkie-talkie, / barbarismos: link, hardware,
funcione tiene suficiente con una muela de agua. spray.

20. En la ribera del río crecen los árboles. / Cada año cavo la tierra y  7. Préstamos: es una palabra esdrújula / inglés: es una palabra
planto un árbol. / Me gusta oír el canto de los pájaros. / Carece aguda acabada en -s. / cerebro: es una palabra llana acabada
de cultura, es muy basto. / Tiraba piedras a sus amigos con la en vocal. / espíritu: es una palabra esdrújula. / viva: es una
honda. / Esa rivera no lleva agua en verano. / Por las mañanas palabra llana acabada en vocal. / temor: es una palabra aguda
canto en la ducha. / El cabo desfilaba delante de los soldados.
/ que no acaba ni en vocal ni en -s ni en -n.
Un vasto panorama se divisaba desde allí. / No te bañes en la lagu-
na; es muy honda.  8. Respuesta personal. Las definiciones pueden comprobarse
en las páginas 23, 24, 27 y 28.
Son homógrafas: canto, honda.
Son homófonas: ribera/rivera; cavo/cabo; basto/vasto.  9. Actividad personal.

20
2 # De la palabra a la oración

 4. Respuesta posible: existía la creencia de que la sintaxis era


En contexto (pág. 35)
propia de los humanos, pero recientemente se ha demostrado
a) Respuesta abierta. Ofrecemos la siguiente: Las primeras pala- que las aves también emplean este sistema para comunicarse
bras que aprende a decir un bebé son sustantivos concretos, y que, si se altera el orden de los constituyentes, el mensaje
ya que tienen relación con objetos, animales y personas que le no se decodifica. También, se ha comprobado que la comuni-
rodean. cación entre primates no humanos y aves ha evolucionado
con el tiempo.
b) Respuesta libre. Proporcionamos algunos ejemplos de oxímo-
ron: [El amor es] «hielo abrasador, es fuego helado, / es herida
que duele y no se siente...» de Francisco de Quevedo. Cuestión de lengua
 5. ¿Qué tiempos verbales…?: Dado que estamos ante un texto ex-
PRACTICA CON LOS TEXTOS (págs. 36-50) positivo predominan los verbos en tiempo presente para pre-
sentar el hecho, la investigación, los resultados y las
conclusiones a las que se ha llegado, etc. También aparecen
Comprensión verbos en pretérito indefinido cuando se describen los descu-
brimientos de investigaciones anteriores, pero en menor me-
 1. ¿Cuál es el repertorio...? Respuesta orientativa: El carbonero dida.
común tiene un repertorio vocal con más de diez notas dife-
rentes que, combinadas de forma específica, le permite trans-  6. En la primera oración, que es una conjunción que introduce
mitir mensajes y comunicarse. una oración subordinada sustantiva y, en el seno de esta, no
cumple ninguna función. En la segunda oración, si es una
¿Qué descubrió...? Previamente a esta investigación, T. Suzuki conjunción que introduce una oración subordinada interroga-
había descubierto ya una serie de normas en los llamados de tiva indirecta. Tampoco cumple ninguna función dentro de la
estos pájaros y que, si no se seguían, el mensaje no podía ser oración subordinada. En la tercera oración, cuyo es un deter-
interpretado por otros carboneros. minante que introduce una oración subordinada de relativo y,
dentro de la oración subordinada, cumple la función de CN
¿Qué se creía...? Hasta el momento, se creía que la sintaxis era
[el significado del llamado].
propia y exclusiva de los humanos, pero gracias a este estudio
se sabe que puede ser un mecanismo de comunicación bási-
co y común para distintos animales.
 7. Oraciones en voz pasiva: Un llamado específico era combinado
con otra nota. / Pero cuando el mensaje era invertido artificial-
Por lo tanto, ¿a qué conclusión...? La principal conclusión a la mente (D-ABC), no tenía significado para ellos.
que llegan es que la sintaxis ha evolucionado varias veces y
En voz activa: Combinaban este llamado con otra nota. / Pero
de forma independiente, no solo en los humanos. Esta evolu-
cuando invertían el mensaje artificialmente (D-ABC), no tenía
ción puede ayudar a comprender la que se ha dado también
significado para ellos.
en la sintaxis de los humanos.
Respuesta libre.
 2. Concretamente, se explicita en el párrafo cuarto. El ejemplo
concreto que demuestra que la alteración del orden de las
notas da lugar a la incomunicación es el siguiente: ABC-D, Cuestión de léxico
en este orden, significa asegurarse de que no hay un depre-
dador y pueden acudir al lugar desde donde llaman. Pero si  8. Las palabras que aparecen entre paréntesis y en cursiva son
se altera (D-ABC), el carbonero no entiende el significado «parus major», es el nombre científico en latín que se ha otor-
del mensaje. gado al carbonero común. Todos los animales tienen, además
de su nombre en las diferentes lenguas, un nombre en latín,
universal y común, que es el que suelen emplear científicos y
 3. El carbonero común es un ave que se encuentra en Europa, Asia
estudiosos.
y África: falsa. El carbonero común es un ave que se en-
cuentra en Europa y Asia. / El equipo de investigación está in- Comenta otros usos…: encerrar aclaraciones o incisos; sepa-
tegrado por japoneses , suecos , suizos y alemanes : falsa. El rar las acotaciones en una obra de teatro.
equipo de investigación está integrado por japoneses, sue-
cos y suizos. / La combinación de sonidos ABC-D y la de D-ABC  9. Respuesta libre, aunque se espera que los alumnos señalen
tienen el mismo significado para el carbonero común: falsa. La que el término “común” en este contexto significa “muy fre-
combinación de sonidos ABC-D y la de D-ABC tienen signifi- cuente”, es decir, una especie de carbonero bastante abun-
cados distintos para el carbonero común. / Toshitaka Suzuki y dante. Otros ejemplos que incluyan este término son “abeto
David Wheatcroft son autores del estudio: verdadera. / Enten- común”, “champiñón común”, “lobo común”, etc.
der la evolución de la sintaxis en las aves no puede ayudar a
comprender la evolución de la sintaxis humana: falsa. Entender 10. Tecnicismos: ave común, llamados, investigación de campo,
la evolución de la sintaxis en las aves puede ayudar a com- sintaxis de la composición, primates no humanos...
prender la evolución de la sintaxis humana.

21
Unidad 2 >  De la palabra a la oración

11. transmiten: comunican, transfieren, otorgan / usada: emplea-


da, utilizada, que sirve / sugiere: propone, insinúa, expone /
La norma ortográfica (pág. 39)
única: exclusiva, particular / demuestra: constata, evidencia,
 1. Reescribimos las oraciones con el término correcto. Si tuviera
prueba / aseguró: afirmó, constató, aseveró.
el valor suficiente, se cortaría el pelo a
sí misma. / Quien respon-
dió la pregunta no sabía quién la había formulado.
/ Aún espero,
12. Gentilicios: croata, ruso, hawaiano, amsterdamés, luxembur-
aun siendo tú quien dijo que no lo hiciera. / No se conoce el para-
gués, bagdadí, napolitano, holandés, zamorano, tunecino, is-
dero del prófugo, ni si ha cruzado las fronteras de la Unión Eu-
landés, vietnamita, ginebrino, noruego, neoyorquino, peruano,
ropea. / Quiero saber qué te respondió Verónica. / Andrés tiene
parisino, madrileño, salamantino, zaragozano, cacereño, car-
auténtica debilidad por el té negro y por la manzanilla. / El profe-
tagenero, gaditano, pacense (badajocense), catalán, extreme-
sor de piano prefiere que no toque ningún solo porque no se fía de
ño, gerundense, oscense, leridano, palentino.
mí. / Los funcionarios salieron a manifestarse porque querían
Los sufijos más habituales son –és, -ense, -ano, -eño, -ino… conseguir más derechos de los que se les había concedido.
Los gentilicios gaditano y pacense, por ejemplo, son diferentes
 2. Copiamos las oraciones propuestas añadiendo las tildes nece-
de su topónimo actual. El primero tiene su origen en el fenicio
sarias: La próxima vez que él me invite, sí aceptaré su invitación. /
gdr ‘ciudad amurallada’, que se tomó en latín como Gadir y
Pero tú no insistas más. / A mí me preocupa que el índice inflacio-
Gades; en árabe se adaptó como Qādis y esta forma nos lleva
nista se dispare aún más esta mañana en las bolsas europeas. ¡Has-
hasta la forma actual. El segundo procede del nombre que Ba-
ta cuándo continuará esta situación! / El trampantojo de la bóveda
dajoz recibió en época de Augusto, Civitas Pacensis.
de la iglesia no estaba aún terminado. ¿Cómo iban a aceptar los
Respuesta abierta. feligreses tres o cuatro meses de demora en la ejecución de las
obras? / ¡Qué vida la suya! ¡Cuántas vicisitudes! Ya sé que la des-
gracia se cierne sobre los más débiles, mas si hubiera justicia, su
Análisis del texto situación cambiaría. / Puede que no te dé miedo. A mí no me gusta.
Tan solo pido que no lo hagas más y que obedezcas a tu hermano
13. La lectura propuesta cumple con los requisitos propios de un mayor cuando estás solo con él.
texto periodístico: en primer lugar, consta de título y entradilla
(ambos destacados en negrita); en segundo lugar, es objetivo,  3. Hazmerreír, rioplatense, lavapiés, tiralíneas rodapié, plácida-
puesto que no se plasma la opinión del autor en ningún mo- mente, veintiséis, tómalos, anglosajón, ágilmente, petigrís, al-
mento y los resultados y conclusiones corroborados con datos tisonante, aguamarina, sociolingüístico, turbodiésel.
científicos y fuentes de autoridad; y en tercer lugar, el tema
que se trata es de interés general, de ahí que el registro sea  4. Acentuamos los latinismos propuestos y proponemos un
estándar y formal, para poder ser comprendido por un públi- ejemplo: Accésit: Este año solo ha habido accésit para el pri-
co amplio. mer premio. Déficit: En España, el déficit público está com-
puesto por el déficit del Estado, el déficit de la Comunidades
14. Se sustituyen los conectores marcados. En algunos casos ha Autónomas y el déficit de los ayuntamientos. Réquiem: El Ré-
sido necesario modificar la puntuación: El estudio [...] fue pu- quiem de Mozart se basa en los actos litúrgicos. Factótum:
blicado enNature Communications; también señala que las aves Confió a su factótum la elaboración de la lista de invitados.
combinan sus llamados [...]. / En un estudio anterior, el autor Memorándum: En el memorándum de este año solo constan los
principal mostró que las aves utilizan estas llamadas complejas datos previos a la fusión de la empresa. Etcétera: En el merca-
como “palabras” que transmiten diferentes significados; asimismo dillo del domingo había de todo: libros, discos, álbumes, etc.
se preguntó si también podrían usarse juntas[...]. / Pero cuando el Códex: el Códex Emilianense se considera la primera muestra
mensaje era invertido artificialmente (D-ABC), no tenía significado de la lengua romance peninsular castellana. Superávit: Con la
para ellos, por lo tanto no respondían. recuperación económica, la economía del país tendrá superá-
vit. Referéndum: Escocia celebró un referéndum en 2014 para
decidir sobre la independencia del país. Summum: Nuestros
Expresión vecinos son el summun de la hospitalidad. De incógnito: El
actual rey viajó de incógnito a Italia durante el mes de marzo.
15. Actividad en grupo. Currículum víitae: Tu currículum vítae debe transmitir clara-
mente tu trayectoria laboral.
16. Síntesis del vídeo: En este breve fragmento se resumen y explican
las técnicas que emplea el alcaudón para atraer a sus presas. A  5. Anotamos las adaptaciones al castellano de las palabras pro-
este pájaro se le llama también el imitador, ya que esa es la princi-
puestas: champú, carné, mitin y mitín, estándar, fuagrás,
pal herramienta que emplea para engañarlas.
chófer, pádel, mánager, plató, cruasán, travelín, récord, capó,
—— ¿Qué recurso emplea...? El alcaudón imita el canto de sus po- chóped, buqué, cabaré, baipás, capuchino.
tenciales presas para atraerlas hacia su terreno y cazarlas.
 6. Copiamos las oraciones propuestas añadiendo las tildes nece-
—— ¿Qué es la «voz de contacto»...? Los especialistas lo resu- sarias: No se celebró la asamblea por falta de quórum. /Devuélve-
men como un sonido cuyo significado es «no hay enemi- me el compás y los bolígrafos que te presté ayer rápidamente. /
gos a la vista, todo está tranquilo y puede seguir Necesitarás un cúter para cortar el celofán. / Aun los más listos
comiendo». se han equivocado en esta ocasión. / Añadiré esa información en un

—— ¿Por qué la ayuda...? Porque es pasivo; el rosal sirve al al- ítem al final del documento. / La habitación de matrimonio está en

caudón para empalar la presa cazada y poder comérsela. la parte superior del dúplex. / El árbitro advirtió que estaba pro-
hibido dar puntapiés durante el partido. / El ordenador se actuali-

17. Respuestas personales y actividades en grupo. zó automáticamente después de instalar el módem. / Tras una

22
Unidad 2 >  De la palabra a la oración

broma de mal gusto se convirtió en el hazmerreír del instituto. / las piedras redondas con que estaban asfaltadas (SA) las ca-
No he respondido ninguna pregunta del examen de Bioquímica. / lles. Mis pisadas huecas, repitiendo sus sonidos en el eco de
Volvió a llegar tarde al trabajo y su jefe le dio un ultimátum. / las paredes teñidas por el sol del atardecer (SPrep).
Nunca confíes un secreto a Laura; siempre fue una correveidile.
 6. Señalamos los complementos del verbo e indicamos el tipo a
  7. Actividad individual. continuación: He escrito varias postales (CD) a Eva (CI) duran-
te el verano (CCT). / Mi vecino vende muy barato (Pvo) su mo-
nopatín (CD). / Los padres de mis amigos son muy exigentes
ACTIVIDADES (págs. 50-53) (Atr). / Me arrepiento de mi comportamiento egoísta (CR). /
Picasso, célebre artista (Aposición) , nació en Málaga (CCL). /
Los cuadros fueron expuestos por los galeristas (Ag). / Los co-
Las categorías gramaticales  (pág. 50) rredores se sitúan cerca de la salida. (CCL).

Complementos del nombre: de mis amigos, egoísta, célebre ar-


 1. Respuesta libre. Se completa el texto a modo de ejemplo. A tista. Complementos del adjetivo: muy (muy barato y muy exi-
la mañana siguiente Frodo despertó lentamente (Av), sin-
gente). Complementos del adverbio: de la salida.
tiéndose descansado y (Conj) bien. Caminó a lo largo de las
terrazas que dominaban las aguas frías (A) del Bruinen y
 7. Respuesta libre. Aportamos un ejemplo de cada tipo de cons-
observó el paisaje (N) pálido y fresco que (Pron) se elevaba
trucción: He comprado un libro a mi amiga. / Los niños llega-
por encima de las montañas, proyectando los (Det) rayos
ron nerviosos del colegio. / Luis vive lejos de la ciudad. / María
oblicuos a través de la espesa (A) niebla de plata; el rocío
estaba harta de su trabajo. / Las cartas fueron entregadas por
refulgía en (Prep) las hojas amarillas y las telarañas colga-
el cartero en su destino.
ban (V) en los arbustos.
 8. Indicamos si las afirmaciones propuestas son verdaderas o
 2. Las (Det) cumbres (N) de (Prep) Sagrado (N) eran (V) her- falsas. El Pvo y el Atr concuerdan en género y número con el su-
mosas (A), de (Prep) un (Det) azul (N) profundo (A), terrible
jeto: verdadera. / El Pvo puede sustituirse por el pronombre lo:
(A), ciego (A). El (Det) sol (N), alto (A) y (Conj) redondo (A),
falsa. / El CC siempre es un sintagma preposicional: falsa. / El CR
como (Conj) una (Det) pupila (N) impertérrita (A), reinaba
viene exigido por algunos verbos que lo necesitan para completar
(V) ahí (Av). En (Prep) la (Det) neblina (N) del (Prep+Det)
su significado: verdadera.
amanecer (N), cuando (Av) aún (Av) no (Av) se oía (V) el
(Det) zumbido (N) de (Prep) las (Det) moscas (N) ni (Conj)
 9. En el bosque mediterráneo abundan los pinos. Prueba: *El pino
crujido (N) alguno (Det), Lope (N) solía (V) despertar (V),
abunda. / Necesitamos un buen diccionario de lengua. Sujeto elíp-
con (Prep) la (Det) techumbre (N) de (Prep) barro (N) enci-
tico (nosotros). Prueba: *Pedro necesita un diccionario. / Una de
ma (Av) de (Prep) los (Det) ojos (N).
las mejores cualidades es la tenacidad. Prueba: *la tenacidad y la
paciencia lo son. / A nosotros no nos importa su impuntualidad.
Prueba: *no nos importan sus impuntualidades. / ¿Conoces los
Los sintagmas y sus funciones  (pág. 50) nuevos modelos del diseñador? Sujeto elíptico (tú). Prueba:
*Vosotros, ¿conocéis los nuevos modelos...?
 3. Los meses de primavera (SN) / muy antigua (SA) /despacio (SAv) /
de parís(SPrep) / un fósil marino (SN) / Quedaré con mis compa-
ñeras en la biblioteca (SV).
10. Rocío llegará de madrugada: enunciativa. Interrogativa: ¿Rocío
llegará de madrugada? / Ojalá llegues a tiempo: desiderativa.
Los sintagmas que forman parte de otro superior son de prima- Imperativa: Tienes que llegar a tiempo. / ¡Habla más bajo!: exhor-
vera, muy, marino, con mis compañeras, en la biblioteca. tativa. Enunciativa: Le pido a Juan que hable más bajo. / Los
exámenes acaban esta semana: enunciativa. Dubitativa: No estoy
 4. Esta película de terror (SN); de terror (Sprep) debe de ser muy seguro de que los exámenes acaben esta semana. / Quizá va-
entretenida (SA); muy (SAv). / El árbol del jardín (SN); del jar- yamos juntas a la biblioteca: dubitativa. Exclamativa: ¡Iremos
dín (SPrep) está (SV) florido (SA) en primavera (SPrep). / juntas a la biblioteca!
Cerca de mi casa (SAv); de mi casa (Sprep) vive (SV) el alcal-
de de la ciudad (SN); de la ciudad (Sprep). / Los buenos mo- 11. Predicativas: El agua fluye cristalina (Pvo) por el río (CCL); Envió
dales (SN) son indicativo de educación (SA); de educación raudo (Pvo) un telegrama (CD) a toda su familia (CI); Encontré a
(SPrep). / Nadie (Pron - SN) conoce (SV) las verdaderas in- mi compañero de piso (CD) dormido (Pvo) en el sofá (CCL).
tenciones de los políticos (SN); verdaderas (SA), de los políti-
Copulativas: Parece muy interesante (Atr) esta película ; Candela
cos (SPrep).
siempre (CCT) fue una alumna aventajada (Atr); Las quejas son
habituales (Atr) entre estos vecinos (CCComp).
 5. Respuesta libre. Completamos el texto con sintagmas a modo
de ejemplo: Era la hora en que los niños (SN) juegan en las
12. Anoche llovió copiosamente: impersonal, predicativa, intransiti-
calles de todos los pueblos, llenando con sus gritos la tarde.
va, activa. / En esta empresa se trabaja a destajo: impersonal,
Cuando aún las paredes negras reflejan (SV) la luz tardía (SA)
predicativa, intransitiva, activa. / El puente fue diseñado por un
del sol. Al menos eso había visto en Sayula, todavía ayer, a esa
ingeniero extranjero: personal, copulativa, intransitiva, pasiva. /
misma hora. Y había visto también el vuelo de las palomas
Expuso a los trabajadores los motivos de su despido: personal,
rompiendo el aire denso (SA), sacudiendo sus alas como si se
predicativa, transitiva, activa.
desprendieran del día. Volaban (SV) y caían sobre los tejados,
mientras los gritos de los niños revoloteaban y parecían teñir-
13. Llaman por teléfono: eventual con 3.ª pers. pl. / Mañana nevará
se de azul en el cielo del atardecer. Ahora estaba muy lejos
en el norte del país: unipersonal (meteorología o fenómenos
(SAv), en este pueblo sin ruidos. Oía caer mis pisadas sobre

23
Unidad 2 >  De la palabra a la oración

naturales). / En esta playa siempre hay mucha gente: gramaticali- vuestro (Det) criterio (N) [CR].
zada. / Se ha elegido a los delegados de curso: impersonal con
se. / Es demasiado temprano para visitar a los abuelos: uniperso- tus (Det) conclusiones (N) me (Pron) [CI] parecen (V- nú-
nal. / En verano amanece más temprano: unipersonal (meteorolo- cleo) muy (Av) interesantes (A) [Atr]. / el (Det) alcalde (N)
gía o fenómenos naturales). de (Prep) la (Det) localidad (N) detenido (A) [CN (de al-
calde)] fue citado (V-núcleo) por (Prep) el (Det) juez (N)
14. Se citó al acusado para declarar ante el juez: Impersonal. / Se [CAg].
vende un terreno en esta finca: Pasiva refleja. / En Navidad se Marta (N), mi (det) prima (N) [Aposición], ha estudiado (V-
envió un lote de regalos a todos los empleados: Pasiva refleja / Se núcleo) un (Det) año (N) en (Prep) Milán (N) [CCL].
comenta que subirán los precios: Pasiva refleja. / Se necesita una
enfermera con experiencia: Pasiva refleja. / En esa escuela se dedi-
 3. Los (Det) padres (N) viven (V) en (Prep) León (N) y (Conj) sus
ca mucho tiempo a los idiomas: Pasiva refleja. / En aquella tienda
(Det) hijos (N) estudian (V) Medicina (N) en (Prep) Salamanca
se compran joyas de oro y plata: Pasiva refleja
(N): oración compuesta por coordinación copulativa.

15. Actividad personal. N osotras (Pron) llevaremos (V) las (Det) bicicletas (N) y
(Conj) vosotros (Pron) llevad (V) los (Det) mapas (N) de
16. Se auxilió a los náufragos: transpositor de impersonalidad. / Mi (Prep) la (Det) zona (N): oración compuesta por coordina-
hermana pequeña se lava sola las manos: pronombre reflexivo. / ción copulativa.
Se los regalé el día de su aniversario:
pronombre personal. / Se
han suspendido todas las clases hasta el próximo miércoles: mar-
cador de pasiva. / Antes de salir a la calle siempre se seca el pelo: Evaluación (pág. 55)
pronombre reflexivo. / Mario y Elena se escriben unas cartas
larguísimas: pronombre recíproco. / Mis abuelos se acuerdan  1. Respuesta abierta. A modo de ejemplo, sintetizamos el texto
mucho del pueblo en el que nacieron: formante del lexema ver- y proporcionamos otro título: «Afinar las letras», «Afinadores
bal. / La verja de aquella casa se abrió con un sonido estremece- de textos». En este texto se defiende la necesidad de apren-
dor: marcador de pasiva. der gramática tomando como argumento principal una com-
paración entre la gramática como herramienta para un
17. [Mis hermanos y todos sus amigos llegaron] Prop. 1 [cuando ya escritor, y el solfeo como herramienta para el estudiante de
nadie losesperaba] Prop. 2: compuesta. / Las plazas libres de música o intérprete. Ciertamente, uno puede tocar un instru-
aparcamiento del primer piso están muy solicitadas desde hace mento «de oído» y se puede escribir sin estudiar las normas
tiempo por algunos vecinos extranjeros: simple. / [Nada dicen gramaticales, pero el resultado será incompleto, pues le fal-
los manuales al respecto] Prop. 1, [pero hay un uso social en tarán las normas básicas para tocar o escribir de forma ar-
España] Prop. 2 [que llama la atención en las costumbres de moniosa.
otros países transpirenaicos] Prop. 3: compuesta. / Reciente-
mente se ha visto la gran importancia de la ambientación en rela-
 2. Escépticos: incrédulos, recelosos. / referencia: guía, pauta,
ción con la actividad biológica , sobre todo en la industria
ejemplo. / se burlaba: se reía, se mofaba. / inutilidad: impro-
farmacéutica: simple.
ductividad, ineficacia. / razonar: argumentar, pensar. / instinti-
vo: involuntario, inconsciente.

Análisis de la oración (pág. 53)  3. Coartar: limitar la expresión de la creatividad. / cercenar: cor-
tar la expresión y recreación del estilo personal. / templar:
 1. Muy (Av-CAv) cerca(Av-SAv) de (Prep-r) la (Det) farmacia (N- disponer un instrumento de manera que pueda producir con
SN): Sintagma adverbial, núcleo «cerca». / Salieron (V) satisfe- exactitud los sonidos que le son propios. /atañer: incumbir,
chos (Pvo) de (Prep-r) las (Det) aulas (N-SN) (SPrep): Sintagma
corresponder. / disonar: sonar desapaciblemente, faltar a la
verbal, núcleo «salieron». / Unos (Det) modelos (N-SN) muy consonancia y armonía.
(Av) sofisticados (A-SA): Sintagma nominal, núcleo «modelos».
/ Implantación (N-SN) de (Prep-r) una (Det) nueva (A-CN) técnica
 4. Escépticos: es una palabra esdrújula. / corsé: es una palabra
(N-SN) quirúrgica (A-CN) (SN-CN): Sintagma nominal, núcleo
aguda acabada en vocal. / televisión: es una palabra aguda
«implantación».
acabada en -n. / solfeo: no se acentúa porque es una palabra
llana acabada en vocal. / qué: lleva tilde diacrítica para dife-
 2. El (Det) estudio (N) [Sujeto] trata (V-núcleo) sobre (Prep) los
renciar este interrogativo del pronombre relativo o conjunción.
(Det) riesgos (N) laborales (A-CN) [CR]. / [1ª p. pl.] estamos
/ además: es una palabra aguda acabada en -s. / oído: se acen-
(V-núcleo) hartos (A) [Atr] de (Prep) su (Det) mala (A-CN)
túa porque contiene un hiato.
educación (N) [CA].
Aquel (Det) actor (N) malagueño (A-CN) ha sido nombrado  5. Muchos (Det) escépticos (A) del (Prep+Det) lenguaje (N) con-
(V-núcleo) hijo (N) predilecto (A) de (Prep) la (Det) ciudad (N) sideran (V) que (Conj) la (Det) gramática (N) supone (V) un
[Pvo] por (Prep) el (Det) alcalde (N) [CAg]. (Det) corsé (N), una (Det) obligación (N) absurda (A) para
(Prep) el (Det) escritor (N) o (Conj) el (Det) periodista (N),
La (Det) limonada (N) es (V-núcleo) una (Det) bebida (N)
para (Prep) el (Det) abogado (N) o (Conj) para (Prep) el (Det)
agradable (A) [Atr] al (Prep+Det) paladar (N) [CA].
presentador (N) de (Prep) televisión (N).
Eduardo (N) llamó (V-núcleo) antipático (A) [Pvo] a (Prep) su
(Det) compañero (N) de (Prep) trabajo (N) [CD].  6. Hay varias respuestas posibles: se tendrá más que decir...; puede
interpretarse también...; se trata de música afinada...
[1ª p. sg.] confío (V-núcleo) plenamente (Av) [CC] en (Prep)

24
Unidad 2 >  De la palabra a la oración

 7. La gramática supone una obligación absurda (CD) para el es- gramática (N): oración simple en la que cabe destacar el
critor (CI). / La gramática no está de moda (Atr). / La novela CR de la necesidad de la gramática. / Un (Det) buen (A)
Niebla fue escrita por Miguel de Unamuno (CAg). / El resulta- autor (N) de (Prep) novelas (N) no (Av) aprende (V) en
do musical coincide con las normas (CR) del más educado (Prep) la (Det) escuela (N) gramática (N) y (Conj) sintaxis
intérprete de cámara (CN) [CN]. (N): oración simple.

 8. Miguel de Unamuno (N) se burlaba (V) de (Prep) la (Det)


necesidad (N) del (Prep+Det) estudio (N) de (Prep) la (Det)  9. Respuesta abierta.

25
3# La oración compuesta. La coordinación

car a qué clase de objeto correspondía. Así lo distinguían de


En contexto (pág. 57)
otras palabras de igual ortografía.
a) Actividad en grupo y respuesta personal.
 2. Respuesta posible. ¿Cómo surgieron...? Como una escritura
b) Actividad individual. que perpetuaba una lengua, la egipcia, que representaba vi-
sualmente un objeto. ¿Por qué surgieron...? Porque necesitaba
signos y símbolos para fijarla.
Practica con los textos (págs. 58-61)
 3. Respuesta posible: Champollion describe la lengua jeroglífica
egipcia como un sistema verdaderamente complejo, en la que se
Comprensión han ido sumando innovaciones y complejidades, no solo a la es-
tructura de la frase, sino incluso a la palabra; es decir, a lo largo de
los años los signos han ido cargándose simbólica, figurativa y fo-
 1. ¿Qué son...? Respuesta orientativa: Los jeroglíficos son
néticamente de tal modo que descifrarla deviene una ardua labor.
símbolos que representan visualmente un objeto.
¿Se mantuvieron...? A pesar de que los principios básicos del  4. Respuesta posible: El sistema inicial de escritura primero fue
sistema jeroglífico se mantuvieron, este compitió con otros figurativo: el signo representa un objeto, después se expresan
sistemas de escritura, que fueron evolucionando de forma di- nuevos significados, por ejemplo, conceptos abstractos (logo-
ferente e independiente. grama) y, por último, se pasa a la escritura fonética mediante
signos que representan sonidos y fonemas.
¿Qué significa...? Significa que algunos signos se empleaban
también para hacer referencia a elementos relacionados con  5. En este ejercicio se trata de fomentar la creatividad y la capa-
la idea que representaban. Por ejemplo, el logograma del sol cidad de síntesis del alumno. A modo de ejemplo, algún título
representaba directamente el astro rey, pero simbólicamente orientativo podría ser «Los inicios de los alfabetos modernos»
podía representar el concepto temporal «día». o «La evolución de los jeroglíficos egipcios».

¿Con qué inconveniente...? El principal problema de la escritura  6. En este caso, se trata de que en el mapa conceptual se desta-
fonética fueron las palabras que se escribían igual. Lo resol- que el proceso evolutivo de la escritura jeroglífica. Se adjunta
vieron incluyendo al final de cada palabra un signo para indi- un mapa conceptual a modo de ejemplo:

Lengua egipcia

Oral Escrita
gran distancia

sistema ideográfico primero


sufrió una en que cada símbolo
representa visualmente un Jeroglíficos
profunda evolución elemento u objeto.

1 signo = 1 elemento
Grandes diferencias
dialectales entre
diferentes regiones. insuficiente

Se usan también de
P. ej: Sol = “astro” y “día” forma simbólica

Confusión Problema Se añade escritura


entre aquellas palabras que
fonética
se escribían igual

añadieron
signo que indicaba a qué
Signos determinativos
tipo de objeto se refería

26
Unidad 3 >  La oración compuesta. La coordinación

 7. Reescritura posible: Champollion, el primer científico en des- no-inhumano, distinto-indistinto, realidad-irrealidad, existente-
cifrarlos, escribió que la escritura jeroglífica es un sistema inexistente, etc. Otra palabra con prefijo es re-presentación, por
complejo, una escritura que es a la vez figurativa, simbólica y ejemplo.
fonética en un mismo texto, e incluso se atrevió a decir que en
una misma palabra. 13. La respuesta correcta es «Las palabras escriba, escribano,
escribiente son palabras derivadas que pertenecen a la mis-
ma familia léxica». Sonido: sonar, sonoridad, sonoro, sonidis-
Cuestión de lengua ta... / Lengua: lenguaje, lingüista, lingüística/o... / Sistema:
sistemático, sistematizar, sistémico... /Pronunciar: pronun-
 8. En este texto predomina el pretérito indefinido (apareció, se cia, pronunciación, pronunciamiento…
desarrolló, experimentó, se escribió, evolucionaron, hubo, se man-
tuvieron...), dado que se está explicando una serie de aconte- 14. Reproducimos las oraciones sustituyendo las expresiones
cimientos ocurridos en el pasado. En menor medida, se subrayadas por los adverbios correspondientes: La lengua
emplea el pretérito imperfecto (era, tenía, había...) en aquellos egipcia se desarrolló continuamente. / Los jeroglíficos evolu-
puntos en los que se describen determinadas situaciones que cionaron diferentemente. / Uno de ellos consistió en usar los
suceden en un pasado relatado. signos simbólicamente. / Existían conceptos abstractos que no
podían representarse gráficamente. Los adverbios acabados

 9. Las perífrasis verbales se caracterizan por ser construccio- en -mente que aparecen en el texto son directamente, simbóli-
nes sintácticas formadas por dos verbos que forman una camente, perfectamente. Equivalen a las expresiones de forma

única unidad léxica, y que a su vez son el núcleo del predi- directa, de forma simbólica y de forma perfecta, respectiva-
cado. La estructura suele ser Verbo 1 [auxiliar] + (preposi- mente. Los adverbios correspondientes a los adjetivos pro-
ción) + Verbo 2 [auxiliado]; la preposición puede aparecer o puestos son profundamente, habitualmente, igualmente,
no. Algunas de las estrategias que se espera que empleen invariablemente, rápidamente, ágilmente, temporalmente,
los alumnos son, por un lado, detectar que uno de los dos diariamente, espaciadamente.
verbos pierde su significado original (el auxiliar) y el conjunto
adquiere un significado nuevo; por otro, que el verbo auxilia- 15. d. e. p.: descanse en paz / fig.: figura / pág.: página / hab.: ha-
do no puede sustituirse por un pronombre, por ejemplo: bitantes / Cía.: compañía / cta.: cuenta / et al.: y otros / fra.:
quiero salir al balcón quiero esto [salir al balcón] frente factura / JJ. OO.: Juegos Olímpicos / N. del T.: Nota del tra-
a sigue leyendo *sigue esto. Los ejemplos de perífrasis ductor / P. D.: Posdata / p. ej.: por ejemplo.
verbales que aparecen en el texto son llegaron a crear (Perí- Pueden considerarse siglas, JJ. OO. y P. D.
frasis terminativa), podían representarse (Perífrasis de posibi-
lidad), pasó a designar (Perífrasis ingresiva), podía emplearse
(Perífrasis de posibilidad), terminaron por desarrollar (Perí- Análisis del texto
frasis terminativa), empezaron a utilizar (Perífrasis incoativa),
podían confundirse (Perífrasis de posibilidad), debería decir
16. Se trata de que el alumno identifique cada una de las partes
(Perífrasis de obligación). especificadas del reportaje: Tema que trata: la complejidad
de los jeroglíficos derivada de su evolución. / Destinatario:
10. Notable: importante, profunda; profunda : verdadera, ulte- público general, no especializado. / Intención comunicativa:
rior; ingenioso : inteligente, agudo; cautivador : atractivo, divulgar la información e informar. / Grado de especificidad del
atrayente. vocabulario: aparecen algunos tecnicismos y el registro es
formal. / Uso de ejemplos y reformulaciones: se emplean ejem-
plos en los diferentes tipos de innovaciones en los jeroglífi-
Cuestión de léxico cos (valor simbólico y determinativo). / Inclusión de citas:
aparece una única cita en el cierre del texto. / Grado de difi-
11. Las definiciones que proporcionamos son las que aparecen cultad y de comprensibilidad del texto: aunque va dirigido al
en el Diccionario de la Real Academia Española: copto: público en general, la revista en la que se publica no es de
Lengua camítica procedente de la última etapa del egipcio, alcance universal, puesto que se requiere cierto nivel de in-
que se habló hasta el siglo VII d. C. y que pervive como terés y de formación.
lengua litúrgica de la Iglesia copta (el texto aporta varian-
tes). / vigente: Dicho de una ley, de una ordenanza, de un 17. En el primer párrafo encontramos los siguientes:
estilo o de una costumbre, que está en vigor y observancia.
por ejemplo: es ejemplificativo (dicho de otra manera, es de-
/ pilono: remite a pilón: Pórtico de forma trapezoidal, muy
común en los templos del antiguo Egipto. /antílope: Cada cir). / pues: es causal (porque).. / sin embargo: es contraargu-
uno de los mamíferos rumiantes de cornamenta persistente. mentativo (pese a ello, no obstante). / además: es un conector
/ ubre: En los mamíferos, cada una de las tetas de la hem- aditivo (asimismo, también, por otro lado).
bra. / sortear: Evitar con maña o eludir un compromiso,
conflicto, riesgo o dificultad.
Expresión
12. Reproducimos las palabras del texto con los prefijos in-/im- y
separamos el prefijo del lexema: in-mutabilidad, in-variable, im-
18. Actividades personales o en grupo.
posible, in-suficiente, in-conveniente.
Este prefijo añade el signi-
ficado opuesto al lexema; es decir, lo convierte en un contrario 19. Actividades personales o en grupo.
de la palabra original. Otras palabras del texto que admiten
estos prefijos son habitual-inhabitual, utilizar-inutilizar, huma- 20. Actividades personales o en grupo.

27
Unidad 3 >  La oración compuesta. La coordinación

a un hospital en ese momento, su hija se sentiría desamparada, como si


La norma ortográfica su madre le infligiera la pena máxima: el abandono.

Ortografía: uso de b/v y g/j   (pág. 61) Actividades (PÁgs. 67-69)


 1. Subcutáneo, subvertir, obstetricia, advenedizo, busto, bizquear,
bivalvo, bienmesabe, eventual, vizcondado, virrey, bisnieto, ábside,
La oración compuesta 
vudú, tremebundo, movilidad, bubónico, buró, beneplácito, absti-
nencia, vicerrector, biopsia.  1. Oraciones simples: Lucía y Elena se compraron la misma camisa; No
voy a decir nada más; Mi madre y mi profesora coincidieron en el cine.
 2. El acervo popular es tan importante como las fuentes escritas a la Oraciones compuestas: Juan se ha ido a Italia y Marta irá a visi-
hora de realizar estudios históricos sobre la lírica tradicional.
tar a sus abuelos; El árbitro afirmó que nunca había visto tan
Frente a la casa una vasta extensión de hierba formaba un verde lleno el campo.
prado. / Para la construcción de esta autovía fue necesario reca-
var en la montaña en varias ocasiones. / Es un hombre muy basto y  2. Oraciones unidas por coordinación: [Vamos de excursión] o [nos
además tiene un carácter acerbo con nosotros porque cree que quedamos en casa]; [El pastel estaba muy bueno], pero [solo comi-
nos hemos injerido en asuntos que no nos competen. /No ingieras mos un poquito]. / Oraciones unidas por subordinación: [Le
ese medicamento sin prescripción médica. Si lees el prospecto, re- traje el vestido] [que me había pedido], [Creo] [que hemos lle-
cabas más información sobre sus efectos secundarios. / Una vez gado muy pronto]. Oraciones unidas por yuxtaposición: [Me
que la leche hierva, no podrás añadir el resto de los ingredientes. voy al médico]; [me duele la rodilla]; [He visto a Inés]; [me ha
dado recuerdos para ti]; [Nada], [corre], [salta].
  3. Ejecución, extranjería, apologético, tejer, carcaj, júnior, repug-
nancia, crujir, alergénico, fingir, marginal, geología, bagaje, mu-  3. Relación de coordinación: Elena hizo una pizza [y] Ana preparó
gir, panegírico, ligero, sonajero, ejecución, salvaje, tragicomedia. el postre;Nunca he corrido tanto [pero] lo intentaré; Queríamos
salir [pero] no encontrábamos la puerta. / Relación de subordi-
 4. Hay varias respuestas posibles. Proporcionamos algunas a nación: Juan puede llevarte [porque] tiene coche; Hoy es tu cum-
modo de ejemplo: Geo-: geografía, geometría, geólogo... / -lo- pleaños [así que] invitas tú.
gía: zoología, podología, astrología... / Gen-: genética, genero-
so, genitivo... /- aje: oleaje, pilotaje, aterrizaje... / - gésimo:
vigésimo, trigésimo, octogésima ... / -jería: extranjería, bruje- La coordinación 
ría, relojería... / -algia: lumbalgia, fibromialgia, nostalgia... /  4. Soy daltónico, es decir, confundo los colores. Coordinación ex-
-eje: despeje, tejemaneje, esqueje... / -gente: ingente, deter- plicativa. / Quedamos en mi casa y vamos todos juntos. Coordina-
gente, agente... / -jear: ojear, burbujear, chantajear. ción copulativa. / Unos recogen la mesa, otros friegan los platos.
Coordinación distributiva. / No nos perdimos, sino que no sabía-
 5. Sinónimo de aceitoso: grasiento. / Hombre que monta a caballo:
mos llegar. Coordinación adversativa exclusiva o total. / Ya dice
jinete. / Persona que lleva la gestión administrativa de una empresa
una cosa, ya dice otra. Coordinación distributiva.
o institución: gestor. / Sinónimo de complicado: complejo. / Cien-
cia que trata de Dios: teología. / Se dice del hijo que nace primero:  5. Laura no es la autora, sino la editora del libro. Enlace de tipo
primogénito. adversativo. / Está acabando la carrera, es decir, pronto empeza-
rá a trabajar. Enlace de tipo explicativo. / Me lo apunté en la
 6. Conjugamos el verbo y proporcionamos una oración a modo
agenda, pero se me olvidó. Enlace de tipo adversativo parcial o
de ejemplo: Sustraer: sustraje y sustrajisteis. Cuando sustraje
restrictivo. / Miguel llegó el primero y reservó los sitios. Enlace
el arma, mi enemigo no se inmutó. El número que sustrajisteis
de tipo copulativo. / Tanto se baña en el mar como toma el sol.
en el sorteo no corresponde a ningún participante. / Predecir:
Enlace de tipo distributivo sin función sintáctica. / Vamos en
predijo, predijimos. La vidente predijo que todo esto sucede-
coche o cogemos el autobús. Enlace de tipo disyuntivo.
ría. En el balance del año pasado, predijimos que los resulta-
dos de la empresa mejorarían. / T raducir : tradujeres y  6. Hay varias respuestas posibles:
tradujeren. Comprueba siempre todo aquello que tradujeres. Mi perro ladra mucho, pero nunca ha hecho daño a nadie. En-
Los profesores que redactaren o tradujeren alguna obra de lace de tipo adversativo parcial o restrictivo. / Ya recogen la
importancia, relativa a la enseñanza de su respectiva asigna- fruta, ya la colocan en las cajas. Enlace de tipo distributivo sin
tura, tendrán derecho a una gratificación anual. función sintáctica. / No sé si hacerme un moño o dejarme el
pelo suelto. Enlace de tipo disyuntivo. / Siempre te digo lo
 7. Copiamos el texto con los verbos conjugados en la forma verbal
mismo, mas nunca me haces caso. Enlace de tipo adversativo
adecuada: Ya desde el despertar el día comenzó mal. El dolor de
parcial o restrictivo. / Ana compró la tela y Begoña tapizó las
vientre era insoportable.Se encogía en la cama, agarrándose la tripa,
sillas. Enlace de tipo copulativo. / Había mucha comida, sin
con repugnancia, como si tuviera entre sus manos al causante de to-
embargo, no fue suficiente. Enlace de tipo adversativo parcial
dos los males del universo. Compungida, se mantuvo firme y, durante
o restrictivo.
más de media hora todavía, se abstuvo de pedir ayuda. Después, se le-
vantó y, aunque contuvo las lágrimas, dirigió una mirada firme a su  7. Conjunciones: o, y. / Locuciones conjuntivas: sin embargo, es
madre y le dijo que no podía soportarlo más. Su madre le sostuvo la decir. / Palabras correlativas: unos... otros, bien... bien.
mirada y le preguntó, fingiendo una frialdad calculada, si esta vez
estaba dispuesta a dejarse ayudar. Claudia se encogió de hombros y  8. Proposiciones coordinadas adversativas parciales: Tenía mucha
pidió a su madre que la protegiera de ella misma y que no le exigiera vergüenza, aunque lo disimulaba. / Siguió las instrucciones al pie
más allá de sus posibilidades. La madre se acercó a ella y la retuvo de la letra, pero hubo algo que no funcionó.
entre sus brazos, sin que Claudia se contrajera ni se alejara. Con- Proposiciones coordinadas adversativas totales: No estoy enfa-
tuvo la respiración, mientras pensaba asustada que, si la condujeran dada, sino que estoy cansada.
/ Nunca viaja sola, sino que siempre

28
Unidad 3 >  La oración compuesta. La coordinación

le acompaña alguien. con claridad los pensamientos. El autor, con todo, añade
que cuanto más rápido es este proceso, menos se piensa.
 9. Respuesta posible: Cristina perdió las llaves y tuvo que llamar Por lo tanto, las consecuencias de la pérdida progresiva de
al cerrajero (coordinada copulativa). / Pedro llegó a la playa, escribir a mano pueden ser nefastas. En el tercer párrafo, se
plantó su sombrilla, contempló la tranquilidad del mar (tres expone la conclusión: la enseñanza de la caligrafía alberga
proposiciones yuxtapuestas). / Luis ha ganado las elecciones, es valores que, si se prima la rapidez, se pierden.
decir, a partir de hoy es el nuevo entrenador del equipo (coor-
dinada explicativa). / Estudia o bien se pone a trabajar (coordi-  2. Es un texto argumentativo, de opinión. En el texto aparece
nada disyuntiva). una serie de elementos característicos de este género perio-
dístico; por ejemplo, la modalización (la ablación de la cursi-
va), las preguntas retóricas (¿dejarán de explicar trigonometría
Análisis de la oración (pág. 69-79) o la diferencia entre arrianismo y monofisismo, pues?), ofrece
ejemplos o argumentos de autoridad (del Instituto Nacional de
 1. En este breve fragmento predomina la yuxtaposición, puesto Educación y de Minna Harmanen).
que las diferentes proposiciones están unidas mediante comas.
 3. Repudia: rechaza; rezuman: transmiten; réplicas: contestacio-
 2. Respuesta posible: Le he dejado el coche, aunque no sé si lo nes; vetusto: antiguo.
admitirá. / Coge un taxi, o no llegarás a la reunión. / No podemos
comprar este piso porque es muy caro.  4. En el contexto del fragmento, la expresión significa «totalmen-
te», «por completo»; es decir, pretende intensificar el hecho de
 3. [3ª pers. pl.-S] Han anulado (V) la (Det) (N-SN) al (Prep +
visita que la pedagogía haya basado su decisión en la funcionalidad.
Det) museo (N-SN) y (Conj-e) nos (Pron.) recogerán (V) un (Det)
poco (A) más (A) tarde (N-SN). Oración compuesta por coordina-  5. Un criminalista diría que de nuevo se confunden las pruebas cir-

ción copulativa (Conj. «y»): CD: «la visita»; CCL: «al museo»; cunstanciales con las incriminatorias. La relación es de subordi-
CCT: «un poco más tarde» / [2ª pers. sing.-S] Coge (V) el (Det) nación. / Escribir a mano es un modo más lento de traslación de

autobús (N-SN) (CD), baja (V) en (Prep) la (Det) tercera (Det) pensamiento que teclear y [...] permite una reflexión mayor. La

parada (N), anda (V) dos (Det) manzanas (N), cruza (V) la (Det) relación es de coordinación copulativa; dentro de la misma
calle (N). Oración compuesta por yuxtaposición./ Unos (Pron-S) hay una de subordinación.
trabajan (V) de (Prep) día (N), otros (Pron-S) lo (Pron-CD) hacen
(V) de (Prep) noche (N). Oración compuesta por coordinación
 6. Es más: enlace coordinante explicativo.
distributiva. / A (Prep) Raquel (N) le (Pron-CI) encantan (V) los O bien: enlace coordinante disyuntivo.
(Det) gatos (N) (S), pero (Conj-e) le (Pron-CI) dan (V) alergia (N). Sino que: enlace coordinante adversativo total.
Oración compuesta por coordinación adversativa parcial. / ¿ [2ª
pers. sg.] Me (Pron-CI) dices (V) qué (Pron. interrog.-CD) te Ya..., ya....: enlace coordinante distributivo.
(Pron-CI) pasa (V) o (Conj-e) [1ª pers. sg.] empezaré (V) a (Prep)
 7. Los niños finlandeses dejarán de aprender la letra cursiva, es de-
preocuparme (V)? Oración compuesta por coordinación disyunti-
cir, recibirán instrucción para que escriban en letra de imprenta.
va. «Empezar a preocuparse» es una perífrasis verbal y se
Oración coordinada explicativa. / Escribir con letras de imprenta
analiza como una única unidad verbal. / [1ª pers. sg.] Mañana
es más rápido y la cursiva solo se usa en el colegio, si bien es difícil
(Av) tengo (V) un (Det) partido (N), es decir (Loc. conj.-e), tengo
de aprender.Oración compuesta por coordinación copulativa y
que descansar (V) bien (Av). Oración compuesta por coordina-
adversativa parcial.
ción explicativa. «tengo que descansar» es una perífrasis verbal
y se analiza como una única unidad verbal. / [1ª pers. pl.] No Escribir (V) con (Prep) letras (N) de (Prep) imprenta (N) es (V)
(av) haremos (V) el (Det) examen (N), sino que (Conj.-e) [1ª pers. más (Av) rápido (A-Sa-Atr) y (Conj-E) la (Det) cursiva (N) solo

pl.] entregaremos (V) un (Det) trabajo (N). Oración compuesta (Av) se usa (V) en (Prep) el (Det) colegio (N), si bien (Loc. conj-
por coordinación adversativa total. / [1ª pers. pl.] Comeremos (V) E) [3ª pers. sg.] es (V) difícil (A-Atr) de (Prep) aprender (V).
en (Prep) el (Det) restaurante (N) o (Conj-e) [1ª pers. sg.] prepa- Oración compuesta por coordinación copulativa y coordinación
ro (V) un (Det) pícnic (N). Oración compuesta por coordinación adversativa parcial.
disyuntiva.
 8. Mecanografía: mecano + grafía: está formado por los términos
griegos mecano- (máquina) y -grafía (escritura). / Anticaligrá-
Evaluación (pág. 71) ficos: anti + caligráficos: está formado por el prefijo anti- (con-
tra) y la palabra griega caligrafía (palabra compuesta), además
  1. Esta lectura aborda el debate actual sobre qué tipo de cali- de morfemas flexivos de género y número. / Funcionalidad:
grafía debe enseñarse a los estudiantes —letra cursiva o está formado por el sustantivo función y el morfema -dad. /
letra de imprenta—, a partir de la decisión que se ha toma- Uniformidad: está formado por adjetivo uniforme y el sufijo de
do en Finlandia, referente mundial por su potente sistema formación de sustantivos -dad.
educativo, de apostar por la letra de imprenta y la mecano-
grafía, en sustitución de la letra cursiva. En el primer párra-  9. Respuesta libre.
fo, se plantea el tema a partir de la decisión tomada en ese
país de suprimir la letra cursiva y sustituirla por la mecano-
grafía, y se apoya en los datos del Instituto Nacional de
Educación. No obstante, el párrafo termina con dos pregun-
tas que cuestionan esta decisión. En el segundo párrafo, se
introduce un nuevo argumento a partir de las declaraciones
de Minna Harmanen: las personas escriben para expresar

29
4# La subordinación sustantiva

Dios nos dio la memoria para que pudiéramos tener rosas en invier-
En contexto (pág. 73)
no: Revivir situaciones anteriores o soñar.

a) [La RAE está] al mando del idioma que hablan unos


559 millones
de personas en el mundo: Se trata de una proposición subordina-  7. Queridos amigos, les agradezco esta flor inmarchitable que
da adjetiva que realiza la función de CN del sustantivo idioma, me han otorgado hoy y que me acompañará en el recuerdo en
que aparece en la proposición principal. Decimos que es una los días más fríos: Fernando Savater compara la distinción de
proposición subordinada porque mantiene una relación de la Universidad con una «flor inmarchitable» que siempre le
dependencia respecto a la proposición principal (La RAE está acompañará y recordará con enorme gratitud en los momen-
al mando del idioma) y está introducida por un nexo subordi- tos difíciles de la vida.
nante, en este caso el pronombre relativo que, el cual realiza
una doble función: la de nexo que introduce la proposición
subordinada, y dentro de la misma proposición realiza, ade- Cuestión de lengua
más, la función de CD.
 8. Predomina la primera persona del singular en los verbos
b) Respuesta abierta. (Tengo que iniciar, he aceptado, prefiero, he intentado, en-
tiendo...) y en los pronombres (me alegra, se me confiere, me
han otorgado...).
Practica con los textos (págs. 75-76)
Justifica el motivo...: El autor habla a partir de su propia expe-
riencia.
Comprensión
 9. Tengo que iniciar estas palabras agradeciendo muy sinceramente

 1. ¿A quién va dirigido el discurso? Al rector de la Universidad Si- este honor: con mucha sinceridad. / Me alegra particularmente
món Bolívar, y a los amigos, amigas y colegas del autor. que este error generoso se me conceda aquí en América: de mane-
ra especial. / Estoy viniendo asiduamente a mantener coloquios:
¿Por qué motivo lo pronuncia? Se trata de un discurso que pro- con frecuencia. / No se puede crear personas que van a vivir,
nuncia Fernando Savater cuando le nombran Doctor Honoris constantemente, dependientes de autoridades...: siempre. / Está
Causa de la Universidad Simón Bolívar. totalmente negado para lo que la vida contemporánea va a exigir:

¿En qué ciudad se celebra el acontecimiento? En Caracas (Vene- por completo. / Prefiero crear personas ingenuamente convenci-
das de que contra todos los males algo se puede hacer: sencillas
zuela).
(en su convencimiento). / No podemos esperar a que venga el si-
glo que viene a ver qué movimientos y corrientes cósmicas nos libe-
 2. La educación como transmisora de valores y como llave para
ran de nuestros males o nos condenan a ellos definitivamente:
escapar del destino.
para siempre, de forma definitiva. / Entiendo que este honor
Localiza en el texto el fragmento...: El tema se enuncia en el que, inmerecida pero gratísimamente, se me confiere hoy...: sin
quinto párrafo: La educación es la única forma que hay de li- merecerlo, con mucho agrado.
berar a los hombres del destino...
10. No comas esto (adjetivo demostrativo), vete por aquí (adverbio
 3. Son, según el autor, valores fundamentales de la ciudadanía. de lugar) no te (pronombre personal) arriesgues. / Formar (infi-
nitivo, valor verbal) personas capaces de cooperar con los demás
 4. Para adaptarse a los cambios continuos de trabajo. (adjetivo indefinido). / Educar (infinitivo, valor nominal) es edu-
car contra (preposición) el destino.
 5. Hay que formar la capacidad de autonomía pues vivimos en
un mundo muy complejo y las personas no pueden ni deben 11. El núcleo del predicado es una perífrasis en estas oraciones:
ser dependientes, sino que deben responsabilizarse para bien Tengo que iniciar (perífrasis modal de obligación) estas palabras
o para mal de lo que hacen. En segundo lugar, hay que formar agradeciendo muy sinceramente este honor. / Hay que formar (pe-
personas capaces de cooperar y colaborar con los demás rífrasis modal de obligación) la capacidad de autonomía. / Los
para poder adaptarse, por ejemplo, a los continuos cambios trabajos van a ser (perífrasis aspectual ingresiva) cada vez más
de trabajo. Por último, hay que crear personas optimistas y aleatorios. / Estoy viniendo (perífrasis aspectual durativa) asi-
esperanzadas en que los cambios son posibles y en que siem- duamente a mantener coloquios.
pre se puede mejorar una situación.
Localiza otras perífrasis verbales en el texto: Si algún mínimo
mérito puedo tener (perífrasis modal de posibilidad)... / Los
 6. Me alegra particularmente que este error generoso se me conceda
van a acompañar (perífrasis aspectual ingresiva) toda la
aquí en América: El autor para mostrar su modestia habla de
vida... / Tiene que ser (perífrasis modal de obligación) trans-
«error» al hecho de que le concedan la distinción de ser Doc-
mitido en el momento en que aún se puede transmitir (perí-
tor Honoris Causa de la Universidad Simón Bolívar, y añade el
frasis modal de posibilidad). / Las personas van a tener que
adjetivo «generoso» para indicar su gratitud.
trabajar (perífrasis aspectual ingresiva + perífrasis modal de
Educar es una labor ética emancipadora: La ética permite que las obligación) en siete u ocho trabajos a lo largo de su vida; en
personas sean libres e independientes (emanciparse). todos ellos van a necesitar (perífrasis aspectual ingresiva) la

30
Unidad 4 >  La subordinación sustantiva

capacidad de saber cooperar con los demás... / Hay que 16. Los trabajos van a ser cada vez más inciertos. / En todos ellos
colaborar (perífrasis modal de obligación), [hay que] coope- van a necesitar la capacidad de saber colaborar con los de-
rar (perífrasis modal de obligación), [hay que] dividir (perí- más. / Presentarnos como candidatos, como una mejor op-
frasis modal de obligación) el trabajo con los otros... / Lo que ción frente a ellos. / La educación es la única forma que hay
se opone a que el hijo del pobre tenga que ser (perífrasis de liberar a los hombres de la fatalidad. / Esto de alguna ma-
modal de obligación) siempre pobre; a que el hijo del igno- nera afecta a los educadores. / Con mucho gusto he aceptado
rante tenga que ser (perífrasis modal de obligación) siempre esa tarea de recordar ciertas cosas elementales. / Estoy vi-
ignorante... / Hay que educar (perífrasis modal de obliga- niendo periódicamente a mantener coloquios, a aprender, a
ción) para la ética, hay que saber (perífrasis modal de obli- mantener intercambios.
gación) que educar es ya, en sí, una labor ética,
emancipadora. / No hay que educar (perífrasis modal de 17. Prefiero crear personas ingenuamente convencidas de que
obligación) para la desesperanza. / Contra todos los males contra todos los males algo se puede hacer, porque estos
algo se puede hacer (Perífrasis modal de posibilidad). / No nunca se resolverán solos; no sé si nosotros los vamos a
sé si nosotros los vamos a resolver (perífrasis modal ingresi- solventar, sé que si no los gestionamos nosotros, no se solu-
va)... / Esto es lo que me parece que hay que transmitir cionarán.
(perífrasis modal de obligación) con unas pautas, no digo de
optimismo desenfrenado loco, pero al menos de un cierto
pesimismo que acepte que hay que actuar (perífrasis modal Análisis del texto
de obligación); que algo hay que hacer (perífrasis modal de
obligación). / No se puede esperar (perífrasis modal de posi- 18. Respuesta abierta. Proponemos la siguiente: Es importante
bilidad) a una ocasión mejor... / Dios nos dio la memoria preparar y organizar un discurso de agradecimiento teniendo
para que pudiéramos tener (perífrasis modal de posibilidad) en cuenta que debe presentar estas partes: introducción o
rosas en invierno. presentación, cuerpo (desarrollo del contenido o razones del
agradecimiento) y conclusión o despedida. Puede resultar in-
12. Proposiciones subordinadas sustantivas: Me alegra particular- teresante añadir al discurso historias y anécdotas personales.
mente que este error generoso se me conceda aquí en Amé- El texto debe ser coherente y estar bien cohesionado y, ade-
rica (Sujeto). / Si se educa diciendo que el mundo es un más, el tono debe ser alegre y elevado.
desastre (CD) que todos los políticos son corruptos (CD)... / Comprueba si estos rasgos...: Los rasgos que se han menciona-
No sé si nosotros los vamos a resolver (CD), sé que si no los do en la respuesta anterior corresponden al discurso de Fer-
resolvemos nosotros, no se resolverán (CD) / Dijo que Dios nando Savater.
nos dio la memoria (CD) para que pudiéramos tener rosas
en invierno. 19. Introducción o presentación: El autor se presenta y explica el
Proposiciones subordinadas adjetivas: Tengo que iniciar estas motivo del discurso mostrando su agradecimiento.
palabras agradeciendo muy sinceramente este honor que se Cuerpo: Savater desarrolla su discurso exponiendo la informa-
me confiere (CN) y que verdaderamente considero un error ción de forma cohesionada.
muy generoso (CN). / En una época que no era corriente
(CN) que las personas de mi generación... (S) / No se puede Conclusión o despedida: El autor vuelve a mostrar su gratitud
crear personas que van a vivir, constantemente, dependien- a los receptores del mensaje, y termina el discurso con una
tes de autoridades (CN). / Quien es incapaz (Sujeto) porque metáfora mediante la que vuelve a mostrar su agradecimiento
no entiende lo que le dicen (CD)... / Culparán de todos los a la Universidad por concederle esa distinción.
males a la situación cósmica que les ha tocado soportar
(CN). / Les agradezco esta flor inmarchitable que me han 20. Se trata de un texto coherente. Indica algunos procedimientos

otorgado hoy (CN) y que me acompañará en el recuerdo en empleados en el texto...:


Las ideas del texto se relacionan entre
los días más fríos (CN). sí y guardan un orden lógico. / La información avanza de for-
ma lógica y progresiva. / La información se estructura en pá-
rrafos cada uno de los cuales desarrolla una idea que, a su
Cuestión de léxico vez, está conectada con el núcleo temático global.
El texto está bien cohesionado. Indica algunos procedimientos
13. Excelentísimo, muy generoso. empleados en el texto...: Se emplean recursos léxicos como la
A continuación, forma los superlativos...: Incierto: inciertísimo. / reiteración léxica (verdaderamente considero un error muy
Noble: nobilísimo. / Fuerte: fortísimo, fuertísimo. / Antiguo: generoso, que este error generoso se me conceda...; los tra-
antiquísimo. / Sabio: sapientísimo. / Nuevo: novísimo. / Fiel: fi- bajos van a ser cada vez más aleatorios... las personas van a
delísimo. / Valiente: valentísimo; valientísimo. / Pobre: paupé- tener que trabajar en siete u ocho trabajos a lo largo de su
rrimo. vida), la sustitución sinonímica (hay que colaborar, cooperar,
dividir el trabajo con los otros...), el campo semántico común
14. Auto-: autobiografía, autorretrato, autocontrol, autodestruc- (Junto con la autonomía, la capacidad de cooperación es im-
ción, autoabastecerse, autoafirmación, autoayuda... prescindible...), el uso de hiperónimos e hipónimos (pudiéra-
mos tener rosas en invierno, esta flor inmarchitable...),
-nomía: astronomía, economía, agronomía, biblioteconomía, expresiones figuradas como la metáfora que cierra el discur-
ergonomía, gastronomía, fisionomía... so. / Se emplean, también, recursos gramaticales como la
pronominalización (Los trabajos van a ser cada vez más
15. Imprescindible: negación. / Antifatalidad: oposición. / Desespe- aleatorios [...] en todos ellos van a necesitar la capacidad de
ranza: negación, privación. / Neoliberalismo: nuevo o reciente. saber cooperar con los demás), y conectores que permiten
/ Inmerecida: negación. enlazar palabras o expresiones del texto que muestran rela-

31
Unidad 4 >  La subordinación sustantiva

ciones lógicas que existen entre ellas (En primer lugar, hay ca los motivos que le han llevado a visitar la América Hispana
que formar la capacidad de autonomía...; En segundo lugar, en varias ocasiones, sintiéndose siempre muy bien recibido.
formar personas capaces...; Además, hay que educar para la
Explica que la capacidad de autonomía y la capacidad de
ética...).
cooperación deben ser los valores fundamentales de la ciuda-
danía, y que la educación es el arma principal de la que dis-
21. Como decía Borges de su éxito: Escritor argentino considera-
ponemos para crecer y avanzar. Hay que intentar ser positivo
do una de las grandes figuras de la literatura en lengua espa-
y creer que es posible mejorar.
ñola del siglo xx. Cultivó varios géneros, pero ocupa un lugar
destacado en la historia de la literatura sobre todo por sus Se despide dando las gracias y asegurando que guardará en
relatos breves. Algunas de sus obras más conocidas son Fic- su memoria la grata distinción que ha tenido hacia él la Uni-
ciones o El Aleph. versidad Simón Bolívar de Caracas.
Recuerdo unos versos de un gran poeta inglés: El autor hace
25. Actividad de expresión oral.
referencia a John Keats, poeta romántico inglés cuya lírica se
caracteriza por un lenguaje muy imaginativo. Compuso poemas
memorables que lo han convertido en una de las grandes figu-
ras de la literatura inglesa y mundial. Algunas de sus obras más
La norma ortográfica (pág. 77)
conocidas son Oda a un ruiseñor, La estrella brillante o Hype-
rion.
 1. Cazadores extrahumanos
están cruzando luceros,
¿Cuál crees que es la intención del autor...? Se trata de citas o
argumentos de autoridad que consisten en dar testimonio de cisnes de plata maciza
personas relevantes para justificar una idea. en el agua del silencio.

22. Dirigirse al público:Excelentísimo y magnífico señor rector; Los chopos niños recitan
queridos colegas; amigas y amigos... / No sé si nosotros los su cartilla; es el maestro
vamos a resolver... / Aunque en este país afortunado en el
un chopo antiguo que mueve
que ustedes viven, las estaciones no tienen el peso simbólico
y dramático que tienen... / Queridos amigos, les agradezco... tranquilo sus brazos muertos.

Mantener la atención: ¿Qué es lo que queremos formar como  2. Estos niños han sido precoces en los desarrollos psicomotri-
valores fundamentales de la ciudadanía? ces. / Los albornoces de los capataces tienen ciertas amarille-
Referirse a aspectos personales: su vida, su profesión, sus senti- ces. / Los portavoces de los jueces carecen de las brillanteces
mientos...: Si algún mínimo mérito puedo tener es que desde propias de sus cargos cuando comparecen ante la prensa. /
hace más de un cuarto de siglo, en una época que no era Cuando los alféreces dejan sus puestos se muestran locuaces
corriente que las personas de mi generación gustasen de y felices. / Los arroces andaluces proporcionan acideces a las
venir a los países de la América Latina, estoy viniendo asi- elaboraciones de las perdices rojas.
duamente... / Con mucho gusto he aceptado esa tarea de Señala, en cada caso, qué cambios...: Las palabras que se escri-
recordar ciertas cosas básicas... / Entiendo que este honor ben con «-z» final forman el plural en «-ces».
que se me confiere hoy, responde a esa actividad que he
llevado a cabo durante muchos años... / Esa flor inmarchita-  3. Convencer, azerí, zueco, cero, zarzo, vejez, cima, elzevir, zis-
ble que me han otorgado hoy y que me acompañará en el zás, zafio, parteluz, zepelín, diezmo, eczema, castizo.
recuerdo en los días más fríos...
 4. Presente de indicativo Presente de subjuntivo
23. Rasgos prosódicos: Los enunciados que emplea el orador son
breves y concisos y con ellos transmite mensajes relevantes. / Complacer: complazco, com- Adolecer: adolezca, adolezcas,
Las pausas equivalen a los signos de puntuación en el lengua- places, complace, complace- adolezca, adolezcamos, ado-
je escrito y permiten al autor regular su discurso. mos, complacéis, complacen lezcáis, adolezcan
Adolecer: adolezco, adoleces, R econocer : reconozca, reco-
Rasgos extranlingüísticos: Las gestos y la postura que muestra
adolece, adolecemos, adole- nozcas, reconozca, reconozca-
el emisor del mensaje, Fernando Savater, acompañan su dis- céis, adolecen mos, recono cáis, reconozcan
curso favoreciendo el intercambio comunicativo con el audito-
rio, mostrando en todo momento su gratitud por la distinción Prevalecer: prevalezco, preva- Cocer: cueza, cuezas, cueza,
leces, prevalece, prevalece- cozamos, cozáis, cuezan
que se la ha concedido. La mirada, los rasgos faciales, los
mos, prevalecéis, prevalecen
movimientos relajados del cuerpo concuerdan en todo mo-
mento con el mensaje que pretende transmitir.  5. Quorum: Número de individuos necesario para que un cuerpo
Rasgos deícticos: Emplea pronombres y otros recursos grama- deliberante tome ciertos acuerdos. / Quadrivium: En la Edad Me-
ticales, que permiten construir un texto bien cohesionado, y dia, conjunto de las cuatro artes matemáticas, aritmética, músi-
además permiten que los receptores del texto (el auditorio) ca, geometría y astrología o astronomía, que, junto con el trivio,
sientan que están participando del acto comunicativo. constituía los estudios que impartían las universidades. / Lugar
donde concurren cuatro caminos. / Statu quo: estado de cosas
en un determinado momento. / Quid pro quo: Cosa que sustituye
Expresión a algo equivalente o que se recibe como compensación por ello.
/ Quid: Esencia, punto más importante o razón de una cosa. /
24. Tras un mensaje de agradecimiento dirigido a las autoridades Quater: Significa «cuatro veces» y, añadido a cualquier número
presentes y a sus amigos y a amigas, Fernando Savater expli- entero, indica que este se ha repetido por cuarta vez.

32
Unidad 4 >  La subordinación sustantiva

 6. bernantes creen que (nexo sin función sintáctica) el país se recu-
perará de la crisis económica. / El que (pronombre relativo,
Presente de indicativo Presente de subjuntivo Sujeto ) tenga mejor nota podrá optar a la carrera deseada. /
Verificar: verifico, verificas, ve- Verificar: verifique, verifiques, Quien (pronombre relativo, Sujeto) bien te quiere, te hará llo-
rifica, verificamos, verificáis, verifique, verifiquemos, verifi- rar. / El profesor preguntó si (nexo sin función sintáctica) sa-
verifican quéis, verifiquen bían la solución del problema.
Convocar: convoco, convocas, C onvocar : convoque, convo-
De las oraciones anteriores, localiza...: Relativas libres: Quien
convoca, convocamos, convo- ques, convoque, convoque-
saque mayor puntuación, saldrá primero en la carrera. /
cáis, convocan mos, convoquéis, convoquen
Quien bien te quiere, te hará llorar. / Relativa semilibre: El que
P ublicar : publico, publicas, Publicar: publique, publiques, tenga mejor nota podrá optar a la carrera deseada.
publica, publicamos, publi- publique, publiquemos, publi-
cáis, publican quéis, publiquen
Becar: beco, becas, beca, be- Becar: beque, beques, beque, La subordinación sustantiva (págs. 80-85)
camos, becáis, becan bequemos, bequéis, bequen
 4. Respuesta abierta. Ofrecemos las siguientes: Me encantaría
Provocar: provoco, provocas, P rovocar : provoque, provo-
que me dijeras la verdad:Me encantaría tu sinceridad. / El pro-
provoca, provocamos, provo- ques, provoque, provoque-
blema es que Pedro ya se ha enterado: El problema es el conoci-
cáis, provocan mos, provoquéis, provoquen
miento. / Gustavo tenía muchas ganas de que vinierais: Gustavo
A continuación, escribe una oración con una forma verbal...: Res- tenía muchas ganas de vuestra visita. / La prensa ha anunciado
puesta abierta. que habrá recortes económicos el próximo año: La prensa ha
anunciado recortes económicos. / Que a Alberto no le guste el
 7. Inyectar, arrullo, perogrullo, epopeya, sellar, adyuvante, queso no es tu problema: Su manía no es tu problema. / Luis

disyunción, yunque, adyacente, subyugar, proyección, mue- estaba satisfecho de que hubiera asistido tanta gente a las jorna-

lle, proveyeras, orilla, destello, cayuco. das gastronómicas: Luis estaba satisfecho de la asistencia de
público a las jornadas gastronómicas. / El jefe me dijo que mi
 8. Saharaui, ley, Eloy, curry, ionosfera, bonsái, hoy, reyes, biqui- traslado era inminente: El jefe me dijo lo inminente.

ni, virrey, convoy, Uruguay.


 5. La policía no encontró al sospechoso que (pronombre relativo)
 9. Apabullar, adyuvante, leyendo, subyacer, disyuntiva, maullar, les indicó el testigo.
/ Los arqueólogos no pensaban que (conjun-
altillo. ción) el templo fuera tan antiguo. / Se preguntó si (conjunción)
lo haría. / Compraré las cosas que (pronombre relativo) me han

La palabra que aparece en la columna central es PALATAL. encargado. / ¡No me gusta que (conjunción) se cuestionen mis de-
cisiones!

ACTIVIDADES (págs. 88-89)  6. No es posible que hayas llegado tan lejos (Sujeto). / Me pregunto
si es pertinente tu respuesta (CD). / La posibilidad de que lleguen
a la cima es muy remota (CN). / No le convencía el hecho de que no
La subordinación (págs. 78-79) vinieran a la cena (CN). / Estoy cansada de que no me toméis en
serio (CA). / Confío en que la marea amaine por la tarde (CR).
 1. Fuimos al parque, paseamos por los jardines, regresamos por la
tarde: Yuxtaposición. / Juzgaron conveniente que nos marchára-
mos: Subordinación sustantiva. / No saben jugar, pero pueden
 7. Prefiero pensar que fue una idea repentina: CD. / Me parece que
la culpa cae aún más sobre mis hombros: Sujeto. / Quiero pensar
aprender: Coordinación adversativa. / Creo que regresarán a
mediodía: Subordinación sustantiva. / Nos quedaremos en la bi-
que Milagros (CD) que no habría forma de encontrar una
sintió
felicidad más intensa (CD). / Prefiero pensar que, de pronto, esa
blioteca y avanzaremos el trabajo de Lengua: Coordinación copu-
mujer de ideas caprichosas tuvo una revelación: CD. / La esperanza
lativa.
de que podía encontrarse con su madre y con su hijo en la vida
eterna: CN.
 2. Me gusta que seas una persona optimista: Sustantiva (Me gusta tu
optimismo). / Fuimos al local donde vendían los helados de moca
con menta: Adjetiva (Fuimos al local más cercano). / Los caba-  8. Respuesta abierta. Proponemos las siguientes: Me gusta que

llos se detuvieron donde terminaba el camino de tierra: Adverbial


seas una persona educada (S). / Los periodistas informaron de
(Los caballos se detuvieron allí). / Los pantalones azules que te que los aeropuertos permanecían cerrados (CR). / Lo peor es
gustan todavía no están rebajados: Adjetiva (Los pantalones azu-
que nadie me comprende (S). / Todos creen que Javier es
les vaqueros todavía no están rebajados). / El equipo local ale- inocente (CD). / Nos preocupan las causas de que la economía
gó cuando ya era demasiado tarde: Adverbial (El equipo local
empeore (CN). / No era consciente de que la lesión fuera tan
alegó demasiado tarde). / Me parece muy buena la idea que me grave (CA).
comentaste ayer: Adjetiva (Me parece muy buena la idea origi-
nal). / Me parece buena idea que te matricules en la escuela de  9. Respuesta abierta. Proponemos las siguientes: Acordarse: Se
música: Sustantiva (Me parece buena idea tu decisión).
acordó de que tenía una reunión. / Enorgullecerse: Mi amigo
se enorgullecía de haberme conocido. / Confiar: El candidato
 3. El gobierno no sabe si (nexo sin función sintáctica) autorizará confiaba en que aprobaría las pruebas de selección. / Ente-
rarse: Nos enteramos de que inauguraban un nuevo centro
las reformas. / Volvimos a la ciudad donde (adverbio relativo,
CCL) nos conocimos. / Nos vimos donde (nexo con función sin- para disminuidos psíquicos. / Insistir: El presentador insistió
táctica, CCL) nos dijiste. / Quien (pronombre relativo, Sujeto) en que la noticia procedía de fuentes directas.
saque mayor puntuación, saldrá primero en la carrera. / Los go-

33
Unidad 4 >  La subordinación sustantiva

10. Sonia me pidió que la ayudara a preparar la cena de aniversa- principal (sabemos). c) Proposición subordinada sustantiva:
rio de la boda de su hermano. / El guía sugirió que podíamos que te quedes en el hospital; funciona como sujeto de la
dejar todas las maletas en la recepción del hotel. / Andrés proposición b.
añadió que nadie debía salir de esta habitación sin su permi-
so. / El escritor aseguró que su última novela sería un éxito Comentó cuándo volvería de Londres. a) Proposición principal:
aún mayor que la anterior. Comentó; b) Proposición subordinada sustantiva interrogativa
indirecta parcial que realiza la función de CD: cuándo volvería
11. Mi compañero me preguntó si yo había solucionado el problema:
de Londres; está introducida por un adverbio interrogativo
total. / Pablo no sabía si la farmacia estaría abierta: total. / Averi- (cuándo), que desempeña la función de CCT dentro de la
gua cuándo sale el próximo tren: parcial. / Javier me preguntó
proposición subordinada.
dónde estaba la biblioteca: parcial. / Emilio ignoraba si había dine-
ro en la caja fuerte: total.
 2. No (Modificador verbal) supimos (V) qué (Pron interrogativo)
había pasado (V) hasta (Prep) la (Det) medianoche (N). a) Propo-
sición principal: No supimos; b) Proposición subordinada
sustantiva interrogativa indirecta parcial: qué había pasado
El infinitivo (págs. 86-87)
hasta la medianoche. Está introducida por un pronombre inte-
12. No podremos ir (perífrasis verbal) esta tarde al cine. / Su afi- rrogativo (qué), que desempeña la función de sujeto dentro
ción favorita es jugar al ajedrez. / Intentaré llegar a tiempo. / de la proposición subordinada.
Tuvo que abandonar (perífrasis verbal) la reunión a las diez. /
Confío (V) en (Prep) que (Conj) mi (Det) encargo (N) esté (V)
Quería actuar junto a su actor favorito.
preparado (A) esta (Det) tarde (N): a) Proposición principal:
Confío (en); b) Proposición subordinada sustantiva: que mi
13. Respuesta abierta. Proponemos las siguientes: Me gusta mu-
encargo esté preparado esta tarde. Desempeña la función de
cho leer poemas en mi habitación. / Siempre tratamos de con- CR del verbo de la proposición principal.
testar amablemente. / Le convenceré para ir en Navidad. / Es
imprescindible aprobar todas las asignaturas. / Te prometo vol- Quería (V) conseguir (V) unas (Det) fotos (N) de (Prep) animales
ver animado. / Siempre quieres confiar en los demás. (N) salvajes (A): Oración simple. El infinitivo conseguir realiza
la función de CD de quería y, a su vez, va acompañado de un
A continuación, especifica qué función...: Me gusta mucho leer CD (unas fotos de animales salvajes).
poemas en mi habitación: Sujeto. / Siempre tratamos de con-
testar amablemente: CR. / Le convenceré para ir en Navidad:
 3. Juan (N) desconoce (V) si (Conj) los (Det) trenes (N) salen (V) a
CCFin. / Es imprescindible aprobar todas las asignaturas: Su-
(Prep) la (Det) hora (N). Oración compuesta por subordina-
jeto. / Te prometo volver animado: CD / Siempre quieres con-
ción. a) Proposición principal: Juan desconoce; b) Proposi-
fiar en los demás: CD.
ción subordinada sustantiva interrogativa total: si los trenes
salen a la hora. Está introducida por el nexo subordinante si y
14. Insistir (S) en lo mismo (CR) es su característica esencial. / Parece desempeña la función de CD del verbo de la proposición prin-
cansado de esperar (CA) tanto tiempo (CCT). / Fingió no (Modifi- cipal (desconoce).
cador verbal) ver (CD) a nadie (CD) para no (Modificador ver-
bal) saludarlos (saludar (CC) + los (CD)). / Están contentos de Mónica (N) distribuyó (V) los (Det) horarios (N) y (Conj) yo
recibir (CA) noticias nuestras (CD). / Aseguraba decir (CD) siem- (Pron) me (Pron) encargué (V) de (Prep) coordinar (V) los (Det)
pre (CCT) la verdad (CD) a sus empleados (CI). / Necesitaban salir equipos (N): Oración formada por dos proposiciones coordina-
(CD) unos días (CCT) de la ciudad (CCL). das copulativas unidas por el enlace y. Las proposiciones son:
a) Mónica distribuyó los horarios y b) yo me encargué de
coordinar los equipos. Dentro de esta última encontramos una
Análisis de la oración (pág. 91) proposición subordinada sustantiva de infinitivo (coordinar los
equipos) que funciona como CR del verbo (encargué).
 1. Explicaron cómo habían logrado convencer a tanta gente. a) Pro- Me (Prep) pregunto (V) si (Conj) puedo traer (Perífrasis verbal)
posición principal: Explicaron; b) Proposición subordinada mi (Det) propio (A) ordenador (N): Oración compuesta por sub-
sustantiva interrogativa indirecta parcial: cómo habían logrado ordinación. a) Proposición principal: Me pregunto; b) Proposi-
convencer a tanta gente. Está introducida por un adverbio in- ción subordinada sustantiva interrogativa total: si puedo traer
terrogativo (cómo) que, además de nexo, desempeña la fun- mi propio ordenador. Está introducida por el nexo subordinan-
ción de CCM dentro de la proposición subordinada. El te si y desempeña la función de CD del verbo de la proposi-
infinitivo convencer funciona como CD de habían logrado y, a ción principal (pregunto).
su vez, está acompañado de un CD (a tanta gente).
Nos (Pron) interesa (V) discutir (V) la (Det) posibilidad (N) de
Laura me preguntó si quería una entrada. a) Proposición princi- (Prep) que (Conj) haya (V) vida (N) en (Prep) Marte (N): Oración
pal: Laura me preguntó; b) Proposición subordinada sustanti- compuesta por subordinación. a) Proposición principal: Nos
va interrogativa total: si quería una entrada. Está introducida interesa; b) Proposición subordinada sustantiva: discutir la
por el nexo subordinante si, y desempeña la función de CD posibilidad de que haya vida en Marte. El infinitivo discutir
del verbo de la proposición principal (preguntó). actúa como sustantivo porque funciona como sujeto de inte-
resa y, también, como verbo, ya que va acompañado de un
No sabemos si es preferible que te quedes en el hospital. a) Pro-
CD (la posibilidad de que haya vida en Marte).
posición principal: No sabemos; b) Proposición subordinada
sustantiva interrogativa total: si es preferible que te quedes Necesitaba (V) salir (V) unos (Det) días (N) de (Prep) la (Det)
en el hospital. Está introducida por el nexo subordinante si, y ciudad (N): Oración simple. El infinitivo salir realiza la función
desempeña la función de CD del verbo de la proposición de CD del verbo principal (Necesitaba) y, a su vez, va acom-

34
Unidad 4 >  La subordinación sustantiva

Unos (Pron) entraron (V) en (Prep) el (Det) cine (N), otros  3. El espectador de cine teoriza sin parar, lo sepa o no, acerca de lo
(Pron) se quedaron (V) en (Prep) la (Det) cafetería (N): Oración que está viendo:
De forma inconsciente, el espectador reflexio-
compuesta por dos proposiciones coordinadas distributivas. na constantemente sobre lo está viendo en la pantalla de cine.
Las proposiciones son: a) Unos entraron en el cine y b) Otros
se quedaron en la cafetería. [El usuario de videojuegos] anticipa qué sucederá y reacciona in-
tentando apresurarlo o evitarlo:A medida que el usuario va
Tengo (V) la (Det) evidencia (N) de (Prep) que (Conj) los (Det) conociendo el juego, puede prever qué ocurrirá para así po-
hechos (N) sucedieron (V) de (Prep) esa (Det) manera (N): Ora- der provocar situaciones concretas o evitar que ocurran.
ción compuesta por subordinación. a) Proposición principal:
Los videojuegos no son una novedad absoluta, sino materialización
Tengo la evidencia; b) Proposición subordinada sustantiva: de
en una nueva forma narrativa de algo que ya ha existido: Los vi-
que los hechos sucedieron de esa manera. La proposición
deojuegos pueden compararse a los cuentos infantiles en
subordinada funciona como CN del sustantivo evidencia de la
proposición principal. cuanto al formato de una historia que se va narrando.

 4. [El usuario de videojuegos] anticipa qué sucederá y reacciona


intentando apresurarlo o evitarlo: Oración compuesta por
Evaluación (pág. 93)
coordinación copulativa.

 1. Respuesta abierta. Proponemos la siguiente respuesta: La vi- La intervención de lector no modifica el libro y tampoco el es-
sualización de una película en el cine es una actividad más pectador de cine o de televisión interviene o modifica lo que
pasiva que jugar a un videojuego, pues el espectador se limita ve: Oración compuesta por coordinación copulativa (nexo: y)
a observar y a reflexionar mientras que el usuario de videojue- y, también, por coordinación disyuntiva (nexo: o).
gos no deja de reaccionar a diversos estímulos. Además, el
jugador es proactivo y tiende a repetir una y otra vez diversas  5. Dicen que los videojuegos son un entretenimiento pasivo: CD. / El
acciones hasta completar el juego. hecho de que esté inmóvil no significa que su mente lo esté:
(CN). /
Los niños piden siempre que les cuenten el mismo cuento: CD.
 2. Párrafo 1: El espectador de cine teoriza y reflexiona continua-
mente mientras está viendo una película, pero se muestra  6. La función sintáctica que realiza en la oración el infinitivo
más pasivo que el jugador de videojuegos. / Párrafo 2: El reaccionar como forma nominal es de complemento del adje-
usuario de videojuegos participa de forma activa explorando tivo (CA).
continuamente el mundo en el que se mueve. / Párrafo 3: El Localiza en el texto otro infinitivo...: Sin embargo, esa actividad
usuario de videojuegos es proactivo y se interesa por todo lo mental nunca se detiene, a riesgo de perderse la película...
que rodea a sus juegos favoritos. Los videojuegos se parecen
en algunos aspectos a los cuentos infantiles.  7. Respuesta abierta.

35
5# La subordinación adjetiva

En contexto (pág. 95) Cuestión de lengua


a) Fíjate en los participios...: Desgarrados complementa al sustantivo  7. La concordancia hace referencia a las reglas gramaticales que
ojos; Retorcido complementa al sustantivo cuerpo; Llenos com- marcan la relación entre las palabras o los enunciados.
plementa al sustantivo dedos; Afilados complementa al sustanti-
vo cuernos. / ¿Concuerdan en género y número con estos? Todos Pon un ejemplo de la manera en que Cervantes...: En estos ejem-
los participios a los que se hace referencia concuerdan en gé- plos el verbo está en singular y el sujeto en plural: Su afabili-
nero y número con los sustantivos a los que acompañan. dad y hermosura atrae los corazones de los que trata a
servirla y a amarla (1.ª parte, capítulo XII). / El buen paso, el
Ahora, localiza los participios...: Forman parte de una forma regalo y el reposo, allá se inventó para los blandos cortesanos
verbal compuesta: había transcurrido, haberse curado, había (1.ª parte, capítulo XIII).
regresado. / ¿Admiten complementos? Sí, admiten complementos.
Proponemos algunos ejemplos: El tiempo había transcurrido  8. Se trata de una prosa «salvaje», extraordinariamente natural y
muy despacio (CCM); Podría haberse curado de su enfermedad expresiva. Abundan los anacolutos y los errores sintácticos.
(CR) con otros medicamentos (CCM); Había regresado antes
del amanecer (CCT).  9. A pesar de los errores y de las incoherencias, lo entendemos
b) Respuesta abierta. bien porque presenta la naturalidad de la expresión hablada.

10. Son interpolaciones.


Practica con los textos (págs. 97-98)
11. Alegrísimo.
Escribe tres adjetivos superlativos...: Facilísimo (fácil), habi-
Comprensión lísimo (hábil), cortísimo (corto), tristísimo (triste), riquísimo
(rico), larguísimo (largo), felicísimo (feliz), amabilísimo
 1. Las dificultades a la hora de editar el Quijote, de Miguel de (amable)...
Cervantes y la satisfacción en la escritura.
12. Lo (Det) normal (A) es (V) que (Conj) un (Det) amanuense (N) lo
 2. ¿A qué se dedica Francisco Rico? Es filólogo y en el momento de la (Pron) pasara (V) a (Prep) limpio (A) y (Conj) que (Conj) ese (Det)
entrevista se dedica a «devolver a los lectores el texto cervanti- manuscrito (N) se (marca de pasiva) llevara (V) a (Prep) la
no más fiel al original» en dos nuevas ediciones del Quijote. (Det) censura (N): Oración compuesta por subordinación sus-
¿Qué utilidad tiene en su trabajo disponer de concordancias elec- tantiva. Aparecen dos proposiciones sustantivas introducidas
trónicas?Ayudan a identificar los centenares de interpolacio- por la conjunción que (que un amanuense lo pasara a limpio
nes que aparecen en la obra maestra de Cervantes. / que ese manuscrito se llevara a la censura) y unidas por una
conjunción copulativa (y). Ambas funcionan como sujeto del
¿A qué atribuye los problemas que presenta el texto del Quijote ? verbo principal (es) y Lo normal es el atributo. Se llevara a la
Al hecho de que existen dos manuscritos de la obra. censura es una oración pasiva refleja.
¿Por qué motivo la novela tiene tanto éxito? Por las personalida- Está (V) hecho (participio) para (Prep) publicarse (V), pero
des de los protagonistas, don Quijote y Sancho. (Conj) la (Det) mentalidad (N) que (Pron) le (Pron) guía (V) no
(Av) es (V) la (Det) de (Prep) una (Det) novela (N) escrita (parti-
 3. Para Cervantes la escritura era un juego y el Quijote era un li- cipio) organizada (A): Oración compuesta por subordinación
bro de diversión. Lo concebía como un texto oral, algo pareci- adverbial adjetiva (que le guía), y además compuesta, tam-
do a una conversación entre amigos y, por ese motivo, va bién, por coordinación adversativa (pero la mentalidad que le
cambiando de tema continuamente a lo largo de la obra. guía no es la de una novela escrita organizada). El verbo prin-
cipal (Está) aparece complementado por el resto del enuncia-
 4. Anda enredado en la quijotesca búsqueda de los restos : el do y todo en conjunto funciona como atributo (hecho para...
adjetivo «quijotesca» puede hacer referencia a la tarea re- organizada). El sujeto de la oración principal es elíptico.
lacionada con don Quijote de la Mancha o, también, puede
estar empleado con el significado de ‘idealista’. / Se ha dedi- Cervantes (N) pasa (V) las (Det) correcciones (N) que (Pron) ha
cado a limpiar el texto de añadidos: se ha dedicado a conse- hecho (V) en (Prep) su (Det) autógrafo (N) al (Prep + Det) ori-

guir que el texto del Quijote sea lo más fiel posible al ginal (N) que (Pron) va (V) a (Prep) ir (V) a (Prep) la (Det) im-

original. / No tenía un sentido reverencial de él: no muestra prenta (N): Oración compuesta por subordinación adjetiva. El

reverencia o respeto. sujeto de la oración principal es Cervantes, el verbo es pasa y


está acompañado por un CD (las correcciones) cuyo núcleo
 5. 1. El amanuense pasa a limpio el original del autor. / 2. El ma- (correcciones) estás complementado por una proposición su-
nuscrito se lleva a la censura. / 3. El autor introduce modifica- bordinada adjetiva que realiza la función de CN (que ha he-
ciones en el original. / 4. Devuelven el original a la censura. cho... imprenta). Dentro de esta proposición aparece otra
proposición subordinada adjetiva, introducida también por el
 6. Descuidado, olvidadizo, desinteresado, despistado, despreo- pronombre relativo que, y cuyo antecedente es original. Apa-
cupado... rece una perífrasis aspectual ingresiva: va a ir.

36
Unidad 5 >  La subordinación adjetiva

Cuestión de léxico las pasa todas, y se equivoca a veces, y de ahí fenómenos,


como que, primero Sancho aparece con el asno, después le
13. Edición, imprenta, cajista, reeditar, manuscrito, autógrafo, roban el asno... / Y lo mete donde haga falta, cinco páginas más
original, amanuense, imprimir... allá de donde hubiera sido necesario. ¿Qué le costaba volver?
Pues no...: «olvidábaseme de decir». / Para nada, pero sobre
14. Fiel: honesto, noble, honrado, leal... / Infiel, desleal, ingrato, todo porque él lo pensaba como discurso oral. / Es un conti-
traidor... nuum a vuela pluma. / El Quijote es una tontería, con la historia
de ese loco... / Dice cosas tan bien dichas, y luego está pirado.
Multitud: gentío, muchedumbre, aglomeración, turba... / Es- / Hay un chiste clásico que dice: «Se nota que Cervantes...». /
casez, carencia, privación... Los horrores de sintaxis no plantean demasiados problemas.
Descuidado: abandonado, desaliñado, desidioso... / Atento,
cuidado, custodiado... 21. En ocasiones el entrevistador no formula preguntas directas
porque lo que hace es realizar valoraciones o sacar conclusio-
Sensato: cauto, prudente, cuerdo, moderado, precavido... / nes de lo que va diciendo Francisco Rico para que así el ex-
Insensato, imprudente, necio, desatinado... perto siga profundizando en el tema.

15. Continuum: Sucesión o serie compuesta de partes entre las 22. Respuesta abierta. Proponemos la siguiente: Quizá el registro
que no hay separación. es demasiado informal en algunas ocasiones teniendo en
cuenta el medio en el que aparece la entrevista (un periódi-
Relaciona los siguientes latinismos...: A priori: Antes de que su-
co), la relación entre los entrevistados (Francisco Rico es filó-
ceda. / Bis: Dos veces. / De facto: De hecho. / In situ: En el
logo e historiador y Javier Ors es periodista cultural), el tema
mismo sitio. / Lapsus: Equivocación.
(el estudio del Quijote) y la intención del texto (aportar infor-
mación sobre una obra literaria).
16. Manuscrito: Texto escrito a mano, especialmente el que tiene
algún valor o antigüedad, o es de mano de un escritor o per-
23. La estructura temática de la entrevista sigue un orden lógico,
sonaje célebre. / Amanuense: Persona que tiene por oficio es-
pero las preguntas evolucionan según la información que va
cribir a mano, copiando o poniendo en limpio escritos ajenos,
ofreciendo el entrevistado. Primero empiezan a hablar sobre
o escribiendo lo que se le dicta. / Autógrafo: Que está escrito
la tarea de Francisco Rico, después sobre los problemas que
a mano por el mismo autor. / Póstuma: Dicho de una obra, que
presenta el texto y, a continuación, sobre la importancia que
sale a la luz después de la muerte de su autor. / Anacoluto:
el propio Cervantes daba a su obra. Por último, Javier Ors, tras
Frase sin terminar. / Ficticio: Fingido, imaginario o falso.
la exposición de Rico, plantea el tema de la relevancia del
Quijote y de la forma de escribir de su autor.
17. A vuela pluma: Muy deprisa, a merced de la inspiración, sin
detenerse a meditar, sin vacilación ni esfuerzo.
24. Francisco Rico Manrique es filólogo e historiador de la litera-
Explica el significado de estas locuciones...: Sin par: Singular que tura de la Edad Media al Siglo de Oro. Ha trabajado en espe-
no tiene igual o semejante. / A cal y canto: Asegurando el en- cial sobre Petrarca y el Humanismo en Italia y en España,
cierro en un lugar de manera que no se pueda abrir, entrar o sobre el Lazarillo de Tormes y sobre el Quijote. Desde 1986 es
salir. / Al pie de la letra: Sin variación, sin añadir ni quitar académico de la RAE.
nada. / De buena gana: Con gusto o voluntad. / A mata caballo:
Atropelladamente, muy deprisa.
Expresión
Averigua si alguna de ellas...: A matacaballo también puede
escribirse en una sola palabra. 25. Respuesta abierta.

26. Respuesta abierta.


Análisis del texto
27. Actividad de expresión oral.
18. Título (suele ser una cita del propio entrevistado), presenta-
ción (breve introducción o resumen), reproducción del diálo-
go entre el entrevistador y el entrevistado. La norma ortográfica (pág. 99)

19. Mientras tanto, Cervantes tenía su autógrafo... (Establece una  1. Elio no había preparado su trabajo a tiempo. Nunca conseguía
relación temporal). / Entonces, devuelven el original... (Esta- terminar las cosas. / El veterinario lidiaba cada día con un hatajo
blece una relación temporal). / Pasa las correcciones que ha de ovejas y carneros. / Cuando lleves el agua al fuego ten cuida-
hecho en su autógrafo al original que va a ir a la imprenta, do al tocar la olla, podrías quemarte. / Para mí es un placer la
pero no las pasa todas... (Establece una relación de oposi- costura. Mientras enhebro la aguja mantengo la cabeza lejos de
ción). / Sancho aparece en el asno, después le roban el asno los problemas. / El reciclaje no solo consiste en seleccionar nues-
y luego vuelve a estar con el asno (Establecen una relación tros desechos, también hay que reusar nuestras cosas y darles
temporal). Eso es porque Cervantes lo hace mal y porque una nueva oportunidad. / Cuando el pastor siente que los lobos
hubo dos manuscritos (Establece una relación de causalidad). acechan a las ovejas, alambra sin más remedio sus tierras.

20. En un pasaje se lee «alegrísimo». Eso no es cristiano. No lo usa  2. Hirsuto, hipotiroidismo, holgado, adherencia, acción, oque-
Cervantes. / Hay dos manuscritos. Ese es el problema. El autor dad, hernia, alhaja, huele, haga, orfandad, huevo, vihuela,
hace su autógrafo... / En ese tiempo a él, con la cabeza que gentilhombre, ¡huy!, haz, haba, huidizo, herencia, holografía,
tenía, se le ocurrieron un montón de modificaciones... / Pero no oscense.

37
Unidad 5 >  La subordinación adjetiva

 3. Holo-: holocausto, holograma, holoceno, holografía... / Hemo-: viajeros impuntuales durmieron en un hotel. / Las ciudades
hemodiálisis, hemofilia, hemodinámica... / Hecto-: hectogra- muy contaminadas podrán solicitar la subvención.
mo, hectolitro, hectovatio, hectómetro... / Hepta-: heptasílabo,
Justifica si las proposiciones subordinadas...:Las personas que
heptagonal, heptaedro, heptacordo... / Ecto-: ectodermo,
vengan tendrán un premio: especificativa. / Los viajeros, que ha-
ectoparásito, ectoplasma... / Homeo-: homeopatía, homeosta-
bían perdido el avión, durmieron en un hotel: explicativa. / Las
sis, homeotermia...
ciudades que tengan un alto índice de contaminación podrán solici-

 4. tar la subvención: especificativa.

Vibrante simple Vibrante múltiple  3. El piso en el que vivo es de mi hermano: CCL. / Siempre recordaré
coro israelita la amistad que compartimos: CD. / He visto cosas de las que prefie-
abrazo risco ro no hablar: CR. / Los alumnos que fueron a la cena estaban
dragar prórroga encantados: Sujeto.

abreviar subrepción
prado honra  4. Me gustaría conocer los pueblos de los que nos hablaste: CR.
/ No está en ninguna de las tiendas por las que pasé: CCL. / El
olvidar contrarréplica
tren en el que viajamos es de alta velocidad: CCL. / Inscribí a
mis hijos en la academia de la que nos hablaron: CR.
 5. Virreina, contrarreloj, contrarrestar, posromántico, subregión,
subrayar, abrogar.
 5. Mi prima, a la cual conoces, está de viaje: CD. / Las niñas de
las cuales hablamos van a Primaria: CR. / Los sellos, los cua-
 6. Ántrax, estricto, dúplex, expurgar, tórax, espontáneo, exprés,
les vendiste, eran de mi colección: CD. / Me extrañó el miedo
estructura, estremecer, látex, espléndido, expropiar, telefax,
con el cual nos acogieron: CCM.
espectáculo, expletivo, sílex.
 6. El cine cuyas(determinante relativo) butacas son tan cómodas
 7. Manifestar, mostrar en público: exhibir. / Instrumento de percu-
está en obras. / Han sido ellos quienes (pronombre relativo) no
sión compuesto por láminas horizontales que se hace sonar gol-
llegaron a tiempo. / Recuerdo la noche cuando (adverbio relativo)
peándolas con dos baquetas: xilófono. / Conceder la extradición a
nos conocimos. / Mi prima Eva, a quien (pronombre relativo) hace
una persona reclamada por la justicia de otro país: extraditar. /
tiempo que no veo, se casa este verano.
Relajamiento físico o psíquico:
relax. / República democrática que
ocupa la parte meridional de América del Norte: México. / Aparato
que se utiliza para sofocar un fuego: extintor. Sustantivación de subordinadas adjetivas
(págs. 104-105)
 8. Obstáculo: obs-tá-cu-lo / Andrajoso: an-dra-jo-so / Esfinge: es-
fin-ge / Instrucciones: ins-truc-cio-nes / Sublunar: sub-lu-nar /  7. ¿Te gusta el restaurante (antecedente: Sujeto) donde (adverbio
Abstracto: abs-trac-to / Instaurado: ins-tau-ra-do / Obcecado: relativo: CCL) comimos? / Monta el mueble del modo (anteceden-
ob-ce-ca-do / Substituir: subs-ti-tuir / Infrahumano: in-fra-hu- te: CCM) como (adverbio relativo: CCM) te explicaron. / Recuer-
ma-no / Transalpino: trans-al-pi-no / Cactus: cac-tus. do los veranos (antecedente: CD) cuando (adverbio relativo:
CCT) estábamos todos juntos.
 9. Substraer, posmeridiano, mnemotecnia, innocuo, postdata,
transgredir, psicofonía, nómico, psoriasis, subscriptor, sépti-  8. El piso que vi era un ático: El que vi era un ático. / Coge todas las
mo, psicosis. cosas que necesites:Coge todas las que necesites. / El concur-
sante que gane será premiado:
El que gane será premiado; Quien
gane será premiado. / Retiraron los alimentos que habían cadu-
Actividades (pág. 107) cado: Retiraron los que habían caducado.

La subordinación adjetiva  (págs. 100-103)  9. Los que hayan hecho el examen (Sujeto) pueden salir. / Quienes
vayan a la playa este verano (Sujeto) tendrán que protegerse del
 1. Pensó que (conjunción que introduce una proposición subordi- sol. / Ayuda a la que lleva dos bolsas en la mano (CD). / Pregún-
nada sustantiva) regalaría la ropa que (pronombre relativo que tame cuanto te interese (CD).
introduce una proposición subordinada adjetiva) no utiliza. /
Juan afirma que (conjunción que introduce una proposición su-
bordinada sustantiva) vio el avión que (pronombre relativo que El participio  (pág. 106)
introduce una proposición subordinada adjetiva) se estrelló. /
Dime que (conjunción que introduce una proposición subordina- 10. Sus argumentos fueron rebatidos (forma pasiva) por el fiscal. /

da sustantiva) has merendado lo que (pronombre relativo que in- Los árboles plantados este invierno ya han florecido (tiempo
troduce una proposición subordinada sustantiva) te preparé ayer. compuesto). / Tiene preparadas (perífrasis verbal) las cosas para
la fiesta. / Si me hubieras preguntado (tiempo compuesto), ya lo

 2. Las personas que vengan tendrán un premio (antecedente: perso- sabrías.

nas). / Los viajeros, que habían perdido el avión, durmieron en un


hotel (antecedente: viajeros). / Las ciudades que tengan un alto 11. Los alumnos de Bachillerato, cansados (participio concertado) de
índice de contaminación podrán solicitar la subvención (antece- esperar, volvieron a sus casas en autobús. / Acabada (participio
dente: ciudades). absoluto) la función, los espectadores abandonaron la sala sin
hacer ruido./ Estrenada (participio absoluto) la película, la ac-
Sustituye la proposición...: Respuesta abierta. Proponemos las triz, entusiasmada(participio concertado) con los comentarios,
siguientes: Las personas puntuales tendrán un premio. / Los firmó autógrafos a todos los admiradores.

38
Unidad 5 >  La subordinación adjetiva

A (Prep) un (Det) olmo (N) viejo (A), hendido (Participio) por


Análisis de la oración (pág. 109)
(Prep) el (Det) rayo (N), con (Prep) las (Det) lluvias (N) de
(Prep) abril (N) y (Conj) el (Det) sol (N) de (Prep) mayo (N) al-
 1. Explicó su experiencia personal a todas aquellas personas que ha- gunas (Det) hojas (N) verdes (A) le (Pron) han salido (V): Ora-
bían perdido la esperanza y necesitaban una orientación:
a) Propo- ción simple en la que destaca una construcción de participio
sición principal: Explicó su experiencia personal a todas concertado que funciona como CN del sustantivo olmo: hen-
aquellas personas. De ella dependen b y c. / b) Proposición dido por el rayo.
subordinada adjetiva especificativa: que habían perdido la
esperanza, CN del antecedente personas. El pronombre rela- Los (Det) aficionados (N) abandonaron (V) el (Det) estadio (N)
tivo que desempeña la función de sujeto dentro de la subordi- por (Prep) la (Det) puerta (N) principal (A) acompañados (Partici-
nada. / c) Proposición subordinada adjetiva especificativa: pio) por (Prep) los (Det) músicos (N) de (Prep) la (Det) banda
que necesitaban una orietanción, CN del antecedente perso- (N) que (Pron) habían actuado (V) durante (Prep) el (Det) des-
nas. El pronombre relativo que desempeña la función de suje- canso (N): a) Proposición principal: Los aficionados abando-
to dentro de la subordinada. A su vez, b y c están coordinadas naron el estadio por la puerta principal. / b) Construcción de
entre sí mediante el enlace copulativo y. participio concertado que funciona como CN del sustantivo
aficionados: acompañados por los músicos de la banda. De
Los estudiantes, que han aprobado todo, tendrán el premio que les
ella depende la proposición c. / c) Proposición subordinada
prometimos: a) Proposición principal: Los estudiantes tendrán el
adjetiva: que habían actuado durante el descanso. Funciona
premio. De ella dependen las proposiciones b y c. / b) Proposi-
como CN del antecedente los músicos de la banda. El pro-
ción subordinada adjetiva explicativa: que han aprobado todo,
nombre relativo que desempeña la función de sujeto dentro
CN del antecedente estudiantes. El pronombre relativo que
de la subordinada.
desempeña la función de sujeto dentro de la subordinada. / c)
Proposición subordinada adjetiva especificativa: que les pro- Es (V) cierto (A) que (Conj) la (Det) tecnología (N) ha produ-
metimos, CN del antecedente premio. El pronombre relativo cido (V) un (Det) mundo (N) cada (Det) vez (N) más (Av) inter-
que desempeña la función de CD dentro de la subordinada. conectado (A), cuyos (Det) conflictos (N) son (V) globales (A)
y (Conj) cuyas (Det) soluciones (N) también (Av) deberían serlo
El mago, que llena todos los teatros, actúa mañana en Segovia: a)
Proposición principal: El mago actúa mañana en Segovia. De (Perífrasis verbal): a) Proposición principal: Es cierto. De
ella depende la proposición b. / b) Proposición subordinada ella depende la proposición b. / b) Proposición subordina-
adjetiva explicativa: que llena todos los teatros, CN del antece- da sustantiva que realiza la función de sujeto respecto a la
dente mago. El pronombre relativo que desempeña la función proposición principal. De ella dependen las proposiciones
de sujeto dentro de la subordinada. c y d. / c) Proposición subordinada adjetiva: cuyos conflic-
tos son globales. Funciona como CN del antecedente mun-
La niña, que tiene familia en
Francia, habla perfectamente el do. El adjetivo relativo cuyos realiza la función de CN del
francés: a) Proposición principal: La niña habla perfecta- sustantivo conflictos. / d) Proposición subordinada adjetiva:
mente el francés. De ella depende la proposición b. / b) cuyas soluciones también deberían serlo. Funciona como
Proposición subordinada adjetiva explicativa: que tiene fami- CN del antecedente mundo. El adjetivo relativo cuyas reali-
lia en Francia. Funciona como CN del antecedente niña. El za la función de CN del sustantivo soluciones. Las proposi-
pronombre relativo que desempeña la función de sujeto ciones c y d son coordinadas. Están unidas mediante la
dentro de la subordinada. conjunción y.
Esta tarde veremos a los chicos holandeses que han venido a
A (Prep) la (Det) derecha (N) del (Prep + Det) santo (N), apre-
aprender español: a) Proposición principal: Esta tarde vere- sado (Participio) por (Prep) un (Det) collar (N) de (Prep) perro
mos a los chicos holandeses. De ella depende la proposición (N), se yergue (V) un (Det) lechón (N) de (Prep) madera (N) ne-
b. / b) Proposición subordinada adjetiva especificativa: que gra (A): Oración simple con una construcción de participio
han venido a aprender español. La proposición subordinada concertado que funciona como CN del sustantivo santo: apre-
funciona como CN del antecedente chicos. El pronombre sado por un collar de perro.
relativo que desempeña la función de sujeto dentro de la
subordinada.
Unos amigos, que tienen una casa en la montaña, nos han invitado Evaluación (pág. 111)
este verano: a) Proposición principal: Unos amigos nos han
invitado este verano. De ella depende la proposición b. / b)  1. El autor guarda gratos recuerdos de sus primeros años de
Proposición subordinada adjetiva explicativa: que tienen una infancia en los que conoció los relatos de aventuras. Cuan-
casa en la montaña, CN del antecedente amigos. El pronom- do tenía once años su vida cambió, cuando se fue a vivir a
bre relativo que realiza la función de sujeto en la subordinada. Lima, y encontró refugio en los libros y en lectura. Y, tam-
bién, explica el gozo que supone para él inventar y cons-
 2. El (Det) modo (N) como (Av) preparaste (V) la (Det) tortilla (N) truir historias.
es (V) un (Det) secreto (N) que (Pron) no (Modificador verbal)
confesará (V) a (Prep) nadie (Pron): a) Proposición principal:  2. Silente: calladas, silenciosas. / Agitada: alterada, turbada. /
El modo es un secreto. De ella dependen b y c. / b) Proposi- Díscolo: revoltoso, rebelde. / Abatido: desalentado, extenuado.
ción subordinada adjetiva: como preparaste la tortilla. Fun-
ciona como CN del antecedente modo. El adverbio relativo  3. El relativo funciona como sujeto en la oración Es una expe-
como desempeña la función de CCM dentro de la subordina- riencia que me sigue hechizando.
da. / c) Proposición subordinada adjetiva: que no confesará
a nadie. Funciona como CN del antecedente secreto. El Indica la función de los relativos restantes: Una realidad que
pronombre relativo que desempeña la función de CD dentro (CD) viví. / Escribir fue jugar un juego que (CD) me celebra-
de la subordinada. ba la familia.

39
Unidad 5 >  La subordinación adjetiva

 4. El vértigo al que nos conduce una novela en gestación. / Refu- verbo haber) abatido.
/ Intentar convertir esa niebla agitada de
giarme en esos mundos donde vivir era exaltante. / En la Prefac- fantasmas en una historia:Se trata de un participio con valor
tura de Piura, en cuyos entretechos anidaban los murciélagos. adjetival que concuerda en género y número con el sustantivo
al que acompaña (niebla).
 5. Era (V) un (Det) señor (N) alto (A) y (Conj) buen (A) mozo (N),
de (Prep) uniforme (N) de (Prep) marino (N), cuya (Det) foto (N)  7. Mi madre me reveló que aquel caballero, en verdad, estaba
engalanaba (V) mi (Det) velador (N): a) Proposición principal: vivo.
Era un señor alto y buen mozo, de uniforme de marino. De
ella depende la proposición b. / b) Proposición subordinada  8. Respuesta abierta. Proponemos las siguientes: Ha sido la
adjetiva especificativa: cuya foto engalanaba mi velador. Fun- que señala la salida del túnel. / Cuando toma forma y pare-
ciona como CN del antecedente señor. El adjetivo relativo ce empezar a vivir por cuenta propia, con los que se mue-
cuya realiza la función de CN del sustantivo foto. ven, actúan, piensan, sienten y exigen respeto y
consideración...
 6. En todas las circunstancias en que me he sentido (Participio con
valor verbal pues forma parte de un tiempo compuesto con el  9. Respuesta abierta.

40
La subordinación adverbial.
6# Estructuras oracionales bimembres
De pequeño, cuando estabas en Primaria, decían que eras un
En contexto (pág. 113)
genio. Oración compuesta por subordinación. Consta de una
a) fíjate en las construcciones…:
Ambas Construcciones se pueden proposición principal a): De pequeño decían; una proposición
cambiar por un adverbio o por un gerundio. adverbial de tiempo b:) Cuando estabas en Primaria, y una su-
bordinada sustantiva que funciona como CD de la principal c:)
b) escucha con atención…: No puedo darle esa información porque
que eras un genio. Principal a): De pequeño decían. Sujeto:
no me consta (causal). / Facilíteme su número si es tan amable
elidido (3ª persona del plural); predicado: De pequeño (CCT)
(condicional).
decían (núcleo). Proposición b): Cuando estabas en Primaria:
cuando (R/CCT) Sujeto: elidido (2ª persona del singular); predi-
cado: estabas (núcleo) en Primaria (CCT) c) que (R) Sujeto:
PRACTICA CON LOS TEXTOS ( págs. 114-117) elidido (2ª persona sing.); predicado: eras (núcleo) un genio
(Atr).
Comprensión
 6. De pronto, Raúl oyó una voz que le saludaba; se dio la vuelta
 1. Indica dónde y cuándo…: Deducimos que la acción ocurre en el y vio a un muchacho de los cursos inferiores. Le sonaba su
patio del colegio durante la hora del recreo. / ¿Qué le ocurre a cara pero no estaba seguro de quién era. Por eso quiso sa-
raúl? ¿A quién se lo cuenta?: Raúl lleva 8 días sin dormir, sueña ber su nombre. El chico le respondió que se llamaba Daniel
con los ciervos y se lo cuenta a Daniel. / ¿Qué les ocurre a los Castro.
ciervos?: Según Raúl, los ciervos están enfermos y mueren,
Fue entonces cuando Raúl recordó que cuando Daniel estaba
excepto el ciervo Rojo. en Primaria todos opinaban que era un genio. Daniel no le
quiso dar importancia a lo de genio y quiso saber el motivo por
 2. En el texto se atribuye…: Raúl, en lugar de devolver el balón, lo el que Raúl presentaba tan mal aspecto.
lanzó lejos y esto molestó al chico que se lo reclamaba. Al
pasar junto a Raúl, debió insultarle en voz baja, lo que irritó a Raúl le confiesa que últimamente no duerme porque cada vez
Raúl y se lanzó sobre él, dándole una brutal paliza. que lo hace, el Ciervo Rojo viene y lo mata. Daniel cree que
Raúl sufre pesadillas en el sueño.
 3. Vuelve a leer la sinopsis del libro…: Deducimos que este frag-
Ahora, imagina el diálogo interior…: Respuesta abierta.
mento pertenece al nudo, pues se ha originado un conflicto
entre Raúl y los otros chicos.
Cuestión de léxico
 4. Indica si las afirmaciones…: Corregimos las falsas en negrita.
Jorge y Guillermo son los mejores amigos de Raúl (F). / Una  7. Enjambre: abejas / bandada: pájaros / jauría: perros / rebaño:
parte de la mente de Raúl sí es consciente de que las imáge- ovejas / banco: peces / piara: cerdos / yeguada: caballos y ye-
nes de los ciervos no son reales. (F) / Guillermo confía en que guas / camada: crías de animales nacidas de un mismo parto /
las cosas mejoren. (V) / El balón que llegó hasta Raúl era de boyada: bueyes y vacas.
un grupo de muchachos que estaban jugando al fútbol. (F) /
Fueron necesarios tres adultos para que Raúl dejara de gol-  8. Escribe una lista… Respuesta abierta, ofrecemos la siguiente:
pear al otro muchacho. (F) perdiz, araña, periodista, lince, taxista, pantera…

 9. Un ojo de la cara: Significa que es excesivamente caro. Hace


Cuestión de lengua referencia al valor e importancia de nuestros dos ojos. / en un
abrir y cerrar de ojos: Significa que lo ha hecho muy rápido;
 5. Analiza sintácticamente… como lo hacemos al pestañear. / Ándate con ojo: Significa
Raúl ignora la fecha en que vive. Oración compuesta por sub- que hay que tener precaución; es decir, observar y andar
ordinación adjetiva. Proposición principal a): Sujeto: Raúl con tiento.
(núcleo) Predicado: ignora (núcleo) la fecha (CD). Proposi-
Escribe otras oraciones…. Respuesta abierta. Ofrecemos las
ción subordinada adjetiva o de relativo b) introducida por el
siguientes: Este coche cuesta un riñón. / Escarmentar en ca-
nexo que (R); realiza la función de CN. Sujeto: elidido (3ª
beza ajena. / Fue mano de santo. / Entré con buen pie. / No le
persona del singular) Predicado: en que (CCT) vive (núcleo).
tocó ni un pelo. / No tiene pelos en la lengua.
Pero da igual, porque cada noche me mata. Estructura ora-
cional bimembre que consta de una proposición principal a) 10. Salvaje: feroz, cruel, inhumano / desmedido: descomunal, inter-
introducida por la conjunción pero, y una subordinada b) minable / irresistible: insoportable, arrollador, insufrible.
introducida por el nexo porque. Entre ambas se establece
una relación de causa. a) Pero da igual. Sujeto: elidido (3ª 11. Palabras de origen latino: son aquellos vocablos que, tomados
persona del singular) Predicado: da igual (locución verbal; del latín, han evolucionado fonéticamente hacia las lenguas
núcleo) b) porque cada noche me mata. Sujeto: elidido (3ª románicas. Por ejemplo: populu, que ha dado pueblo. Latinis-
persona del singular) Predicado: cada noche (CCT) me (CD) mos: son giros o palabras tomados directamente del latín; por
mata (núcleo). ejemplo: in extremis, versus, a priori.

41
Unidad 6 >  La subordinación adverbial. Estructuras oracionales bimembres

Análisis del texto pensiones : proposición adjetiva explicativa separada por co-
mas. / El tren no salió a su hora , así que nuevamente llegamos

12. Identifica…: El diálogo entre Daniel y Raúl. / Raúl no puede a la cita tarde y angustiados: separa la principal de la subordi-

seguir la explicaciones de las clases y tampoco es consciente nada, señalando consecuencia.


de la fecha en que vive. / La pelea entre Raúl y los chicos que
jugaban al fútbol.  2. Coloca los puntos y las comas
¿Qué elementos…?: Los elementos que nos ayudan a diferen- —— ¿Analógicos? ¿Digitales? ¡Biológicos!
ciar las partes son el diálogo y la narración de los hechos.
—— Pero, ¿hasta cuándo?
Traza un esquema temporal…: Durante el recreo tiene lugar el
A lo largo de los siglos, los seres humanos nos hemos vali-
diálogo entre Daniel y Raúl; durante las horas de clase (hay
do de diversos dispositivos para mejorar, aumentar o regu-
un salto atrás), Raúl se da cuenta de que no puede seguir las
lar nuestras capacidades. Hemos creado el vestido para
explicaciones del profesor, y deducimos que es en otro recreo
protegernos del frío, del viento y del sol. Y, por medio de
cuando surge la pelea.
herramientas e instrumentos creados por nosotros, hemos
aumentado artificialmente nuestra capacidad física para
13 marcadores temporales…: lleva ocho días, ahora, últimamente,
proveernos alimento y refugio. Mazas y martillos nos ayu-
a veces, hace mucho… /
dan a golpear con mayor fuerza, anteojos con lentes de
¿En qué parte…?: Los marcadores temporales son más fre- aumento nos permiten ver aquello que nuestra vista no
cuentes en la parte del texto donde Raúl se da cuanta de que alcanza a distinguir con suficiente claridad. Inventamos
está perdiendo concentración y noción del tiempo. micrófonos y altavoces para potenciar nuestra voz, denta-
duras postizas, brazos y piernas ortopédicas de creciente
sofisticación técnica, audífonos y otras prótesis de diferen-
Expresión tes tipos, para reparar o mejorar artificialmente las funcio-
nes de un órgano o de un miembro de nuestro cuerpo (o
14. En parejas…: Debe procurarse que la entonación sea adecua- parte de él).
da, acompañada del lenguaje no verbal.
 3. A ti , lector , cordialmente …: Al dejar una oración incompleta.
15. ¿Qué efectos…?: La falta de sueño le produce alucinaciones, / Quizá ganemos , pero … está difícil : para expresar temor,
enrojecimiento de los ojos, pérdida de la noción temporal, duda, sorpresa, expectación. / «Caminante […] se hace camino
alejamiento de la realidad, irritabilidad… / busca informa- al andar .»: Para indicar en una cita textual una parte omitida.
ción…: Los efectos de la falta de sueño son: estrés, cansancio, / Los alimentos lácteos son : el queso , el yogur …: Al dejar una
agotamiento, aumento de la tensión, ojeras, mal humor, pérdi- enumeración incompleta.
da de memoria, desórdenes psicológicos… / ¿has padecido...?
Respuesta abierta.  4. Escribe coma, punto…
16. Escribe un breve…: Respuesta libre. Se puede describir como Por la ventana, un gran mural de cristal que provocaba la
un chico alto y delgado, con ojeras, mirada perdida… curiosidad; solo se veía una esperpéntica masa forestal: hele-
chos, hayas, robles y zarzas dibujaban siluetas fantasmagóri-
17. ¿Te gustaría seguir…?: Respuesta libre. Los motivos para se- cas, incluso a plena luz del día. La actitud de los adultos ante
guir leyendo podrían ser: averiguar si Raúl vuelve a recobrar la el cristal era desafiante, como batallando contra el miedo; la
noción de la realidad; el misterio de los ciervos; las conse- de los niños, furtiva para que la memoria no retuviera una de
cuencias de la pelea… esas imágenes que más tarde les pudiera acompañar entre
las sábanas. Los pequeños no paraban de moverse, agitán-
18. Resume el contenido del vídeo: El 20 de abril de 1999 dos jóve- dose y haciendo cada vez más ruido, convirtiendo sus jue-
nes de 17 y 18 años respectivamente entraron en un instituto gos, sus gritos y sus carreras nerviosas en un escudo que no
norteamericano armados con cuchillos, bombas caseras y, les permitía oír el exterior. La noche caía, y uno de los adultos
escopetas, y mataron a 12 compañeros y a un profesor. El decidía echar las cortinas, tal vez para poder dejar de imagi-
hecho causó un gran impacto en la sociedad americana y nar lo que había fuera. Sin embargo, para uno de los niños,
sirvió de argumento para una película. / Los sucesos….: Res- esto provocó un enorme sentimiento de indefensión; ahora
puesta abierta. / ¿Crees que desde…?: Respuesta abierta. / En- no podría ver cómo el peligro acechaba ni cómo o cuándo
tablad un debate… Respuesta abierta. llegaría hasta ellos. La angustia llenó su garganta y su gargan-
ta inundó la sala con un pavoroso grito al que los demás ni-
ños acompañaron; antes de que oscureciera completamente,
los coches emprendieron el camino de regreso, dejando de-
La norma ortográfica (pág. 117)
trás de la ventana un terrorífico imaginario.
 1. La obra presentaba frescura , originalidad , osadía y una gran
dosis de humor : enumeración de elementos. / Los alumnos de  5. Ordena las siguientes oraciones…
primero hicieron la invitaciones al acto ; los de segundo , carte- La compañía minera Río Tinto recortará 14 000 puestos de
les ; los de cuarto eligieron la música , y los profesores actua- trabajo de su plantilla, presionada por la necesidad de redu-
ron : relación de elementos separados por punto y coma, en cir su deuda neta, que a 31 de octubre suma 38 900 millo-
el que el último elemento se separa por coma. / Chicos , va- nes de dólares.
mos a empezar la ruta en ese cruce y llegaremos hasta el refugio
de la montaña : vocativo, aislado de la oración. / El acuerdo, El grupo está procediendo con un programa de inversiones
que debe ser ratificado por el senado , incluye mejoras en las para alcanzar su meta de reducción de deuda, a través de la

42
Unidad 6 >  La subordinación adverbial. Estructuras oracionales bimembres

reducción del gasto. De este modo, la minera ha señalado  9. Identifica: mientras trataba de encontrarlo (Sub)/ acercándose a
que para 2009 espera reducir su gasto total de capital, desde las… (Ger) / en cuanto su espalda tocó… (Sub) / cuando apa-
los más de 9 000 millones de dólares actuales, hasta 4 000 reció… (Sub) / había estado persiguiendo… (Ger) / levantando
millones de dólares. la voz… (Ger) / mientras cogía… y la colocaba… (Sub) /rozan-
do los bordes… (Ger).
La compañía ha destacado que, dadas las medidas que se
están tomando, el dividendo a repartir entre los accionistas
se mantendrá en los 1,36 dólares, al mismo nivel del ejerci-
Análisis de la oración (pág. 125)
cio 2007.
 1. Me encontró donde íbamos todas las tardes. La oración está for-
 6. Respuesta abierta. Ofrecemos los siguientes ejemplos: detrás mada por una proposición principal a), de la que depende
de encabezamientos en cartas, instancias … Apreciados señores: una proposición subordinada adverbial de lugar b), que des-
Les remito esta carta con el motivo… / resumir el sentido global empeña la función de CCL. Proposición principal a): Me en-
de lo expresado después de una enumeración . Fiebre, tos y muco- contró. Sujeto: elidido (3ª persona singular); predicado: me
sidad: tres síntomas de la gripe. / Antes de iniciar los elementos (CD) encontró (núcleo). Proposición subordinada adverbial de
de una enumeración : se reunieron todos sus amigos: Mónica, lugar b): donde íbamos todas las tardes. Donde (R/CCL); suje-
Paula, Luis, Fernando… / para citar textualmente un documen- to elidido (1ª persona del plural); predicado: íbamos (núcleo)
to: André Breton escribió: «Haced abstracción de vuestro ge- todas las tardes (CCT).
nio, de vuestros talentos y de los ajenos».
Cuando acabaron el trabajo de sociales, vieron una película. La ora-
ción está formada por una proposición principal a): vieron una
película. De ella depende una subordinada adverbial de tiempo
ACTIVIDADES (págs. 123-125)
b), que desempeña la función de CCT: cuando acabaron el tra-
bajo de Sociales. Proposición principal a): Sujeto elidido (3ª per-
La subordinación adverbial  (pág. 123) sona del plural); predicado: vieron (núcleo) una película (CD).
Proposición subordinada adverbial de tiempo b): cuando (R/
CCT); sujeto elidido (3ª persona del plural); predicado: acabaron
 1. Cuando salí, empezó a llover. Entonces; adverbial de tiempo. /
(núcleo) el trabajo de Sociales (CD).
Envuelve los paquetes como te expliqué. Así; adverbial de
modo. / Ana llegó antes de que pasara la comitiva. Previa- Todos los excursionistas cogieron el teleférico como habían acorda-
mente; adverbial de tiempo. / Volveremos a casa por donde do. La oración está formada por una proposición principal a):
siempre (volvemos). Por allí; adverbial de lugar. Todos los excursionistas cogieron el teleférico. De ella depende
una proposición adverbial b) que funciona como CCM: como
 2. Organizaron los grupos según se había dicho en la reunión. / habían acordado. Proposición principal a): Sujeto: todos los
Tan pronto acabó la función, los espectadores abandonaron la excursionistas; predicado: cogieron (núcleo) el teleférico (CD).
sala. / Me trasladaré en cuanto pueda. / Conforme iban lle- Proposición subordinada b): como (R/CCM); predicado: habían
gando, ocupaban sus asientos. acordado (núcleo); sujeto elidido (3ª persona del plural).
Emilio habló con sus padres como le había aconsejado el profesor.
 3. Indica qué tipo…: Cómo, introduce una sustantiva; Donde, una La oración está formada por una proposición principal a): Emi-
adjetiva. / Donde, una adjetiva. / Como, adverbial de modo. / lio habló con sus padres. De ella depende una proposición ad-
Cuando, adverbial de tiempo. verbial b) que funciona como CCM de la principal: como le
había aconsejado el profesor. Proposición principal a): Sujeto,
 4. Distingue…: Donde: adverbio relativo. / Cuando: adverbio de Emilio; predicado: habló (núcleo) con sus padres (complemen-
tiempo. / En cuanto: adverbio de tiempo. / Cuando: adverbio to circunstancial de compañía). Subordinada b): como (R/
de tiempo. / Cómo: adverbio interrogativo de modo. / Como: CCM) le (CD) había aconsejado (núcleo) el profesor (sujeto).
adverbio de modo. / Como: adverbio relativo. / Cuándo: ad-
El cantante abandonará los escenarios en cuanto acabe la gira que
verbio interrogativo de tiempo. / Donde: adverbio relativo.
tiene acordada. Oración formada por tres proposiciones a), b) y
c). Proposición principal a): El cantante abandonará los esce-
 5. Cuando: adverbio de tiempo. / Cuando: adverbio relativo. /
narios. De ella depende la proposición subordinada adverbial
Cuándo: adverbio interrogativo. / Cuando: adverbio de tiempo.
de tiempo b) que realiza la función de CCT: tan pronto como
/ Cuando: adverbio relativo.
acabe la gira. Y de esta proposición adverbial depende la c),
que es una subordinada adjetiva o de relativo: que tiene acor-
 6. Vendiendo: funciona como adverbio; como verbo lleva un CD (re- dada. Proposición principal a): Sujeto: el cantante; predicado:
lojes). / Practicando: funciona como adverbio; como verbo lleva abandonará (núcleo) los escenarios (CD). Proposición subor-
un CC (cada día). / Sonriendo: funciona como adverbio. / Estuvo dinada b): Sujeto elidido (3ª persona del singular); redicado:
ordenando: funciona como adverbio; forma una perífrasis verbal y
en cuanto (R/CCT) acabe (núcleo) la gira (CD). De esta propo-
lleva un CD (las fotos del viaje). / Viendo: funciona como adverbio; sición depende la adjetiva c), que complementa al sustantivo
como verbo lleva CD (películas de ciencia ficción). gira: Sujeto elidido (3ª persona del singular); predicado: que
(R/CD) tiene (núcleo) acordada (complemento predicativo).
 7. Acabando: construcción absoluta. / Celebrando: construcción
concertada. / Comiendo: construcción concertada. / Cogiendo:  2. Cambiaré los neumáticos del coche como me recomendó el mecáni-
construcción concertada. co. Cambiaré (V) los (Det) neumáticos (N) del (Prep+ Det)
coche (N) como (Av) me (Pron) recomendó (V) el (Det) mecá-
 8. Al acabar: CCT/ estudiando: CCM / terminadas: CCT / preparando: nico (N). La proposición principal a) es Cambiaré los neumá-
CCT; llamarte: CR / comprobados: CCT. ticos del coche. De esta depende una proposición subordinada

43
Unidad 6 >  La subordinación adverbial. Estructuras oracionales bimembres

adverbial b) que funciona como CCM: como me recomendó el mentada por dos proposiciones subordinadas adverbiales
mecánico. Proposición principal a): sujeto elidido (1ª persona (b) y c)) que están coordinadas entre sí por la conjunción y.
del singular); predicado: cambiaré (núcleo) los neumáticos La proposición c) lleva a su vez una subordinada sustantiva
del coche (CD) [neumáticos (núcleo) del coche (CN)]. Propo- interrogativa d). Proposición principal a): puedes ir por el
sición subordinada adverbial de modo b): Sujeto: el mecánico periódico dando un paseo. Sujeto: elidido (2ªpersona singu-
(núcleo); predicado: como (R/CCM) me (CI) recomendó (nú- lar); predicado: puedes ir por el periódico dando un paseo.
cleo). Puedes ir (núcleo) por el periódico (CC de finalidad) dando
un paseo (CCM) [dando (núcleo) un paseo (CD del gerundio
Ayer volví donde me dijiste cuando nos vimos. Ayer (Av) volví (V)
dando)]. Proposición subordinada adverbial de tiempo b):
donde (Av) me (Pron) dijiste (V) cuando (Av) nos (Pron) vimos
Mientras escribo la nota del supermercado. Sujeto elidido (1ª
(V). La proposición principal a) es: Ayer volví. De ella depende
persona del singular); predicado: mientras (R+CCT) escribo
una subordinada adverbial de lugar b) con función de CCL:
(núcleo) la nota del supermercado (CD). Proposición subor-
donde me dijiste; esta, a su vez, incluye otra subordinada ad-
dinada adverbial de tiempo c) que está coordinada con b):
verbial c) en función de CCT: cuando nos vimos. Proposición
recuerdo. Sujeto elidido (1ª persona singular); predicado:
principal a): Ayer volví. Sujeto elidido (1ª persona del singu-
recuerdo (núcleo). De esta proposición depende la d), que
lar); predicado: volví (núcleo). Proposición subordinada ad-
es una subordinada sustantiva en función de CD del verbo
verbial de lugar b): donde me dijiste. Sujeto elidido (2ª
recuerdo. Proposición subordinada sustantiva d) dónde he
persona del singular); predicado: donde (R/CCL) me (CI) dijis-
puesto el abrigo. Sujeto elidido (1ª persona del singular);
te (núcleo). Proposición subordinada adverbial de tiempo c):
predicado: dónde (R+CCL) he puesto (núcleo) el abrigo
cuando nos vimos. Sujeto elidido (1ª persona del plural); pre-
(CD).
dicado: cuando (R/CCT) nos (CD) vimos (núcleo).
Vino corriendo desde donde estaba cuando oyó el ruido de la
Debemos salir del edificio por donde indican las señales y cuando
casa de al lado . Vino (V) corriendo (V; gerundio) desde
avise el guía.
(Prep) donde (Av) estaba (V) cuando (Av) oyó (V) el (Det)
Debemos salir (V; perífrasis verbal) del (Prep+Det) edificio (N) ruido (N) de (Prep) la (Det) casa (N) de (Prep) al
por (prep) donde (Av) indican (V) las (Det) señales (N) y (Prep+Det) lado. Oración compuesta, formada por tres pro-
(Conj) cuando (Av) avise (V) el (Det) guía (N). Oración com- posiciones a), b) y c). De la proposición principal depen-
puesta, formada por tres proposiciones a), b) y c). De la prin- den dos proposiciones subordinadas adverbiales.
cipal a) dependen las subordinadas adverbiales b) y c), que Proposición principal a): Vino corriendo. Sujeto elidido (3ª
están coordinadas entre sí mediante la conjunción y. Proposi- persona del singular); predicado: vino (núcleo) corriendo
ción principal a): Debemos salir del edificio. Sujeto elidido (1ª (CCM). Proposición subordinada adverbial de lugar b) que
persona del plural); predicado: debemos salir (núcleo) del funciona como (CCL) y depende de a): desde donde esta-
edificio (CCL). La proposición b) depende de a) como subor- ba. Sujeto elidido (3ª persona del singular); predicado:
dinada adverbial de lugar y funciona como CCL: por donde desde donde (R+CCL) estaba (núcleo). Proposición subor-
indican las señales. Sujeto: las señales; predicado: por donde dina adverbial de tiempo c) que funciona como (CCT) y
(R/CCL) indican (núcleo). Proposición subordinada adverbial depende de a): cuando oyó el ruido de la casa de al lado.
de tiempo c) que va coordinada a la b) por el enlace y, pero Sujeto elidido (3ª persona del singular); predicado: cuando
depende de a): cuando avise el guía. Sujeto: el guía;: predica- (R+CCT) oyó (núcleo) el ruido de la casa de al lado (CD) [el
do: cuando (R+CCT) avise (núcleo). ruido (núcleo) de la casa de al lado (CN de ruido) de al lado
(CN de casa)].
Las personas que quieran visitar el castillo deben dirigirse hacia
donde están las ruinas. Las (Det) personas (N) que (Pron) quie-
ran (V) visitar (infinitivo) el (Det) castillo (N) deben dirigirse (V; ACTIVIDADES (págs. 133-135)
perífrasis verbal) hacia (Prep) donde (Av) están (V) las (Det)
ruinas (N). Oración compuesta, formada por tres proposiciones
a), b) y c). De la proposición principal a) depende una adjetiva Estructuras oracionales bimembres  (pág. 133)
b) y una adverbial de lugar c)
Proposición principal a): Las personas deben dirigirse. Sujeto:  1. Completa estas oraciones con un nexo que indique finalidad…: Le
Las personas (núcleo); predicado: deben dirigirse (núcleo). di la toalla para que se secara. / Cogemos las vacaciones con
Proposición subordinada adjetiva que funciona como CN de las el fin de descansar. / Te dejo el secador para que te seques el
personas b): que quieran visitar el castillo. Sujeto: que (R+S); pelo. / Le enseñé el mapa con la intención de que no se per-
predicado: quieran (núcleo) visitar el castillo (CD) [visitar (nú- diera.
cleo; al ser también verbo, lleva su propio CD que es el casti-
llo)]. Proposición subordinada adverbial de lugar en función de  2. Justifica el uso de los modos… Si tuviera que cocinar, pasaría

CCL c): hacia donde están las señales. Sujeto: las señales (nú- mucha hambre: se construye en subjuntivo porque expresa
cleo); predicado: hacia donde (R+CCL) están (núcleo). una condición de cumplimiento imposible o improbable. /
Cuando las barbas de tu vecino veas pelar, pon las tuyas a
Mientras escribo la nota del supermercado y recuerdo dónde he remojar: se construye en subjuntivo porque es acción solo
dejado el abrigo , puedes ir por el periódico dando un paseo . posible. / Si lo hubieses entregado, te hubiese subido un pun-
Mientras (Av) escribo (V) la (Det) nota (N) del (Prep+Det) to: se construye en subjuntivo porque expresa una acción
supermercado (N) y (Conj) recuerdo (V) dónde (Av) he deja- anterior que no se dio. Va en pluscuamperfecto porque se re-
do (V) el (Det) abrigo (N) puedes ir (V; perífrasis verbal) por fiere al pasado. / En caso de que no esté, dáselo al portero: se
(prep) el (Det) periódico (N) dando (V; gerundio) un (Det) construye en subjuntivo cuando expresa una objeción o difi-
paseo (N). Oración compuesta, formada por cuatro proposi- cultad posible.
ciones: a), b), c) y d). La proposición principal a) va imple-

44
Unidad 6 >  La subordinación adverbial. Estructuras oracionales bimembres

 3. Completa las oraciones…: Aunque estudie todo, nunca podré Me compré un piso con el objeto de independizarme, luego me mudé:
llegar a aprobar la oposición. / No te bañes en el río aun cuan- con el objeto de independizarme (relación final) / luego me
do haya pasado la digestión. / No me lo creí, aunque parecía mudé (yuxtaposición).
sincero. / A pesar de que llegues antes, encontrarás mucha
Explícame dónde encontraste este disco que estaba descatalogado:
gente.
dónde encontraste este disco (sustantiva de CD) / que estaba
descatalogado (adjetiva; no es estructura oracional bimem-
 4. Ana salió tarde porque no sonó el despertador. / Estuvo aho- bre).
rrando para comprarse un coche. / No llegará a tiempo si no
se levanta ya. / Juan me confesó que este invierno había he-
 2 llegó un poco antes a fin de que yo pudiera salir, así que me fui.
cho más frío que el pasado invierno.
Llegó (V) un poco antes (Loc. av) a fin de que (Loc. conjunti-
 5. Señala los nexos…: Fuimos al zoo para que Ana viera a los va) yo (Pron) pudiera salir (V, perífrasis) así que (Loc. conjun-
animales. (finalidad) / Como no encontró los guantes, le com- tiva) me fui (V). Estructura oracional bimembre que consta de
pré otros. (causal) / A condición de que le acompañara, me una proposición principal a): Llegó un poco antes; una subor-
compró la entrada. (condicional) / A pesar de que han visitado dinada b) introducida por el nexo a fin de que (relación de
varias ciudades, ninguna les parece más bonita que la suya. finalidad) y otra subordinada c) con relación consecutiva, in-
(concesiva y comparativa de superioridad) / José está más troducida por el nexo así que.
ilusionado este año que el año pasado. (comparativa de supe-
Principal a): Llegó un poco antes. Sujeto: elidido (3ª persona
rioridad).
del singular); predicado: llegó (núcleo) un poco antes (CCT).
Subordinada con relación de final b): a fin de que yo pudiera
 6. Completa en tu cuaderno…: Aunque me levante temprano,
salir. Nexo subordinante : a fin de que. Sujeto: yo; predicado:
nunca llego a tiempo a la parada del autobús. / No cruces la
pudiera salir (núcleo). Proposición subordinada consecutiva
calle aun cuando no pase ningún coche. / No le ayudaba,
c): así que me fui. Nexo subordinante: así que. Sujeto: yo;
aunque me lo pedía de rodillas. / A pesar de que has invertido
predicado: me fui (núcleo).
muchas horas en este trabajo, no podrás llevarlo a cabo. / No
te preocupes. No lo hubieras conseguido, aun cuando lo hu- El profesor explicó el análisis tantas veces como le pedimos.
bieras intentado.
El (Det) profesor (N) explicó (V) el (Det) análisis (N) tantas
(Det) veces (N) como (Conj) le (Pron) pedimos (V). Estructura
 7. El problema no va a desaparecer porque hayamos firmado el oracional bimembre que consta de una proposición principal
contrato. (Expresa posibilidad o duda) / Aunque quiero que-
a) y una subordinada b) introducida por los nexos tantas…
darme más tiempo contigo, tendré que irme. (Expresa una
como. Entre ellas se establece una relación comparativa.
objeción real) / Si pintamos el salón esta mañana, podríamos
colocar los muebles por la tarde. (Expresa una condición posi- Principal a) : El profesor explicó el análisis tantas veces. Suje-
ble) / Aun cuando las previsiones anuncien tormentas durante to: El (D) profesor (núcleo); predicado: explicó (núcleo); CD:
todo el día, es muy probable que vayamos al monte a buscar el (D) análisis (núcleo); CCT: tantas (D) veces (núcleo). Subor-
setas (Expresa una objeción o dificultad). dinada b): como le pedimos. Sujeto elidido (1ª persona plu-
ral); nexo subordinantes: como; predicado: le (CI) pedimos
 8. Identifica las estructuras…: …pero eran tan exactamente (núcleo).
iguales que todos los tomaban por mellizos. (relación compa-
Los niños encontraron la barca que habían comprado sus padres
rativa de igualdad) / …como Pedro Luis se había atrasado un
con la finalidad de que pudieran navegar por la laguna.
año […] a partir de ese momento habían hecho juntos…(rela-
ción causal) / …y de ese modo solo estudiaban la mitad, ya Los (Det) niños (N) encontraron (V) la (Det) barca (N) que
que cada uno daba dos veces… (relación causal) / Su solida- (Pron) habían comprado (V) sus (Det) padres (N) con la fina-
ridad y colaboración fraternales llegaban a tales extremos que lidad de que (Loc. conjuntiva) pudieran navegar (V, perífrasis)
en más de una ocasión atendieron intermitentemente a algu- por (Prep.) la (Det) laguna (N). Estructura oracional bimem-
na noviecita. (relación consecutiva). bre que consta de una proposición principal a); esta a su vez
incluye una subordinada adjetiva b) y una subordinada que
depende de la adjetiva c) introducida por el nexo con la fina-
Análisis de la oración  (pág. 135) lidad de que. Entre ambas (b y c) se establece una relación de
finalidad.
 1. Te agradecería que vinieras, a menos que no te apetezca, pues se te
Principal a): Los niños encontraron la barca. Sujeto: Los (D)
puede hacer muy pesado:que vinieras (subordinada sustantiva
niños (núcleo); predicado: encontraron (núcleo) la (D) barca
de CD) / a menos que no te apetezca (relación condicional) /
(CD). Subordinada adjetiva o de relativo que funciona como
pues se te puede hacer muy pesado (relación causal).
CN de barca b): Sujeto: sus (D) padres (núcleo); predicado:
Si quieres salir de viaje, es mejor que lo hagas ahora, porque ha
que (R + CD) habían comprado (núcleo). Subordinada con
dejado de llover:Si quieres salir de viaje (relación condicional) relación de finalidad c): Nexo subordinante: con la finalidad
/ que lo hagas ahora (sujeto) / porque ha dejado de llover: de que; sujeto: elidido (3ª persona del plural); predicado: pu-
(relación causal). dieran pasear (núcleo) por la laguna (CCL).
Trajo esta estufa para que no pasara frío; con que la encenderé.
Tanto va el cántaro a la fuente que al final se rompe. Tanto (Av)
para que no pasara frío (relación final) / con que la encenderé
va (V) el (Det) cántaro (N) a (Prep.) la (Det) fuente (N) que
(relación consecutiva).
(Conj.) al final (Loc. adverbial) se rompe (V). Estructura ora-
Cuando acabes de comer, debes recoger tu habitación. Cuando cional bimembre que consta de una proposición principal a) y
acabes de comer (adverbial de tiempo; no es estructura ora- una subordinada b) unidas mediante los nexos tanto…que.
cional bimembre). Entre ambas se establece una relación consecutiva.

45
Unidad 6 >  La subordinación adverbial. Estructuras oracionales bimembres

Proposición principal a): Tanto va el cántaro a la fuente. Suje-  2 justifica…: Es un texto cuya tipología se incluye en la narrati-
to: el (D) cántaro (núcleo); predicado: tanto (CC de cantidad) va. / Personajes: Fermina, Florentino… / Tiempo: el día del
va (núcleo) a la fuente (CCL). Subordinada b): que al final se funeral. / Lugar: la iglesia y la casa de Fermina. / Acción: al
rompe. Nexo subordinante: que; sujeto: elidido (3ª persona encontrarse Fermina y Florentino surge un conflicto que se irá
del singular); predicado: al final (CCT) se rompe (núcleo). desarrollando a lo largo de la novela.
La mudanza fue más larga que (fueron) las obras, por tanto
tardamos más de tres meses en estar instalados. La (Det) mu-
 3 depurado: pulido / profanación: violación / apelación: reclama-
ción / contrariados: insatisfechos.
danza (N) fue (V) más (Av) larga (A) que (Conj.) [fueron (V)]
las (Det) obras (N) por tanto (Loc. conjuntiva) tardamos (V)
más (Av) de (Prep) tres (Det) meses (N) en (Prep) estar (in-
 4. Señala en cuál…: d) Florentino estaba tan enamorado de ella
finitivo) instalados (V participio). Estructura oracional bi- que fue capaz de esperarla cincuenta años.
membre que consta de una proposición principal a): La
mudanza fue más larga; una subordinada b), unida median-  5. Adverbiales de tiempo: …cuando vio a Florentino Ariza, vesti-

te los nexos más…que, cuya relación es de comparación: do…/…cuando aún estaba caliente en la tumba… /…cuando
que fueron las obras, y finalmente otra subordinada que es- entró en el dormitorio vacío… /…hasta mucho después que
tablece la relación de consecuencia mediante el nexo por acabaron de cantar los gallos /…y que mientras dormía… /…
tanto c): por tanto tardamos más de tres meses en estar después de que Fermina Daza lo rechazó…
instalados. Adverbial de modo: …como lo había hecho siempre…
Principal a): Sujeto: La (D) mudanza (núcleo); predicado: fue
(núcleo) más (CA) larga (Atr.) Subordinada b) Nexo: que; Su-  6. Analiza morfosintácticamente…: Fermina Daza (N) se habría

jeto: las (D) obras (núcleo); predicado: fueron (núcleo). Su- creído (V) frente a (Locución prepositiva) un (Det) loco (N) si
bordinada c) Nexo: por tanto; sujeto: elidido (1ª persona del (Conj.) no (Av) hubiera tenido (V) motivos (N) para (Prep)
plural); predicado: tardamos (núcleo) más de tres meses pensar (V, infinitivo) que (Conj) Florentino Ariza (N) estaba (V)
(CCT); en estar (núcleo) instalados (Atr) [CC]. en (Prep) aquel (Det) instante (N) inspirado (V, participio) por
(Prep) la (Det) gracia (N) del (Prep+Det) Espíritu Santo (N).
Comió tanto pastel en su fiesta de cumpleaños que se indigestó.
Comió (V) tanto (Det) pastel (N) en (Prep) su (Det) fiesta (N) Estructura oracional bimembre que consta de una proposi-
de (Prep) cumpleaños (N) que (Conj) se indigestó (V) ción principal a) Fermina Daza se habría creído frente a un
loco y una subordinada introducida por el nexo si b) si no hu-
Estructura oracional bimembre que consta de una proposi- biera tenido motivos para pensar. Entre ambas se establece
ción principal a): Comió tanto pastel en su fiesta de cum- una relación de condición. La subordinada incluye, a su vez,
pleaños y una subordinada introducida por las partículas una subordinada sustantiva en función de CD c) que Florenti-
correlativas tanto…que. Entre ambas se establece una re- no Ariza estaba en aquel instante inspirado por el Espíritu
lación consecutiva. a) Comió tanto pastel en su fiesta de Santo.
cumpleaños. Sujeto: elidido (3ª persona del singular); pre-
dicado: Comió (núcleo) CD: tanto (D) pastel (núcleo) CCT: Proposición principal a). Sujeto: Fermina Daza (núcleo); pre-
en su fiesta de cumpleaños; fiesta (núcleo) de cumpleaños dicado: se habría creído (núcleo) frente a un loco (CCL). Su-
(CN de fiesta). b) que se indigestó. Nexo: que. Sujeto: elidi- bordinada con relación condicional b). Nexo: si; sujeto: elidido
do (3ª persona del singular; predicado: se indigestó (nú- (3ª persona del singular); predicado: no (modificador) hubiera
tenido (núcleo); motivos para pensar (CD). El infinitivo pensar
cleo).
lleva una subordinada sustantiva en función de CD. Proposi-
ción c). Nexo: que; sujeto: Florentino Ariza (núcleo) predica-
do: estaba (núcleo) en aquel instante (CCT) inspirado (Atr.)
EVALUACIÓN (pág. 137)
por el Espíritu Santo (Complemento agente del participio ins-
pirado).
 1. Resume el texto…: Fermina, el día del funeral de su marido, se
encuentra con Florentino, del que había estado enamorada
 7. Explica la diferencia…: a) Si Fermina se reconcilia…: Expresa
hacía 51 años. Florentino le confiesa que siempre ha estado
una condición de cumplimiento posible. / b) Si Fermina se
enamorado de ella. Florentina, que en ese momento recuerda
reconciliara…: Expresa una condición de cumplimiento im-
algún hecho oscuro que cometió Florentino en el pasado, le
probable.
desprecia y le pide que se vaya. Sien embargo Fermina, al ser
consciente de que su marido ha muerto y que se encuentra
 8. Escribe tres palabras…: El fin de sus días, pasar a mejor vida,
en la soledad más infinita, comienza a pensar en Florentino.
dejar este mundo.
Título: Proponemos estos: Encuentro con un amor del pasa-
do. Amor y odio.  9. Desarrolla en tu cuaderno…: Respuesta Libre. Se ha de tener

46
7# El discurso

solicitar permiso para defenderse, implica tratar a los ameri-


En contexto (pág. 139)
canos como ciudadanos dependientes, incapaces de solucio-
nar sus propios problemas.
a) Las diez reglas son: 1. No sean multitareas. Estén presentes. 2.
No sean dogmáticos. 3. Usen preguntas abiertas. 4. Sigan el ¿A qué conclusión has llegado? Respuesta abierta, pero la idea
ritmo. 5. Si no saben, digan que no lo saben. 6. No equiparen es que se den cuenta de que el uso de estas palabras en
sus experiencias a las de ellos. 7. Procuren no ser repetitivos. 8. Bush no era gratuito y que lo que trataba de conseguir es que
Eviten los detalles. 9. Escuchen. 10. Sean breves. Todas ellas se los demás considerasen legítima la política de defensa instau-
resumen en la máxima: tengan interés en las personas. rada por su gobierno.
Respuesta abierta.
 5. Se refiere a las preguntas del ejercicio anterior. A aquellas que
b) Respuesta abierta debemos hacernos para desenmascarar las intenciones de
nuestros interlocutores cuando emplean metáforas o expre-
siones como “alivio fiscal” o “permiso escrito”. Pues, según la
practica con los textos (págs. 141-143) teoría del marco, estas expresiones no son neutrales, sino que
llevan implícitas una ideología.
Comprensión Respuesta abierta.

 1. ¿En qué consiste el ejercicio del elefante? En pedirles a los alum-  6. Respuesta abierta. Pero, es importante que cumplan los re-
nos que no piensen en un elefante. / ¿Con qué objetivo lo utiliza quisitos con que se pauta el ejercicio: una buena selección de
el autor en sus clases? Lakoff lo utiliza para demostrar a sus la información, en la que se eviten los ejemplos y las opinio-
alumnos que una palabra como elefante evoca un marco y que, nes personales.
por tanto, no podemos utilizarla sin imaginar todo aquello que
esa palabra evoca o lleva asociado. / ¿Qué consejo básico propone
George Lakoff si hemos de discutir un tema con alguien? No utilizar Cuestión de lengua
el lenguaje del adversario, pues sus palabras llevan asociada su
ideología. / Según el autor, ¿en qué trampa cayeron los senadores
 7. Puede ser (modal de posibilidad), Hay que discutir (modal de
obligación), ha tenido que haberle ocurrido (modal de obliga-
demócratas? Habían aceptado el marco de sus rivales, puesto
ción), viene a ser (modal de posibilidad), empezarán a utilizar
que también utilizaban la expresión “alivio fiscal”. Esta expre-
(aspectual incoativa), tiene que ver (modal de obligación), tu-
sión comporta un marco que da por sentada la ideología de los
viste que pedir (modal de obligación), tiene que pedir (modal
conservadores. / ¿Por qué es inútil negar un marco? Porque in-
de obligación), tiene que hacerse (modal de obligación).
cluso cuando lo niegas, lo estás evocando.
Predominan las perífrasis modales de obligación.
 2. Los tres ejemplos son: por un lado, el escándalo del Watergate
en que se vio involucrado Nixon. Cuando este dijo ante las  8. Estilo directo: «El ejercicio es: «No pienses en un elefante.
cámaras que no era un chorizo, lo que realmente consiguió es Hagas lo que hagas, no pienses en un elefante».» Estilo indi-
que todos pensaran que lo era. Por otro lado, está el caso del recto: El ejercicio consiste en pedirles que no piensen en un
presidente George Bush que se inventó la expresión “alivio elefante. / Estilo directo: «Se presentó ante los ciudadanos y
fiscal”, para presentarse como un héroe capaz de aliviar el dijo: «No soy un chorizo.» Estilo indirecto: Se presentó a los
yugo de los impuestos. El éxito de la expresión fue tal que ciudadanos y les dijo que no era un chorizo. / Estilo directo:
hasta los demócratas acabaron utilizándola y, por tanto, acep- Bush dijo: «No necesitamos presentar ningún permiso escrito
tando la ideología de sus rivales. Por último, otra anécdota para defender a América.» Estilo indirecto: Bush dijo que no
también protagonizada por Bush fue cuando afirmó que los necesitaban presentar ningún permiso escrito para defender
americanos no necesitaban solicitar un permiso escrito para América. / Estilo directo: Bastaba con que hubiera dicho: «No
actuar contra el terrorismo. pediremos permiso.» Estilo indirecto: Bastaba con que hubie-
Respuesta abierta. ra dicho que no pedirían permiso.

 3. Marco: todas las ideas, imágenes, prejuicios, conocimientos  9. Piensa en quién tiene que que pedir justificantes. Proposición
que una palabra lleva asociadas. / Evocar: es hacer presente, subordinada sustantiva de CRV. Nexo: quién: pronombre inte-
imaginar todas las ideas, connotaciones que tiene implícitas rrogativo que realiza función de relacionante y de sujeto. /
una palabra. / Enmarcado: efecto de enmarcar o asociar un Recuerda cuándo fue la última vez que tuviste que pedir algu-
marco a una palabra. no. Proposición subordinada sustantiva de CD. Nexo. Cuándo:
adverbio interrogativo que funciona como relacionante de la
 4. «Recuerda cuándo fue la última vez que tuviste que pedir al- subordinada y desempeña función de CCT en ella. / Piensa
guno»: Probablemente, en la escuela o en el instituto, cuando en a quién se le piden. Proposición subordinada sustantiva de
era menor y estaba bajo la tutela de un adulto (padre o tutor). CRV. Nexo: quién: pronombre interrogativo que, además de
/ «Piensa en quién tiene que pedir justificantes»: los que no relacionante, desempeña función de CI.
son responsables de sí mismos, por su incapacidad o minoría
de edad. / «Piensa en a quién se le piden.»: A la persona 10. Cuando negamos un marco, evocamos el marco. Proposición
responsable: padre, tutor, director… / «Piensa en la relación subordinada adverbial de tiempo. / Todo el mundo pensó que
entre una cosa y otra.»: Sugerir que los americanos han de lo era. Proposición subordinada sustantiva de CD. / Fue pre-

47
Unidad 7 >  El discurso

sionado, para que dimitiera. Estructura oracional bimembre zo)». «No será el marco que tú quieres». «Es verdaderamente
con relación final. / Pensemos en el marco que evoca la pala- asombroso». / Se utiliza léxico valorativo y connotativo: «conti-
bra alivio. Proposición subordinada adjetiva. / Si hay gentes nuamente», «héroe», «malvado facineroso», «desgracia»,
que intentan parar al héroe, se convierten en malvados. Pro- «verdaderamente», «asombroso», «muy interesante» / Se usa
posición subordinada condicional y dentro de ella subordina- el imperativo y perífrasis modales de obligación: «no utilices»,
da adjetiva de relativo. / Se hablará de ello en la CNN y la «hay que discutir», «tiene que hacerse», «pensemos», «re-
BBC, porque es el plan del presidente. Estructura oracional cuerda», «piensa».
bimebre con relación causal. / Este es el tipo de preguntas
que uno tiene que hacerse si quiere entender el discurso polí- 18. Repetición de palabras y estructura: «se distribuye, a través de
tico contemporáneo. Proposición subordinada adjetiva y Pro- notas de prensa, a todas las emisoras de radio, a todos los
posición subordinada condicional. canales de televisión, a todos los periódicos…», «Piensa en
quién tiene que pedir justificantes. Piensa en a quién se le
piden. Piensa en la relación entre una cosa y otra.» / Metáfo-
Cuestión de léxico ras: «héroe», «alivio fiscal», «circula», «tender una trampa» /
Personificaciones: «su lenguaje elige un marco…», «las pala-
11. asocia: relaciona, vincula, enlaza. / chorizo: ladrón, aprovecha- bras arrastraron a los demás».
do, mangante. / reapareció: apareció, se presentó, usó. / asom-
broso: sorprendente, admirable, increíble. / aliviar: calmar,
mitigar, moderar, aligerar / arrastraron: empujaron, atrajeron. Expresión
12. Alivio fiscal: rebaja de los impuestos fiscales. / Héroe: Bush y 19. Respuesta abierta.
su gobierno que promete bajar los impuestos. / malvado facine-
roso: los demócratas y sus políticas sociales que implican una
subida de los impuestos. La norma ortográfica (pág. 143)

13. Tirar piedras sobre el propio tejado: comportarse de manera  1. Las arcas comunitarias y el banco europeo de inversiones (bei) se
que perjudica al propio interesado. / tender una trampa: enga- encargarán de aportar el resto de los fondos: los paréntesis per-
ñar a alguien. / Dar lo mismo ocho que ochenta: no importarle a miten intercalar un dato, aquí unas siglas. / Se aplicarán medidas
uno nada. / Poner las peras a cuarto: mostrarse duro con una de apoyo a los sectores más afectados por la crisis (como el automó-

persona y obligarle a hacer algo que no quiere. / Meterse en vil y la construcción): para intercalar una aclaración. / Los(as)

camisa de once varas: complicarse la vida, meterse en asuntos alumnos(as) contarán con la presencia de un(a) profesor(a) invitado

que no le competen a uno y que pueden ocasionarle proble- en cada semestre que ampliarán su formación literaria: para indicar

mas y preocupaciones. / Cantar las cuarenta: reñir fuertemen- varias opciones en un texto. / Las preguntas no son nunca indis-
te a alguien con razón y dejarle sin argumentos. / Quedarse a cretas. Las respuestas, a veces sí. (Oscar wilde, 1854-1900): para

dos velas: quedarse sin dinero. introducir una fecha, un autor, etc. / Este verano pasamos unas
relajadas (?) Vacaciones en compañía de la familia: para añadir una
14. Respuesta abierta. Se sugieren algunas posibilidades. duda. / Las exequias se celebraron en florencia en el 878
Elefante: se asocia al zoo, al circo. Por tanto, a algo divertido, a(nno) d(omini): para reconstruir abreviaturas.
superficial, carente de preocupaciones. / Alivio fiscal: está re-
 2. Alfonso Sastre está realmente encantado de que el Centro
lacionado con las políticas liberales de reducción de impues-
Dramático Nacional (CDN) estrene hoy La taberna fantástica,
tos. Lleva asociada una connotación positiva. / Zorro: se
una obra ambientada en el mundo marginal de un barrio de
asocia a cacería, a astucia, a engaño… / Verde: naturaleza,
chabolas del Madrid de los sesenta, y que hace 23 años que
ecología, sano, natural, auténtico… / Guerra: enfrentamiento,
no se representaba. (...)
dolor, muerte…
«Espero —comenta uno de los autores más importantes y
prolíficos del último medio siglo, con unas 80 piezas teatrales
Análisis del texto escritas— que La taberna fantástica haya resistido bien el
paso del tiempo. Es cierto que está muy centrada en un mo-
15. El texto puede dividirse en tres partes: mento histórico, con unos personajes marginales muy concre-
Primera parte: Introducción. Se presenta la tesis del texto: que tos, como los quinquilleros [gente que se dedicaba a la venta
todas las palabras evocan un marco y que, por tanto, llevan de quincalla y al arreglo de cacharros de cocina], pero al
asociadas una ideología que no puede obviarse si se utilizan. mismo tiempo creo que conserva virtudes dramáticas al tra-
Coincide con el primer párrafo. tarse de una tragedia compleja» (...).
Segunda parte: Desarrollo o cuerpo argumentativo. Se presen- Alfonso Sastre (Madrid, 1926) formó parte de una generación
tan los argumentos y ejemplos que sostienen la tesis. Va del de dramaturgos españoles realistas y muy críticos que sufrie-
párrafo dos al nueve. ron los rigores de la censura franquista, y cuyo teatro fue poco
Tercera parte: Conclusión. El texto termina con una conclusión y mal representado hasta la llegada de la democracia. (…)
que se desprende o deriva de la tesis demostrada: para enten-
der el dircurso político, hay que analizar las palabras de acuerdo  3. La condesa desmintió: «La sucesión monárquica no se pon-
a la teoría del marco. Coincide con el último párrafo del texto. drá en cuestión por la actitud de un adolescente». / No se lo
tengas en cuenta. Los que le conocemos sabemos de sus
16. Persuadir. «boutades». / Mi enorme impresión al ver por primera vez
«La muerte de Marat» se debió a la maestría de la obra. /
17. Se usa la primera persona en verbos y pronombres: «enseño», «lo Primero dijo «voy»; después, «llamaré a confirmar», y al fi-
primero que hago», «he encontrado», «pondré»…/ Hay afir- nal ni una sola palabra. / En el capítulo «La percepción mu-
maciones tajantes:
«Todo el mundo pensó que lo era (un chori- tua entre las culturas» encontré la mayoría de las claves

48
Unidad 7 >  El discurso

para la elaboración del trabajo. sentido. Se manifiesta en una buena selección y organización
Las comillas se usan para reproducir citas, diálogos y pala- de la información; así como en la ausencia de contradiccio-
bras textuales. Para señalar extranjerismos o palabras y ex- nes. / cohesión: propiedad textual que consiste en que las
presiones que no son propias del castellano. Para diferenciar partes de un texto estén relacionadas y conectadas, gracias a
el título de películas de cine, obras literarias y de arte… los mecanismos lingüísticos. / destinatario: persona o personas
a las que va dirigido el texto y que condiciona el registro o ni-
 4. En la lectura las comillas se usan para reproducir el diálogo en vel de lengua empleado, entre otros factores. / registro: nivel
estilo directo; es decir, para señalar las citas textuales. de lengua o variedad diafásica que puede ser formal o infor-
mal y que depende de factores como el tema, el destinatario,
 5. En la play station figuran equipos de toda Europa, con todos el contexto…/ anáfora: mecanismo de cohesión referencial
los jugadores posibles. Cada uno debe elegir un equipo de que consiste en sustituir información previa, mediante ele-
fútbol y competir con él. mentos gramaticales como determinantes, pronombres, ad-
verbios… / catáfora: mecanismo de cohesión referencial que
Siempre elige el Chelsea. Dice que es el que más le gusta,
consiste en anticipar información. / deíxis: conexión del texto
que él es del Chelsea. A mí eso me da pena. Que no sea
con elementos extralingüísticos como el espacio, el tiempo, la
seguidor de un equipo vasco. «Yo a tu edad era del Athle-
persona…/ elipsis: omisión de elementos textuales que se so-
tic», le suelo decir, haciéndole chantaje emocional. «Pero si
breentienden. / conector: expresiones de distinta categoría
el Athletic siempre pierde», se queja él, «yo prefiero ser del
gramatical que no realizan función sintáctica en la oración,
Chelsea y ganar la Champions League». Salgo cabizbajo de
sino que conectan con otras partes del texto, poniendo de
su habitación.
manifiesto las relaciones lógicas que mantienen.
Hace poco entré en el cuarto de Unai y lo encontré jugando
a la play. «Tengo una buena noticia para ti», me anunció  2. Indica qué recursos lingüísticos utiliza el autor del texto para
con una sonrisa. «Estoy jugando con el Athletic y estamos a conseguir la cohesión: para relacionar las distintas ideas que se
punto de ganar la Champions League.» Yo no cabía en mí de exponen, el autor recurre a una serie de procedimientos lin-
alegría. Al final el chaval ha elegido el camino correcto, pen- güísticos, como el empleo de conectores (preposiciones,
sé con orgullo. Pero de repente me di cuenta de que un ju- conjunciones, locuciones, etc.), que relacionan el significado
gador del Athletic era negro. «Oye, ¿quién es ese?», le de las distintas partes de la oración o de las oraciones entre sí;
pregunté, «no lo conozco». «Ese es Drogba, delantero del por ejemplo: en total, a pesar de, a ello se suma, sin duda,
Chelsea. Lo he fichado para el Athletic», me contestó tan mientras, entre ellos...
campante. «Y también a Torres y a Messi. Ahora el Athletic
Teniendo en cuenta el medio en el que se publica, explica si el texto
es el mejor equipo del mundo.»
es adecuado: El texto se ha publicado en un periódico y, por

 6. —¿Cuántos años tiene Tomás?— pregunté esperando cierta tanto, va dirigido a un público general. En este sentido, utiliza
consternación por su parte o al menos un titubeo. un lenguaje adecuado, con un vocabulario accesible y una
sintaxis poco compleja, de manera que pueda ser comprendi-
—Catorce— dijo Josefina con resolución.— Los cumple hoy do con claridad por el lector al que se dirige.
precisamente.
Indica el tema del texto y analiza las ideas que se desarrollan, para
—Sí— añadió José—, íbamos a celebrar una pequeña fiesta justificar si es coherente: El texto es coherente, pues las ideas
familiar pero ya sabes, ya sabes… que se exponen están relacionadas con el tema “Los proble-
—El corazón, el oído, el hígado— dije yo. mas en la universidad”. La primera idea presentada por el
autor, el descenso de alumnos en la universidad, se relaciona
—Lo hemos tenido que acostar en su cuarto.
lógicamente con la segunda: la universidad está amenazada
por serios problemas. La tercera es una ejemplificación de la
 7. In-flar / suc-ción / abs-trac-to / des-hi-dra-tar / le-chu-ga / cons-
segunda: la escasa representación femenina en los órganos
tan-te / u-nes-co / subs-tra-to / ins-tan-tá-ne-a / abs-te-ner-se / i-
de gobierno universitarios. De la idea fundamental, la escasa
nú-til / a-cre-e-dor / ist-mo / sols-ti-cio.
inversión educativa, se deduce la conclusión: la necesidad de
 8. Antiinflamatorio / hispano-argentino / científico-técnico / invertir más en la enseñanza universitaria.
asturleonés / francoalemán / vicecónsul / hispanoamericano / Señala en cuántas partes puede dividirse su estructura: Se pue-
kilómetro hora / psicoanálisis / anti-OTAN. de dividir en tres partes, que corresponden a cada uno de
Se debe recordar que en las palabras compuestas el guion es los párrafos. El primero es una presentación del problema: el
un paso previo a la lexicalización; cuando se extiende su uso, descenso de alumnos en la Universidad española. El segun-
tiende a desaparecer (asturleonés, hispanoamericano, fran- do desarrolla el tema, presentando otros síntomas de la mala
coalemán). salud de las universidades españolas. En el último párrafo,
para concluir, se propone una solución: el aumento de la
inversión educativa.
ACTIVIDADES (págs. 159-161) Indica en qué párrafos podemos encontrar secuencias textuales
expositivas y secuencias argumentales:
En el primer párrafo se
recurre a secuencias expositivas para presentar datos objeti-
El texto y sus propiedades  (pág. 159) vos: 1,2 % de alumnos, 1.389.394 personas... Este tipo de
secuencias aparecen también en el segundo párrafo, donde
 1. Adecuación: propiedad textual que consiste en el ajuste del
continúa la exposición de datos; pero en este caso las se-
texto y su lenguaje a la situación (destinatario, tema, inten-
cuencias expositivas se mezclan con las argumentativas: en
ción, contexto y canal). / coherencia: propiedad que han de
estas se emplean adjetivos calificativos (preocupantes, exi-
cumplir los textos y que consiste en que tengan unidad de

49
Unidad 7 >  El discurso

guos) o expresiones (sin duda) que implican la valoración Respuesta abierta.


subjetiva propia de la argumentación. En el último párrafo,
 7. Respuesta abierta, pero es importante que cumplan con las
predominan las secuencias argumentativas, en las que el
pautas que se les han dado, que coinciden con los rasgos del
autor intenta convencer con datos y razonamientos de la
texto argumentativo: tesis y cuerpo argumentativo, uso de la
necesidad de aumentar el gasto en la educación universita-
primera persona, expresiones valorativas (en mi opinión…).
ria para garantizar su futuro.
 8. Respuesta abierta, pero es importante que antes de la realiza-
Atendiendo a sus rasgos, di en qué tipología textual podría en-
ción del debate se elijan dos grupos (que defenderán postu-
cuadrarse y argumenta a qué subgénero pertenece: Dado que
ras contrarias), un público experto en el tema, un secretario y
trata un tema de actualidad y utiliza un vocabulario riguroso
un moderador. Cada uno de estos grupos debe preparar su
pero accesible, se puede decir que estamos ante un texto
intervención en el debate, antes de llevarlo a cabo.
periodístico. Aunque se exponen datos objetivos, la presen-
cia de secuencias argumentales sugiere que es un texto de  9. Respuesta abierta, pero es importante que aparezcan las si-
opinión. La ausencia de firma podría hacernos pensar que guientes ideas: modalidad estándar del lenguaje por ir desti-
se trata de un editorial. nado a un público amplio; abundancia de neologismos, siglas
y extranjerismos por su anclaje a la actualidad; combinación
de códigos de distinto tipo (verbal y no verbal), concisión y
Variedad del discurso  (págs. 147-149) precisión en la expresión.

 3. • Los textos orales se caracterizan por estos factores contex- 10. Respuesta abierta.
tuales presencia de los interlocutores, inmediatez, interac- 11. No compre sin ton ni son, compre thompson: aliteración, anáfora,
ción, espontaneidad y fugacidad que determinan estos rima, juego fónico. / Ya es primavera... En el corte inglés: metá-
rasgos lingüísticos: uso de recursos no verbales, abundan- fora, hipérbole. / Vamos al futuro, ¿subes? (Renfe): metáfora,
cia de deícticos, presencia de estructuras gramaticales más interrogación retórica. / Nada más efectivo (tarjeta euro 6000):
sencillas y empleo de un registro informal. silepsis (usa una palabra con sentido literal y figurado a la
vez). / Rexona no te abandona: rima. / Autoemoción (seat): metá-
• En los textos escritos, se usa un registro formal, caracteriza-
fora, neologismo. / Fujitsu. El silencio: metáfora, hipérbole.
do por la planificación y corrección del lenguaje, que se
concreta en un léxico culto, la corrección gramatical y la Explica las características del lenguaje publicitario a partir de los
complejidad sintáctica. eslóganes anteriores: El lenguaje publicitario se caracteriza por
la concisión, la ponderación y la originalidad. Estos tres rasgos
• Algunos textos digitales poseen características de los textos
se observan en los eslóganes de la actividad anterior. La conci-
orales, como la posibilidad de interacción entre emisor y
sión se comprueba en la brevedad de los mensajes, que con-
receptor, la espontaneidad.
centran en pocas palabras gran cantidad de información. La
ponderación es otro rasgo común. Los eslóganes resaltan las
 4. informe policial: texto escrito, texto objetivo y expositivo. / en-
cualidades de un producto: de Thompson se destaca la sensa-
trevista a un famoso: texto oral, subjetivo, argumentativo, dialo-
tez; de El Corte Inglés, la alegría; de RENFE, la modernidad; de
gado y periodístico. / cuento popular: puede ser oral o escrito,
la Tarjeta Euro 6000, la utilidad; de Rexona, la eficacia; de Seat,
narrativo y literario. / anuncio televisivo: texto digital, subjetivo,
la emoción; y de Fujitsu, el silencio. El tercer rasgo, la originali-
argumentativo y publicitario. / libro de texto de física: texto es-
dad, se aprecia en la abundancia de recursos retóricos, como
crito, objetivo, expositivo, científico. / manual de instrucciones de
se ha visto en la actividad anterior. Todos tienen como objeto
la play: texto escrito e instructivo.
que los mensajes sean atractivos para el receptor, y que perdu-
ren en su memoria para aumentar su eficacia.
Textos objetivos y subjetivos  (págs. 150-157) 12. Respuesta abierta, pero debe ajustarse a los requisitos de la
tipología; especialmente, la estructura y el uso del lenguaje
 5. Uso de la primera persona gramatical: somos, entramos, sospe-
formal, siguiendo un modelo como este:
cho, nos interesa… / Presencia de oraciones de modalidad interro-
gativa, exclamativa …: ¿Somos los únicos seres capaces de MODELO DE INSTANCIA GENERAL
reírnos? ¡desde luego que no! / Uso de figuras retóricas: entra-
Apellidos:
mos en un terreno resbaladizo (metáfora), …: ¿Somos los úni-
cos seres capaces de reírnos? (interrogación retórica). / Uso de Nombre:
expresiones valorativas: ¡Desde luego que no! (afirmaciones tajan-
Domicilio:
tes), sospecho, pensar, nos interesa (verbos de opinión)…
Ahora, reescríbelo…: Respuesta abierta. C.P.: Localidad: Provincia:
Teléfono: D.N.I.:
 6. uso de la 3.ª persona gramatical:
hay, son, está… / Verbos en
presente de indicativo: hay, son, está, contabilizan… / Predomi-
EXPONE:
nio de la modalidad enunciativa: Son nueve las embajadoras

acreditadas ante gobiernos de estado. / Oraciones impersona- SOLICITA:
les: En las universidades españolas hay ocho rectoras frente a
75 rectores. / Pasivas reflejas: Se evidencia que la presencia Nombre del lugar, de del 20  
de mujeres en los mandos de grandes organizaciones en Es- (Firma)
paña todavía está lejos de la paridad. / Léxico denotativo: uni-
versidades, mujeres, varones, Tribunal Constitucional… ILMO/A SR-/A:

50
Unidad 7 >  El discurso

13. Explica el tema y la postura…: en este texto, el autor trata de  4. Tema: el texto advierte contra los comportamientos que mini-
defender su postura ante la llegada masiva de extranjerismos mizan los riesgos de la violencia machista, como el anuncio
(sobre todo, anglicismos) en el terreno de la ciencia y la tec- de El Corte Inglés que difunde una idea tóxica del amor o el
nología. Este aluvión de préstamos y neologismos demuestra uso de expresiones como micromachismo o guerra de sexos.
que el español no participa de la innovación mundial en estos
ámbitos. / Indica de qué tipo de texto se trata: Es un texto subje-  5. El texto es adecuado porque utiliza un registro estándar, con
tivo (escrito desde la perspectiva personal del autor), argu- ciertas dosis de informalidad, evidentes en el uso de vulgaris-
mentativo (trata de justificar razonadamente su postura y mos como “ganas de montar un pollo”, “mándalo a la mierda”,
convencernos) y periodístico (se ha publicado en prensa El adecuado para un público amplio (es un texto publicado en
Periódico). Dentro de esta última tipología, se clasificaría en el prensa), pero especialmente dirigido a los más jóvenes, con los
artículo de opinión. / ¿Por qué al autor le parece que el español que trata de empatizar usando un lenguaje cercano a ellos.
no es un idioma apto..?: Porque sus hablantes han dejado de
estar en primera línea y viven de lo que otros descubren e in-  6. Mecanismos léxicos: repetición de palabras relacionadas con el
ventan. / ¿Estás de acuerdo?: Respuesta abierta. tema: anuncio, controlar, machismo…: uso de sinónimos:
anuncio-spot, pelea-guerra-enfrentamiento…; sinónimos
14. Lee estos textos descriptivos y explica qué diferencias existen en- contextuales: El Corte Inglés-grandes almacenes, Tinder-apli-
tre ellos:La diferencia entre ambas descripciones radica en cación de contactos; palabras del campo semántico de las
el punto de vista y la intención estética. En el texto A se trata nuevas tecnologías: aplicación, chat, Instagram, Tinder, dar
de una descripción que responde a la subjetividad del autor likes, colgar fotos…
(carecía de cierta gracia), que ofrece su particular punto de Mecanismos gramaticales: anáforas: formaba parte de su cam-
vista sobre un personaje, Benina, con adjetivos claramente paña de San valentín…; alertando sobre lo que no lo es. ; ella
connotativos (dulce, interesante). Lo hace con clara inten- le va detrás…; algo que parte de un respeto mutuo…. / Catá-
cionalidad estética, utilizando recursos retóricos (por ejem- foras: y el argumento era este: no hace mucho que salen
plo hipérbatos) que embellecen la expresión. En cambio, el juntos… / deíxis: Él insiste; Nos conocimos; Yo intentaré; esos
texto B describe objetivamente el planeta Tierra, sin inten- grandes almacenes ; así las advertencias; quererte… / Elip-
ción artística. El lenguaje empleado requiere una especiali- sis: él sí borró la aplicación, [él] no acepta chats privados con
zación (núcleo, corteza, nitrógeno) en la materia que se otras [chicas]; [Nosotros] estamos en el minuto 1’40. Pero no
trata. También los adjetivos son claramente denotativos, [es así], [Tú] ten la suerte de quererte.
porque se ajustan a una descripción objetiva de la realidad
(metálico, densa). Mecanismos de conexión: conectores: Mientras, pero, tampoco.

Explica en qué tipología textual podrían clasificarse cada uno  7. El texto tiene una estructura inductiva, ya que va de lo concre-
de los ejemplos anteriores:
Por todo lo dicho en la pregunta to a lo general. El texto empieza explicando la campaña publi-
anterior, el texto A es una descripción literaria, concreta- citaria que El Corte Inglés ha tenido que retirar (párrafos 1 y
mente un retrato, pues describe tanto aspectos físicos 2). En el párrafo 3, se critica el anuncio, puesto que fomenta
como psicológicos. Por el contrario, el texto B es una des- una idea del amor que origina muchas de las actitudes ma-
cripción científica. chistas y en el último y cuarto párrafo, la autora hace una ge-
neralización criticando la minimización que se hace de ciertas
actitudes que fomentan la violencia de género.
Evaluación (pág. 163)
 8. Para reproducir citas y palabras textuales: «Nos concimos
dándonos likes, ¿quién me dice que no haces lo mismo con
 1. ¿Qué se anuncia en la campaña publicitaria de El Corte Inglés? El ellos?». «Intentaré controlarme con los likes. »
anuncio forma parte de la campaña de San Valentín de El
Corte Inglés. Por tanto, se promocionan todo tipo de objetos y
 9. Es un texto subjetivo, en el que la autora defiende su perso-
regalos para ese día. / ¿Por qué han tenido que retirarla?: Por-
nal punto de vista. Uso de la 1.ª persona: Estamos en el minu-
que el anuncio presenta como normal una relación de pareja
to 1’40; Yo intentaré; Quiero creer… / Presencia de oraciones
tóxica, en la que el chico controla, presiona y chantajea a su
imperativas: mándalo a la mierda. Ten la suerte de quererte. /
novia por celos. Así que el anuncio fomenta implícitamente el
uso especial del lenguaje con figuras retóricas (“una mente
machismo y la violencia de género. / ¿La autora está de acuer-
educada”) y expresiones chocantes para llamar la atención
do?: Sí, porque considera que en el anuncio se presenta una
(“montar un pollo”, “mándalo a la mierda”) / Verbos de opi-
relación insana de dominación y manipulación y no una rela-
nión: creer, abordar, sugerir… / Expresiones valorativas: es in-
ción amorosa.
creíble, no ayuda, / léxico connotativo. Ingenua, tóxicas,
sumiso, machista, exactamente, peligros, manipulación,
 2. tóxicas: dañinas, enfermizas, nocivas, perjudiciales. / adverten- mereces, suerte…
cias: avisos, consejos, alertas. / germinal: principal, original,
inicial. / término: palabra, concepto, expresión. 10. Es un artículo de opinión, puesto que ha sido publicado en
prensa, La Vanguardia, y cumple los rasgos de los géneros de
 3. Con esta frase la autora trata de remarcar la importancia opinión.
que tiene la confianza en uno mismo, puesto que solo las
personas que realmente se quieren y están seguras de 11. Respuesta abierta. Teoría en la página 149 del Libro del
ellas mismas, no corren el peligro de ser víctimas de mani- alumno.
pulaciones y chantajes emocionales como los del anuncio
y, por tanto, es menos probable que sean víctimas de la 12. Respuesta abierta. Teoría en la página pág. 155 del Libro del
violencia de género. alumno.

51
8# El español actual

 5. Los niños deben experimentar, aprender de sus errores. No


En contexto (pág. 165)
hay que protegerles en exceso.
a) Respuesta abierta. Proponemos la siguiente: Conocer y ha-
blar cualquier idioma siempre es una ventaja. El español, en  6. Respuesta abierta. Proponemos las siguientes expresiones: La
concreto, es una lengua que hablan más de 500 millones de aventura era como para asustar a cualquier niño de su edad,
personas, es decir, permite comunicarse en muchos países pero Julius, llevado por el ansia de encontrar al pintor Peter del
del mundo. Además, se trata del tercer idioma más utilizado mercado, olvidó el miedo y no se sintió perdido en ningún mo-
en Internet y está considerada como una de las lenguas del mento. / Estuvieron largo rato conversando frente a las botellas.
futuro. Actualmente aprender y emplear el español abre mu- Julius respondía con precisión a todas las preguntas, le contó
chas puertas, tanto a la hora de conseguir oportunidades la- enterita la historia de su familia. / La idea de cruzar el río por el
borales, como a la hora de viajar. Estas ventajas están puente que tiembla lo fascinó. / Julius como que captó el asun-
directamente relacionadas con el interés, cada vez mayor, to y empezó a escuchar con atención...
que existe por aprender nuestro idioma.
 7. Se había convertido en toda una institución en el mercado,
b) Respuesta abierta. El alumno debe comprender y explicar pues conversaba con todo el mundo, explicando historias de
que los hablantes empleamos la lengua dependiendo de fac- su país, de sus viajes...
tores como la situación en la que nos encontramos o de la
finalidad del mensaje que queremos transmitir. El registro
lingüístico que utilizamos al hablar puede ser formal (redac- Cuestión de lengua
tar la respuesta de un examen) o coloquial (chatear con los
amigos por Internet).  8. Dijera: pretérito imperfecto de subjuntivo. / Había convertido:
pretérito pluscuamperfecto de indicativo. / Insistía: pretérito
imperfecto de indicativo. / Fue: pretérito perfecto simple de
Practica con los textos (págs. 167-168) indicativo. / Cogió: pretérito perfecto simple de indicativo. /
Estuvieron: pretérito perfecto simple de indicativo. / Regresara:
pretérito imperfecto de subjuntivo.
Comprensión
 9. Tenía guardada la pintura: aspectual resultativa. / Iba a llevar el
 1. ¿Cómo se llama la avenida principal de Chosica Baja? Avenida 28 cuadro a su casa: aspectual ingresiva. / Le preguntó si quería
de Julio. / ¿Por qué Nilda y Vilma no pudieron acompañar a Ju- venir al restaurante: No hay ninguna perífrasis.
lius? Estaban ocupadas en casa y no pudieron acompañar a
Julius. / ¿En qué lugar tiene guardado Peter el cuadro que ha 10. Lo (Julius) llamaba con una mano y con la otra (mano) le (Julius)
pintado para Julius? En el quiosco. / ¿Por qué a Peter le costa- señalaba algo (cuadro) allá al fondo./ No faltaba quien (gente)
ba comunicarse con los nacionales? Es extranjero y no sabe lo tratase burlonamente. / Unos 10 minutos después se despidió de
hablar castellano. / ¿Qué le pasa a Julius cuando oye hablar de todos (gente). / Julius le (Peter) dijo que se iba a llevar el cuadro
su hermana Cinthia? Se pone nervioso. No podía recoger el a su casa y que lo (cuadro) iba a colgar en su dormitorio.
vaso de la mesa, palidecía. / ¿Cómo es el hotel de la estación?
Era un hotel muy antiguo, todo de madera, en el que casi 11. Su pipa se desplazó ligeramente hacia la derecha cuando sonrió al
nadie se alojaba, aunque en sus buenos tiempos había lle- ver a Julius: Oración compuesta por subordinación adverbial
gado a hospedar a presidentes y ministros. de tiempo. / Le dijo que ya tenía el cuadro: Oración compuesta
por subordinación sustantiva. / Peter aprendía mucho, pero su
 2. No es habitual que un niño de su edad vaya solo por una gran acento era francamente malo: Oración compuesta por coordina-
ciudad. ción adversativa.

 3. Julius: Es un niño curioso, valiente e intuitivo. / Peter: Es 12. Julius olvidó el miedo y no se sintió perdido en ningún momento:
muy sociable, conversa con todo el mundo en el mercado, Oración compuesta por coordinación copulativa, formada por
a pesar de que apenas sabe hablar en castellano. Tartamu- dos proposiciones (Julius olvidó el miedo y [Julius] no se sin-
dea al hablar. tió perdido en ningún momento) unidas mediante el enlace y.
En la primera proposición el verbo aparece complementado
¿Qué relación crees que tienen?Son amigos. / ¿Qué le atrae al por un CD (el miedo) y en la segunda aparece un Pvo (perdi-
uno del otro? A Julius le llama la atención Peter por el hecho do) y, también, un CCT (en ningún momento).
de ser extranjero y por el tipo de vida que lleva. A Peter, Julius
le hace recordar su propia niñez. ¿De dónde son? Peter es un Se fue hasta el quiosco donde tenía guardada la pintura: Oración
pintor estadounidense y Julius es de Lima, pero procede de compuesta por subordinación adjetiva. La proposición princi-
una familia de ascendencia británica. pal (Se fue hasta el quiosco) está complementada por una
proposición subordinada adjetiva (donde tenía guardada la
 4. Vilma es la niñera de Julius, y Nilda también trabaja en la pintura) que realiza la función de CN y que está introducida
casa donde vive Julius. También se hace referencia a la fa- por el adverbio relativo donde, que realiza al mismo tiempo la
milia del niño (a su padre, a su madre, a su hermana función de nexo y de CCL. En ambas proposiciones el sujeto
Cinthia, a su abuelo...). es elíptico (Peter).

52
Unidad 8 >  El español actual

Cuestión de léxico 21. Descripción del entorno: Seguir siempre de frente hasta encon-
trar una de las escaleras que bajan a la avenida 28 de Julio, la
13. Respuesta abierta. Estas son algunas palabras y expresiones: principal de Chosica Baja, ancha, llena de tiendas, bazares y
cuadras, placera, míster, estuvo largo rato, canastota, gringo... bodegas. En una de las últimas bocacalles estaba el mercado,
al fondo, cerca del río, no era difícil ubicarlo. / Al otro lado del
14. Placera: tendera, vendedora. / Ubicarlo: localizarlo, situarlo. / puente, le señaló el Hotel de la Estación. Se estaba viniendo
Ansia: afán, deseo. / Listo: acabado, preparado. / Atareado: abajo de viejo, pero tenía historia y encanto. [...] Era un hotel
ocupado, dedicado. / Excitaba: agitaba, inquietaba. / Atinada: muy antiguo, todo de madera.
acertada, certera. / Conversando: charlando, platicando. / Ce-
Descripción de los personajes: Ahí estaba Peter, con su paleta y
sando: dejando, terminando. / Espantado: asustado, alarmado.
todo el instrumental en una bolsa. Conversaba con una place-
ra, rodeado de curiosos... / El míster se había convertido en
15. Diminutivos: Le dijera saluda niñito; Ahí estaban él y Vilma
una especie de institución en el mercado, siempre pintando,
igualitos, Allá iba solito... / Explica las connotaciones que tienen:
siempre conversando, siempre contando de su país, de sus
El empleo de diminutivos tiene una connotación emocional
viajes, siempre con la pipa en la boca, tartamudeando ade-
que muestra el afecto del autor hacia ese personaje.
más. Mucho trabajo le costaba comunicarse con los naciona-
les, pero insistía. / El gringo andaba emocionado y todo. En el
16. Tiendas, bazares, comercios, negocios, boticas... / Quioscos,
fondo era un solitario.
puestos, tenderetes, casetas... / Petate, bolsa, saco, maleta,
valija... / Calle, cuadra, manzana, avenida...
22. En el primer párrafo se narra la llegada de Julius a Chosica
17. Se refiere a que el modo que empleaba Peter para aprender Baja y cómo encuentra al pintor. En los párrafos siguientes se
castellano era charlando con la gente de la calle. explica cómo Peter muestra al protagonista el cuadro y, des-
pués, le propone ir a tomar unas gaseosas. Hablan sobre su
18. Era solo cuestión de tomar la primera calle a la izquierda: Se- familia y pasean hasta el puente colgante. Por último, en el
guir un camino o dirección. / Preguntó si quería venir al res- último párrafo, el autor explica cómo las vivencias del niño
taurante sobre el río a tomar una gaseosa: Beber. / El pintor hacen al pintor recordar su propia infancia.
Peter del mercado lo introdujo al grupo: Meter o incorporar a
alguien en un ambiente o grupo. / El gringo andaba emociona-
do y todo: Estar o encontrarse en un determinado estado o Expresión
situación de emoción.
23. Respuesta abierta.
19. Tartamudísimo: Peter tartamudeaba mucho. / Burlonamente: De
forma burlona. / Canastota: Canasta de grandes dimensiones. 24. ¿Qué marca la diferencia entre su literatura y la de los autores
/ Enterita: El sufijo aporta una connotación de afecto. anteriores? Bryce Echenique se considera un observador de
Añade otros sufijos que cambien el sentido: tartamudito, burlan- lo que representaban los autores del boom de la literatura
do, canastilla, enterísima. hispanoamericana, pues aprendió de todos ellos (García
Márquez, Cortázar, Vargas Llosa...). Esos grandes autores
trataron temas colectivos de la sociedad y, en cambio, en la
Análisis del texto literatura posterior, en la que él se incluye, triunfa lo indivi-
dual sobre lo colectivo. / ¿Qué es lo que ha triunfado de su lite-
20. Narrador. Aparece un narrador omnisciente en tercera perso- ratura? La visión de lo que ha sido su vida, el triunfo de los

na. Lo conoce todo sobre la trama y sobre los personajes (ac- sentimientos, de las lealtades, de las amistades y de los
ciones, sentimientos más íntimos...). afectos, de la ternura por la vida privada y cotidiana antes
que por las grandes ideas o los grandes temas. / ¿Qué opina
Espacio. La acción se desarrolla en Lima (Perú). Se nombran el autor sobre la clasificación que se hace de su novela Un mun-
diversos lugares: la avenida 28 de julio, un mercado, el puen- do para Julius? Según el autor El mundo para Julius quiso
te colgante... clasificarse como literatura comprometida y, para él, se trata
Tiempo. ¿Cuánto tiempo transcurre en el fragmento que has leído? de una novela individualista que simplemente intenta incor-
Unas horas: desde que Julius busca a Peter y lo encuentra en porar el placer por contar historias.
el mercado hasta que, después de tomar unas gaseosas, lle-
gan hasta el puente colgante. / ¿En qué época o momento se si-
túan los hechos que se narran? A medidados del siglo xx. La norma ortográfica (pág. 169)

Personajes. El personaje principal es Julius, el niño, y el se-


cundario Peter, el pintor.
 1. Meteorología, espurio, gasoducto, paralímpico, línea, Joa-
quín, aeropuerto, cotidianidad, geranio, espontáneo.
Trama. Julius es un niño inteligente y curioso que vive en el
seno de una familia acomodada, cuya infancia transcurre  2. Aero-: aeronáutico, aeromotor, aeropuerto, aeroplano, aeros-
rodeado por los mayordomos y las empleadas que trabajan tático, aerofagia, aerolito, aerofobia, aerómetro, aeródromo... /
en su casa. Su padre muere cuando él es muy pequeño y su Meteoro-: meteorología, meteorólogo, meteorito, meteoro,
madre vuelve a casarse después. Años más tarde también meteorismo...
pierde a su hermana Cinthia, a quien quería y admiraba
mucho. Debido a la falta de atención de sus padres, busca  3. El contraataque de las tropas por tierra tuvo lugar de madru-
apoyo en la servidumbre y en otros adultos, como Peter, el gada. / Los antiinflamatorios que me recetó el traumatólogo no
pintor. me están haciendo efecto. / El alcohol perjudica seriamente la

53
Unidad 8 >  El español actual

salud. / Esta semana estamos estudiando las proposiciones Antes del latín, en laPenínsula Ibérica se hablaban las lenguas
coordinadas copulativas. prerromanas: Verdadero. / Las lenguas románicas, entre las que
se encuentra el castellano, proceden del latín vulgar: Verdadero.
 4. Reelegir, reescribir, contraalmirante, extraordinario, extraaca- / Al principio, el romance castellano solo se hablaba en la zona de
démico. Castilla: Falso. / El Cantar de mío Cid es la primera obra litera-
ria escrita en castellano: Verdadero. / La unificación de los distin-
Escribe una oración...: Respuesta abierta.
tos reinos a finales del siglo xv favoreció la unificación del
castellano: Verdadero.
 5. Leer: ilegible, releer, lector, lectura, lectivo... / Poseer: despo-
seer, coposesión, poseído, poseedor, posesión, poseso... / Transforma los enunciados falsos... : El español es la cuarta
Crear: procrear, recrear, creación, creador, creativo, creacio- lengua más hablada en el mundo, tras el chino, el inglés y el
nista... / Proveer: desproveer, aprovisionamiento, proveedor, hindi. / En sus inicios, el romance castellano comenzó siendo
proveimiento... un habla local de una zona que corresponde a la actual Can-
tabria y parte de la provincia de Burgos.
 6. Entrad, estáis en vuestra casa. / Entregadle las llaves al nuevo
inquilino del piso y explicadle las condiciones del alquiler. /  2. Sustrato: Lenguas habladas antes de que se implantara el la-
Sentaos como es debido en la mesa y lavaos los dientes des- tín que fueron absorbidas por este, pero dejaron huellas en el
pués de comer. / Señalad los epítetos de esta composición y latín vulgar que se desarrolló en cada zona. Ejemplos: celta,
analizad su métrica. / ¡Abridles la puerta, que se han dejado íbero. / Superestrato: Lenguas de los pueblos que se estable-
las llaves en la oficina! / Compraos la playstation con este di- cieron en las regiones donde estaba implantado el latín. No
nero, os lo merecéis. llegaron a imponerse, pero influyeron en ciertos aspectos de
las lenguas románicas que evolucionaban en la región. Ejem-
 7. cc: calefacción, accidente, accesible, reacción, transacción, plo: árabe.
acceso, eccema... / ct: activar, actor, reactor, efecto, noctám-
bulo, intelecto... / gn: agnóstico, magnífico, ignífugo, ignorar,  3. Arabismos: almohada, adalid, alacena, alubia, albahaca, ace-
digno... / cn: acné, tecnológico, tecnología, técnica, pirotec- quia, azúcar, bellota, carmesí, cenefa, cítara, dársena, enca-
nia... / ng: ingenuo, ángel, angustia, engendro, ingrato... / ns: ramar, engarzar, faltriquera, fanega, halagar, hazaña, jarcha,
transformar, enseñar, ansiedad, monstruo, transporte... jarra, jinete, laúd, marfil, mazmorra, nácar, noria, rambla,
riesgo, sandía, sirope, sorbete, tabique, tifón, zaguán...
 8. Llegamos a la sala de congresos y comenzamos la reunión
ipso facto. / Me explicó grosso modo en qué consistía el tra-  4. La Real Academia Española tiene la función de fijar y estabili-
bajo que debíamos realizar. / Aquella extraña señora quiso zar la ortografía española. Se dedica a elaborar reglas norma-
pagarnos la reparación del coche motu proprio. / Se cree tivas para la lengua española y velar por la preservación y
que en los próximos años cambiará el status quo de la eco- unidad de esta lengua en todos los territorios donde se habla.
nomía mundial.
 5. ¿A qué variedad peninsular crees que pertenece cada uno de los
 9. Siempre me desternillo de risa cuando le veo imitar a ese fragmentos? Texto A: Fabla. / Texto B: Bable. / Texto C: Anda-
político. / No sé si mi madre ha hecho albóndigas o croque-
luz.
tas para cenar (la palabra almóndiga está aceptada por la
RAE, aunque es más correcta albóndiga). / Mete toda la ropa Deduce los rasgos a partir de ejemplos extraídos de los textos:
planchada en tu armario (la palabra almario está aceptada Texto A: Conservación de la «f-» inicial latina (fabla). / Utiliza-
por la RAE, pero es más correcta armario). / Utiliza este den- ción de la combinación pronominal CI + CD con orden inverso
tífrico para lavarte los dientes. / El vagabundo era muy cono- (Sentiba una fabla que me s’antojaba). / Empleo de las desi-
cido en el barrio (la palabra vagamundo está aceptada por la nencias «-iba» y «-eba» en el imperfecto de indicativo (tene-
RAE, pero es más correcta vagabundo). / El rubí estaba in- ba, sentiba...).
crustado en el anillo de oro. / Me gusta ver cazar mosquitos
Texto B: Cierre de vocales neutras finales (ridículu, pueblu,
a los murciélagos (la palabra murciégalo está aceptada por
prestixu, negociu...). / Contracción de las preposiciones en y
la RAE, pero es más correcta murciélago). / Ponte el casco
por con artículos y determinantes (nel, nesi, na, pola...). /
para coger la moto.
Posposición de se y de los pronombres de CD y CI (allegóse,
abriósela...). / Pérdida de consonantes (ruíu). / Pérdida de la
10. Raíz o tronco de una familia o linaje: estirpe. / Arrancar de
«-e» final (paez). / Conservación de la «f-» latina inicial (fa-
cuajo o de raíz: extirpar. / Esperanza de realizar o conseguir
cer). / Se usa la desinencia «-iba» para el imperfecto de indi-
algo: expectativa. / Expeler el aire aspirado: espirar. / Acabar
cativo (diba). / Diminutivos en «-ín» (Anxelín).
con la vida: expirar. / Usurpar algo a alguien utilizando amena-
zas: extorsionar. Texto C: Seseo (grasia, dise...). / Neutralización de ⁄r⁄ y /l/ (er
talabartero, er mundo...). Pérdida de sílabas finales (to, na...)
11. /s/: xilófago, xenón, xenófobo, xilófono. / /ks/: exánime, in- y del fonema /d/ a final de palabra (verdá) y entre vocales
flexión, extranjero, tórax. /j/: México, Texas. (carcajás).

 6. El poema presenta los siguientes rasgos: seseo (lus, reflo-


ACTIVIDADES (págs. 184-185) resío), desaparición del fonema /d/ intervocálico (queao, par-
tío); vacilación en el timbre de las vocales (cimenterio);
conservación de consonantes sordas intervocálicas (disquía)
El español actual (PÁGS. 170-179) y presencia de los sufijos -ico/-ica en los diminutivos (sendica,
cascabelicos, palmerica...).
 1. El español es la segunda lengua más hablada del mundo: Falso. /

54
Unidad 8 >  El español actual

 7. Sitúa en un mapa de España...: El alumno debe situar en un ción de la h procedente de la «f-» inicial latina, neutralización
mapa de España: euskera (País Vasco, N de Navarra y SO de de los fonemas /r/ /l/ en posición implosiva, alteraciones del
Francia); catalán (Cataluña, Comunidad Valenciana [valencia- timbre de las vocales...
no], Baleares, Andorra, S de Francia y la ciudad italiana de
L’Alguer (Alghero, Cerdeña); y gallego (Galicia, franja occiden- 12. Maracas: guaraní. / Caoba: arauaco. / Chinchilla: aimara. / Cón-
tal de Asturias, O de León y NO de Zamora (Sanabria). dor: quechua. / Jaguar: guaraní. / Chirimolla: quechua. / Coli-
brí: arauaco.
Explica qué factores...: Para preservar el euskera, se han adop-
tado medidas educativas (planes de estudio de la enseñanza
13. La diversidad lingüística en Hispanoamérica se debe a moti-
obligatoria y universitaria) y en los medios de comunicación.
vos económicos, políticos y culturales, así como al diferente
La lengua ha resurgido gracias al interés popular por recupe-
sustrato de las lenguas indígenas y a la variedad del español
rarla. / El catalán se enseña en las escuelas y cuenta con
que se implantó en cada zona. Hay variedades que han reci-
abundantes medios de comunicación que contribuyen a su
bido mayor influencia del andaluz y el canario (el caribeño y
normalización lingüística. / El gallego está impulsado por los
el rioplatense); otras han recibido más influencia del nava-
medios de comunicación y por su implantación en la ense-
rroaragonés y el asturiano (variedades centroamericana, co-
ñanza obligatoria y universitaria.
lombiano-ecuatoriana ribereña y chilena), y otras más
¿Cuál de esas tres lenguas se diferencia...? La lengua que más cercanas al español estándar (mexicano, colombiano y pe-
se diferencia del castellano es el euskera. / ¿A qué crees que se ruano ribereño centrales).
debe? El euskera no es una lengua indoeuropea, por tanto no
es románica. 14. Explica el significado...: (Requete)amurada: Según la RAE, amu-
rar significa llevar a barlovento los puños de las velas y suje-
Explica algunas características...: Respuesta abierta. Ofrece- tarlos con la amura para que queden bien orientadas. Sin
mos la siguiente: Según se deduce del texto, en catalán el embargo, en este contexto tiene otro significado. En lunfardo
plural se forma en «-es» (diverses, parles, orelles...); a veces, (jerga originada y desarrollada en la ciudad de Buenos Aires),
se emplea el apóstrofe para suprimir una vocal (l’alta); los amurar significa dejar a alguien abandonado y, por tanto, (re-
participios de los verbos de la primera conjugación acaban en quete)amurada significa (requete)abandonada. / Pavada: gro-
«-at» (captivat); en ocasiones, se pospone al verbo (como en sería, tontería. / Engrupido: soberbio, fatuo, pretencioso. /
castellano), pero en catalán se separa con guión (fes-la). / El Empacar: empaquetar, encajonar. / Plata: dinero. / Morfar: co-
gallego contrae la preposición con el artículo (d’os meus mer. / Guita: dinero. / Secante (seco): que aburre.
ollos); mantiene la «f-» inicial latina (figueiriñas); no diptonga
las vocales breves tónicas (fontes, terra...). Explica las características del español de América...: no diptonga-
ción, cambios en los morfemas verbales, uso de voseo (¿Qué
 8. El español de Filipinas se caracteriza por su léxico propio: querés vos?) y diminutivos (moneditas).
anglicismos, neologismos y arcaísmos, préstamos léxicos y Lee en voz alta la canción...: Respuesta abierta. Se trata de que
cambios semánticos procedentes de las lenguas indígenas. El el alumno aplique sus conocimientos fonéticos del español de
chabacano, lengua criolla mezcla de español y lenguas nati- América al leer la canción.
vas, presenta seseo, «h-» aspirada en lugar de la «j-» castella-
na, etc. / El español de Guinea Ecuatorial se caracteriza por la 15. En estos anuncios y fragmentos cinematográficos se habla la
vacilación en el timbre de las vocales, presencia de yeísmo y variedad del español de América de Argentina. Se escucha
seseo, pérdida de «-l» a final de palabra, pérdida frecuente seseo, la pronunciación de fricativas sordas en lugar de la /y/
del morfema de plural «-s», empleo irregular o pérdida del castellana, y la aspiración de la /χ/. Por otra parte, destacan el
artículo y cambio de significado de algunos adverbios. Como uso de vos, la no diptongación y cambio de algunos morfemas
el español de Filipinas, destaca por la conservación de arcaís- verbales (perdés, podés, pagás) y la tendencia al uso del pre-
mos y la presencia de préstamos de lenguas autóctonas, an- térito indefinido (¿qué pasó?). Además, se usan palabras pro-
glicismos... Por otra parte, el español de Guinea Ecuatorial se pias del lunfardo argentino: guita, pibe, laburar...
caracteriza por la influencia de las lenguas aborígenes ban-
túes, que son tonales.
¿Cuáles de estas características coinciden...? El seseo es un rasgo El español en la red (págs. 180-183)
común del andaluz, el chabacano y el español de Guinea Ecua-
torial. En chabacano encontramos yeísmo y «h-» aspirada. 16. Respuesta abierta. Proponemos la siguiente: Las semejanzas
entre ambas comunicaciones, además del receptor y del emi-
 9. Para facilitar la comunicación con los indígenas americanos sor, están en el tipo de mensaje. Es decir, ambos interlocuto-
se utilizó a intérpretes como intermediarios. A veces, se fo- res intercambian una información similar en un mismo tono
mentó el uso de una lengua autóctona (náhuatl en México, coloquial y distendido. Sin embargo, existen importantes dife-
quechua en Perú...). Los misioneros y los jesuitas elaboraron rencias. Por ejemplo, en el código, oral en el primer caso y
diccionarios y gramáticas para enseñar y fijar las lenguas escrito en el segundo. Así, la comunicación de chat está con-
indígenas. dicionada por los elementos técnicos (utilización de ordena-
dores, teclados, conexión a Internet, etc.). Estos medios
10. No diptongación en las desinencias verbales (vomitá, tenés, permiten que la comunicación sea instantánea, pero no el
podés...); uso de diminutivos (hijito); vocabulario propio (pla- tiempo de respuesta. El emisor pierde una parte importante
nilla, pieza...). de la espontaneidad porque puede revisar su mensaje y vol-
ver a releer las intervenciones anteriores. Por otra parte, el
11. Presencia del seseo, aspiración o pérdida del fonema /s/ en emisor y el receptor no comparten el mismo contexto físico.
posición implosiva, síncopa o pérdida de consonantes, aspira- Ello provoca que la comunicación pierda toda la información

55
Unidad 8 >  El español actual

no verbal, que se intenta suplir a través de símbolos y marcas 20. Respuesta abierta.
gráficas que reproducen los estados de ánimo.
21. Respuesta abierta. Proponemos la siguiente:
17. Reescribe el texto corrigiendo los errores ortográficos y la pun-
La ciberhabla es un lenguaje novedoso que combina propie-
tuación:
dades del habla, la escritura y el medio electrónico.
Hola, Pedro: he mirado el archivo que me pedías y tiene un
Según el filósofo H. P. Grice, son cuatro las máximas conver-
virus que alucinas. ¡No hagas nada hasta que yo te mire el PC!
sacionales que debe presentar la ciberhabla: la máxima de
Bájate el software nuevo de www.virustop.com y ¡a ver si ac-
calidad según la cual todo lo que se diga en Internet debe
tualizas el equipo!
ser cierto y tenemos que contar con pruebas que lo confir-
¡No te va a pasar nada si el año que viene te compras un anti- men; la máxima de relevancia, pues todo lo que digamos
virus! :-) debe ser relevante; la máxima de cantidad que indica que la
contribución que hacemos al comunicarnos a través de In-
Es posible que te haya entrado por un spam. Si te conectas a
ternet debe tener un propósito y no debe ser más informativa
la Intranet de la facultad verás que hay peña igual que tú. Esta
de lo necesario; y, por último, la máxima de manera según la
noche nos vemos en casa de Toni para estudiar...
cual al expresarnos mediante la ciberhabla debemos ser
Identifica en el texto los rasgos más característicos...: El texto claros, breves y ordenados.
contiene muchos rasgos de la lengua usada en Internet. En
No es fácil trabajar sobre lo que ocurre en Internet, entre otras
primer lugar, encontramos un uso extendido de tecnicismos
cosas, porque los individuos intervienen y participan de ma-
procedentes del mundo de la informática y la lengua inglesa
nera anónima, especialmente cuando se trata de grupos de
(software, spam, etc.), y de neologismos (Intranet, antivirus...)
chat y mundos virtuales.
para aludir a las nuevas realidades de este nuevo medio de
comunicación. Actividad PBL: Es importante que los alumnos sean cons-
cientes de los riesgos que supone el ciberacoso y conocer la
También observamos el desarrollo de una terminología propia
forma de evitarlo como parte del desarrollo de su competen-
de la Red, que alude a funciones informáticas de los progra-
cia digital. Para ello, no solo deben conocer los códigos de
mas utilizados en la comunicación (bajarse, actualizar...).
conducta y de buenas prácticas que es necesario emplear a
De igual modo, encontramos en el texto el empleo de una la hora de navegar por Internet, sino que también deben sen-
simbología que reemplaza al lenguaje no verbal, como emoti- tirse parte de la solución ante un caso de ciberacoso cuando
conos [:-)], glosas para aclarar la intención del emisor (<gru- detectan un problema o situación peligrosa. Es importante
ñidos>), el uso de las mayúsculas para dar intensidad al que entiendan que es fundamental participar para ayudar a
mensaje (NO HAGAS NADA HASTA QUE YO NO TE MIRE EL sus iguales a evitar, gestionar mejor o erradicar las situaciones
PC), etc. También encontramos la utilización de siglas (PC). de abuso en la Red.
Uno de los rasgos más llamativos es la relajación de la pun-
tuación y la ortografía, que encontramos en todo el texto (pe-
dias, q, entrao, etc.).
Evaluación (pág. 189)

¿Qué características de la lengua oral...? El texto, aunque está  1. Respuesta abierta.
escrito, intenta reflejar la coloquialidad de la lengua oral. Esta
intención se observa en el uso de emoticonos y signos que  2. Para el autor «leer», sea en la red o a través de otro medio,
intentan suplir la información no verbal. Igualmente, el em- consiste en acceder a la información que ofrece un texto, en-
pleo del léxico (flipar, facu, peña...) y la relajación ortográfica tendiéndolo e interpretándolo, de forma calmada. Y «surfear»
denotan un intento de emular el tono coloquial. en la red, según el autor, consiste en navegar por Internet sin
buscar nada concreto.
18. Respuesta abierta. Proponemos la siguiente: Seguramente, la
lengua no corre ningún peligro por el uso coloquial y descui-  3. ¿Por qué Internet no está hecho para leer? Porque para leer
dado que se hace de ella en Internet. Sin embargo, este rasgo de forma correcta hay que hacerlo con calma para poder
se puede convertir en una amenaza si los usuarios generali- comprender y asimilar lo que hemos leído. / ¿Para qué está
zan los malos usos, que llegan a poner en peligro la compren- hecho Internet? Internet está hecho para transmitir informa-
sión del mensaje y la uniformidad de la lengua. ción vertiginosamente, o para acceder a ella con la misma
velocidad. / ¿Entre qué dos clases de intelectuales distingue el
Uno de los problemas para la proliferación de malos hábitos autor del texto? Distingue entre quienes leen con lentitud y
radica en que muchos usuarios no perciben las faltas de orto- lo que leen vertiginosamente. / ¿De qué carecen los mensajes
grafía en Internet como un signo de ignorancia, sino como que se transmiten mediante Internet? Carecen de argumenta-
una libertad que se perdona por la rapidez con la que se es- ción y de sutileza.
criben los mensajes. Es importante concienciarse de la nece-
sidad de respetar las normas ortográficas, ya que de otro  4. El autor recomienda a los lectores que utilicen Internet para
modo se estaría contribuyendo al empobrecimiento del idio- buscar información «vertiginosamente», pero después esa in-
ma y, en un caso extremo, a la creación de jergas incompren- formación la lean de forma reposada y fructífera, inteligente-
sibles para la mayoría de los hablantes que pondrían en mente en su sillón.
peligro la comunicación y la unidad del idioma.
 5. Eficacia: Capacidad para obrar o para producir el efecto
19. Respuesta abierta. La actividad está encaminada a que los alum- que se desea o se espera. / Sutileza: Agudeza, ingenio o
nos descubran por sí mismos errores cometidos en los medios habilidad para hacer algo. / Elocuencia: Facultad para ha-
de comunicación escritos y sean capaces de corregirlos. blar o escribir de modo eficaz para deleitar, conmover o

56
Unidad 8 >  El español actual

persuadir. / Ingenio: Facultad mental para discurrir, crear o trata de listados de palabras traducidos del latín al castellano
inventar con rapidez. porque ya no se comprendían las palabras latinas.

 6. Vertiginosamente: velozmente, aceleradamente, trepidante- 10. Variedades históricas, procedentes del latín que se conforman
mente. / Fulgurantes: fúlgidos, centelleantes. / Acortar: abre- en la península: navarroaragonés y asturleonés. / Variedades
viar, reducir, simplificar. / Fructífera: provechosa, productiva. meridionales, caracterizadas por rasgos que las diferencian
del castellano hablado en otros lugares: andaluz, canario, ex-
 7. Respuesta abierta. tremeño y murciano.

 8. Superestrato: Lenguas de los pueblos que se establecieron 11. El judeoespañol era la modalidad hablada por los judíos espa-
en las regiones donde estaba implantado el latín ya evolu- ñoles o sefardíes. Actualmente, se encuentra en proceso de
cionado. No llegaron a imponerse, pero influyeron en cier- progresiva decadencia y ha quedado reducido al ámbito fami-
tos aspectos de las lenguas románicas que evolucionaban liar. Se caracteriza por su semejanza con el castellano medie-
en la región. / Sustrato: Lenguas habladas antes de que se val y por la conservación de rasgos arcaicos, como la distinción
implantara el latín que fueron absorbidas por este, pero de los fonemas /b/ y /v/ en posición inicial.
dejaron huellas en el latín vulgar que se desarrolló en cada
zona. / Romanización: Proceso que se inicia con la llegada
12. La comunicación en Internet presenta una serie de rasgos pro-
pios: el mensaje es relativamente efímero, es decir, se suele
de los romanos a la Península, en el año 218 a. C., y supu-
eliminar tras la conversación; la comunicación es simultánea,
so la adopción de la lengua latina. / Ciberhabla: Es la forma
aunque no lo es el tiempo de respuesta; el emisor y el receptor
peculiar que adopta una lengua en Internet. / Emoticono:
no comparten un contexto real sino virtual; abuso de la función
Signo que reproduce en Internet un elemento de la comu-
fática para abrir el canal de comunicación entre los interlocuto-
nicación no verbal. Se forma normalmente con la combina-
res; Internet es un medio que estimula la formación de comuni-
ción de signos ortográficos del teclado que se esté
dades lingüísticas articuladas en torno a un determinado tema.
empleando.
13. Respuesta abierta.
 9. Los primeros términos escritos en castellano son las Glosas
silenses y las Glosas emilianenses. Datan del siglo x y se

57
0# Literatura

 1. Jorge Manrique: Edad Media, lírica culta. / Emilia Pardo Bazán:  3. El texto de Miguel de Cervantes pertenece a la corriente de la
edad contemporánea, naturalista, novela. / José Cadalso: edad narrativa idealista y el de Benito Pérez Galdós es un texto
moderna, Neoclasicismo, didáctica. / Garcilaso de la Vega: edad plenamente realista. La diferencia entre ambos radica en la
moderna, Renacimiento, poesía italianizante. / José de Espronce- temática; si bien en el primero aparecen personajes idealiza-
da: edad contemporánea, Romanticismo, poesía. / Gonzalo de dos que viven historias fantásticas, en los de corte realista se
Berceo: Edad Media, mester de clerecía. / Francisco de Quevedo: recrean escenas de la vida cotidiana protagonizadas por per-
edad moderna, Barroco, conceptismo. / Leopoldo Alas «Clarín»: sonajes de la sociedad de su tiempo.
edad contemporánea, realismo, novela. / Anónimo, Cantar de Mio
Cid: Edad Media, épica. / Juan Ruiz, Arcipreste de Hita: Edad  4. a) Respuesta orientativa. En el cuadro aparecen dos persona-
Media, mester de clerecía. / Baltasar Gracián: edad moderna, jes totalmente contrapuestos. En un primer plano, apoyada
Barroco, prosa didáctica. / Lope de Vega: edad moderna, Barroco, en la barandilla del balcón, se encuentra una joven mu-
comedia nacional./ Anónimo, Lazarillo de Tormes: edad moderna, chacha ataviada con sus mejores galas. Detrás de ella en-
Renacimiento, novela picaresca. / Benito Jerónimo Feijoo: edad contramos a una anciana y fea mujer que, con una mirada
moderna, Neoclasicismo, didáctica. / Luis de Góngora: edad mo- pícara, parece que estuviera tramando algo.
derna, Barroco, culteranismo. / Fernando de Rojas: Edad Media,
b) Posible respuesta. La mujer anciana representa una celes-
teatro. / Mariano José de Larra: edad contemporánea, Romanti-
tina, o alcahueta, que probablemente quiera obtener be-
cismo, artículo de opinión. / Don Juan Manuel: Edad Media, na-
neficios, favoreciendo las relaciones de la muchacha con
rrativa didáctica. / M iguel de C ervantes : edad moderna,
un posible pretendiente.
Renacimiento, novela de caballerías. / Benito Pérez Galdós: edad
contemporánea, realismo-naturalismo, novela. / Fray Luis de c) Tal vez podría establecerse una relación entre la escena
León: edad moderna, Renacimiento, poesía religiosa. / José Zorri- representada en el cuadro con la temática de los matrimo-
lla: edad contemporánea, Romanticismo, teatro. / Leandro Fer- nios de conveniencia de las obras de Leandro Fernández
nández de Moratín: edad moderna, Neoclasicismo, comedia de Moratín por tratarse en ambos casos de relaciones inte-
burguesa. resadas. Si bien en el primer caso la Celestina puede obte-
ner un provecho por actuar de intermediaria en las
 2. El poema A es de Jorge Manrique, uno de los grandes poetas relaciones de los interesados, en los matrimonios de con-
del siglo xv. El texto pertenece al poema elegíaco Coplas a la veniencia el interés repercute en la familia de la mucha-
muerte de su padre, en el que el autor alaba la vida y las vir- cha, que con el enlace de su hija con un viejo se asegura
tudes de su padre. En este extenso poema (40 coplas) Jorge su tranquilidad económica.
Manrique reflexiona sobre la muerte y la fugacidad del tiem-
po, a la vez que invita a aceptar la muerte con resignación. El  5. Las tres composiciones tienen una función didáctica-morali-
autor emplea un estilo sencillo para expresar sus reflexiones zante. El conde Lucanor pertenece a la narrativa didáctica
y sentimientos. de la Edad Media. A partir de un cuento o narración breve
(enxiemplo) el autor presenta un caso con la finalidad de
El poema B de Garcilaso de la Vega es renacentista. El autor entretener y aleccionar al lector. El Criticón, de Baltasar Gra-
está considerado el gran renovador de la poesía castellana cián, pertenece a la prosa didáctica y filosófica del siglo xvii.
del siglo xvi al introducir las formas italianizantes (versos En su obra, el autor exalta las virtudes necesarias para triun-
heptasílabos y endecasílabos, y estrofas, soneto, lira…). En far y presenta unas enseñanzas útiles para orientarse en la
este poema el autor recrea en el paisaje sus sentimientos vida. El ciervo y la fuente pertenece al género de la fábula.
más íntimos. Este género fue muy utilizado en el siglo xviii para transmitir
las ideas reformistas de la Ilustración. A partir de la recrea-
En el poema C, el autor romántico Gustavo Adolfo Bécquer ción de una escena protagonizada por animales, personas y
emplea un estilo sencillo y sereno para expresar sus senti- seres animados e inanimados se pretende instruir a la vez
mientos más íntimos. que se entretiene.

58
9# El modernismo y la generación del 98

 3. a) El profesor debe mantener con los alumnos una relación
EN CONTEXTO (pág. 195)
más abierta y cercana, suprimiendo las barreras que pue-
a) La Residencia de Estudiantes desempeñó un papel muy impor- dan encontrarse entre ellos.
tante en la historia y la cultura europeas durante las primeras
b) El autor promueve una actitud de diálogo y tolerancia, en la
décadas del siglo xx. Desde la Residencia se promovía una co-
que los estudiantes puedan intervenir y se sientan partíci-
municación interdisciplinar, en la que se fomentaba el diálogo.
pes de la obra del maestro. De este modo podrán sentirse
Por ella pasaron artistas de diferentes disciplinas: cineastas
protagonistas del devenir del mundo, pues tendrán con-
(Luis Buñuel), compositores (Manuel de Falla), pintores (Salva-
ciencia de sí mismos y de su individualidad.
dor Dalí), científicos (Severo Ochoa, Blas Cabrera), médicos
(Ramón y Cajal), filósofos y pensadores (José Ortega y Gasset),
escritores, intelectuales, poetas (Eugenio d’Ors, Manuel Altola- El modernismo  (págs. 199-201)
guirre, Pepín Bello, Federico García Lorca, Rafael Alberti, Jorge
Guillén, Miguel de Unamuno, Pedro Salinas).
 4. Poema A. El tema es el recuerdo de la amante. / 10A 10B 10A
La inquietud que compartían todos ellos era la reforma y la mo- 10B. Rima consonante. Poema B. El tema es el destierro del
dernización de España. Cid. / 7- 11A 11- 11A 11- 11A 11- 11A. Rima asonante. Poe-
ma C. El tema es la angustia existencial. / 11A 11B 11B 11A
b) El azul es un color asociado con la fantasía y las ideas utópicas,
11A 11B 11B 11A 11C 11D 11C 11B 11D 11B. Rima conso-
con la divinidad, la nostalgia y la tristeza. Algunos autores mo-
nante. Poema D: El tema es la creación poética. / 14A 14A 14B
dernistas emplean este color en sus composiciones porque les
14B 14C 14C. Rima consonante y asonante. Poema E: El tema
sugiere el exotismo y el misterio que querían adoptar en sus
es la identidad cultural emprendedora. 14A 14B 14B 14C 14C.
obras. Por otro lado, el modernismo tiende a expresar en sus
creaciones la inquietud, la tristeza y la melancolía que le produ- Justifica el tratamiento modernista: Poema A: el poeta busca
ce el rechazo a lo establecido, que también puede asociarse al evadirse recreándose en la mitología clásica; emplea un len-
color azul. guaje refinado y culto que aporta musicalidad. Poema B: Se
reproduce un pasaje glorioso de la historia de España. Poema
C: hastío vital y malestar existencial. Poema D: lenguaje original
ACTIVIDADES (págs. 210-213) y tratamiento exótico. Poema E: exaltación de los auténticos
valores españoles para contrarrestar la influencia americana.

Contexto sociocultural  (págs. 196-197)  5. a) Son versos dodecasílabos con rima consonante ABAB
CDCD EFEF. Este tipo de métrica y rima es muy utilizada
 1. a)  Los dos acontecimientos que se mencionan son, en primer por los poetas modernistas.
lugar, la Restauración borbónica, etapa política comprendi- b) Son serventesios.
da entre 1875 y 1931, de cierta estabilidad política, basada
en el bipartidismo; y, en segundo lugar, las guerras colonia- c) El poeta utiliza un léxico refinado y sensual para expresar
les de Cuba y Filipinas (1898), en las que España perdió sus emociones, introduciendo incluso algún cultismo (eo-
sus dos últimas colonias a manos de los americanos. lias, del dios Eolo). Aunque predominan los sustantivos
(aire, hada, vuelos, terraza, ramajes, trajes…) la adjetiva-
b) La idea de desazón aparece asociada a la generación de ción cumple un importante papel, pues sirve para crear el
los nacidos hasta 1870. Estos hombres vivieron las secue- ambiente poético refinado (suave, pausados, vagas, te-
las de la Restauración, sufrieron el fracaso político y colo- nues, sedosos…).
nial, y no supieron distinguir la realidad.
d) Paralelismo, versos 11 y 12: el vizconde rubio de los desa-
c) La afirmación significa que esta generación empezó a te- fíos / y el abate joven de los madrigales. Aliteración, verso
ner conciencia de los problemas que afectaban el país e 4: entre los sollozos de los violoncelos (la música de los
intentó afrontarlos ahondando en su historia y buscando violoncelos). Personificación, verso 4: entre los sollozos de
una solución en Europa. los violoncelos. El llanto es una cualidad humana que se
atribuye al instrumento.
 2. Posible respuesta. La concepción de intelectual que se des-
prende del papel de los escritores modernistas y noventayo- e) Los temas modernistas que aparecen en el poema son la
chistas sufre una evolución. Así, en el modernismo los evasión (presenta un universo idílico y exótico, con un
escritores y artistas se distancian de la sociedad y se refugian hada en una terraza llena de flores...) y el amor (una mar-
en el arte y la literatura. Con la generación del 98 los intelec- quesa que es pretendida por un vizconde y un abate).
tuales (principalmente escritores) se implican más en la so-
ciedad y el país en el que viven, tratando de proponer  6. a) El tema de este poema es la angustia causada por el dolor
soluciones a sus problemas. Esta actitud coincide más con la de vivir y el enigmático sentido de la vida. / Pertenece al
concepción que se tiene actualmente del intelectual: una libro Cantos de vida y de esperanza (1905), que forma
persona con cierta capacidad para pensar la realidad social y parte de la etapa más trascendente y reflexiva del autor.
cultural y de influir críticamente en la opinión mediante el en- En esta etapa el poeta emplea un lenguaje más sobrio para
sayo o la presencia en los medios de comunicación. expresar su angustia y sus preocupaciones existenciales.

59
Unidad 9 >  El modernismo y la generación del 98

b) La primera estrofa presenta la dicha de los seres inanima- La generación del 98  (págs. 202-207)
dos (árbol, piedra) porque ellos no son conscientes de su
existencia y no sufren.  9. Respuesta orientativa. La generación del 98 cumple bastantes
requisitos para tener esta consideración: semejante formación
La segunda y tercera estrofas, profundizan en los motivos
cultural (son todos universitarios), relaciones personales entre
de esa pesadumbre: la incertidumbre de nuestro destino,
ellos (solían encontrarse en los cafés para discutir distintos
la certeza de la muerte, el sufrimiento que acarrea la vida y
temas), participaban en actos comunes, publicaban en revis-
lo desconocido, y el engaño de la carnalidad.
tas, presencia de un acontecimiento histórico (pérdida de
Finalmente, la cuarta estrofa actúa de colofón del poema, Cuba y Filipinas en 1898), presencia de un guía (Grupo de los
pues remacha la idea del sinsentido de la existencia alu- Tres, Rubén Darío), utilización de un lenguaje específico y
diendo a un futuro incierto y a unos orígenes desconocidos. edades similares entre sus integrantes.
c) Las dos primeras estrofas son serventesios de versos ale- Explica las semejanzas y las diferencias… Los modernistas y los
jandrinos, de rima consonante cruzada ABABA CDCD. / autores de la generación del 98 se asemejan en el hecho de
La tercera estrofa es un terceto de versos alejandrinos con que mantienen una actitud de rebeldía y en que tratan en sus
rima consonante EFE. La última estrofa está compuesta obras los problemas existenciales. Las diferencias radican
por un verso eneasílabo y un verso heptasílabo con rima principalmente en el estilo y en el modo de enfrentarse a la
consonante EF. realidad. Los modernistas buscan la belleza y cuidan las for-
mas en una poesía que busca evadirse de la realidad. Los
d) El lenguaje es sobrio, menos colorista que el de la primera noventayochistas emplean un lenguaje rudo y arcaico para
etapa. Aunque aún aparece algún adjetivo epíteto (piedra expresar la situación agónica que sufren.
dura), se emplea un léxico bastante preciso, que aporta
fluidez al discurso. El ritmo del poema es, por tanto, rápido 10. Explica con tus palabras cómo es el viejo Tellagorri. Respuesta
debido principalmente a la anáfora (uso reiterado de la abierta en la que el alumno/a debe mencionar el carácter de Te-
conjunción copulativa «y» al principio de la segunda y ter- llagorri (alegre, jovial, buen amigo, violento, astuto, ladrón, cínico,
cera estrofas). El encabalgamiento (al final de la segunda independiente....) y su manera de vivir (hombre de rapiña...).
estrofa y el principio de la tercera) produce sensación de Indica a qué tipo de personaje propio… Es un personaje típica-
desasosiego y, junto con la rima consonante, confiere so- mente barojiano, un antihéroe, un marginado social que lucha
noridad y musicalidad al poema. por sobrevivir.
Baroja utiliza con gran habilidad y sutileza un recurso… Utiliza
 7. a) Mirto: arrayán, arbusto. Galantería: acción o expresión
la personificación del perro de Tellagorri, Marqués, para com-
obsequiosa, cortesana o de urbanidad. Fronda: conjunto
pletar la descripción de su amo: «un perro chiquito, feo, con-
de hojas o ramas que forman espesura. Madrigal: compo-
tagiado hasta tal punto con las ideas, preocupaciones y
sición literaria que debe tratar, en combinación armónica
mañas de su amo, que era como él…».
o sencilla, un pensamiento ingenioso o delicado general-
mente dirigido a una dama. Venerable: digno de venera- 11. a) El pueblo fue a casa del mártir a por reliquias y recuerdos
ción, de respeto. Monástica: perteneciente al estado de suyos, no querían creer que don Manuel había muerto y
los monjes. Quimérico: fabuloso, fingido o imaginado sin que era un santo.
fundamento.
b) Porque le dio fe; una nueva vida.
b) Como ejemplos del lenguaje poético modernista podemos c) Habla de dos tipos de feligreses. Unos, convencidos de la
citar: La riqueza y belleza del léxico empleado (mirtos vida de ultratumba, que atormentan a los demás para que
seculares, vejez señorial y melancólica, fronda, cipreses se ganen la otra, despreciando esta; otros, los que creen
venerables, huellas ideales y quiméricas…). / El refina- que solo existe esta vida, esperan una sociedad futura y
miento de las palabras cultas (galantería, madrigales, niegan al pueblo el consuelo de creer en otra.
evoqué, cándido, heráldico, límpido, quimérica…). / La
abundante adjetivación (fuente abandonada, ¡Hermosos d) Alegría y consuelo en la vida.
y lejanos recuerdos!, mañana otoñal y dorada, constante
lluvia…), en algunos casos referida a los sentidos (jardín 12. a) El autor desarrolla una visión personal (prefiero…) me-
húmedo y reverdecido, manos blancas…). / Las compa- diante un estilo claro que evita las expresiones filosóficas
raciones y metáforas construidas con imágenes precio- complejas y la terminología rebuscada; por ejemplo: «Pues
sistas y casi mágicas (como un pájaro de oro, el el progreso de la vida espiritual consiste en pasar de los
terciopelo de la yerba, como si danzasen invisibles ha- preceptos negativos a los positivos».
das, iban deshojando las margaritas…). b) El texto trata de la necesidad de decir la verdad. El autor
argumenta que la verdad es el valor más importante del
Con todo ello consigue recrear un universo de ensueño: un
ser humano, y que no basta con no mentir: es necesario
jardín y un palacio, con su fuente, abandonados.
decir la verdad. Plantea el caso de lo que ocurriría si todo
el mundo dijera la verdad.
 8. a) El tema típicamente modernista que aparece reflejado en
estos versos es el de la angustia de la condición humana. c) Trata sobre el sentido de la vida (problemas existenciales)
y la necesidad de vivir según la verdad.
b) El poeta expresa sus deseos de fundirse, identificarse con
el mar, exaltando la belleza y la inmensidad de la naturale- 13. a) En el fragmento aparece la descripción de una típica es-
za. Vivir y el deseo de una muerte que lo alivie. tampa de la vida castellana y la creación literaria.
c) Con la expresión «no pensar en nada» se refiere a la b) Azorín quiere recordar un pasaje glorioso de la realidad
quietud y la paz que le proporciona haberse fundido con histórica de España, para que se tenga en cuenta la gran-
el mar (místico). deza del país.

60
Unidad 9 >  El modernismo y la generación del 98

Antonio Machado  (pág. 208) 2. Contextualización


a) El país vivía una etapa de pobreza y descontento social. La so-
14. a) El poeta relaciona el olmo que se muere con la enferme-
ciedad española era mayoritariamente rural, con recursos eco-
dad de su mujer Leonor, que padece tuberculosis y está
nómicos escasos y una gran desigualdad social. A pesar de los
grave
intentos reformistas, la alternancia de partidos hacía difícil que
b) Se contraponen la casi certeza de la muerte que está al pudieran llevarse a cabo.
llegar («… hendido por el rayo / y en su mitad podrido»,
b) La política española vive unos años convulsos, motivados princi-
«Antes que te derribe», «antes que rojo en el hogar…»)
palmente por la pérdida de las últimas colonias y las continuas
con una última esperanza de vida, de curación de Leonor
alternancias de poder. Los intelectuales españoles miran a su
(«algunas hojas verdes han salido», «rama reverdecida»,
país con tristeza. Ven cómo se frustran sus ansias de progreso
«mi corazón espera», «otro milagro de la primavera»).
social e intelectual y sienten la necesidad de reflexionar sobre la
c) Quiere dejar constancia, a través del poema, de la posible identidad de España.
resurrección del olmo y de Leonor.
c) En 1907 Machado se traslada a Soria para trabajar como profe-
d) Olmo: la enfermedad de Leonor; camino: recuerdo de los sor de francés y quedó prendado de aquellas tierras. En Soria
lugares recorridos; río: la vida, siguiendo la imagen de fue donde conoció a su mujer Leonor.
Jorge Manrique; mar: la muerte; corazón: el poeta; rama
d) Leonor Izquierdo fue la mujer de Antonio Machado, la conoció en
verdecida: la esperanza.
1908 y se casó con ella al año siguiente. Murió muy joven, enfer-
e) 11A 7b 11A 11B 11C 11D 11C 11D 11E 11F 11E 11F 11G ma de tuberculosis, lo que afectó hondamente al poeta. Ese sen-
11G 11H 11H 11I 11J 11I 11J 7l 11L 11G 11- 7G 11F timiento se refleja en su obra, en la que a partir de entonces la
11M 7f 11M 11F. Rima consonante, salvo el verso 24, que imagen del campo de Castilla se asocia con el recuerdo de su
queda suelto. mujer y el sentimiento de soledad que experimenta el poeta.
f) Epítetos: hojas verdes (v. 4), pardos ruiseñores (v. 11).
Anáfora: antes que (vv. 15, 19, 22, 24). Personificación: 3. Expresión
que lame el Duero (v. 6); álamos cantores (v. 9). Metáfo-
ra: Ejército de hormigas (v. 12). Hipérbaton: antes que a) Machado abandona el vocabulario modernista. Con un estilo cla-
rojo en el hogar, mañana, / ardas en alguna mísera caseta ro, sobrio y directo, describe el paisaje en sintonía con sus senti-
(vv. 19-20), Mi corazón espera / también, hacia la luz y mientos. El poeta utiliza un léxico sencillo y huye de la retórica
hacia la vida, / otro milagro de la primavera (vv. 28-30). excesiva. En sus textos predomina la sencillez del estilo nominal.
Estos recursos buscan la expresión del sentimiento con b) A través de los adjetivos representa los tonos del paisaje soria-
imágenes claras y precisas y sustantivos finales: luz, vida, no. Poema A: oscuros, ariscos, calvas, blancos, plateadas, gri-
primavera. ses, cárdenas; poema B: plomizos, raídos, polvorientos. La
g) El poema pertenece al libro Campos de Castilla, obra ausencia de formas verbales en ambos poemas provoca una
fundamental del poeta. Machado, en un estilo ya depura- sensación de quietud en el paisaje y da protagonismo a la des-
do, describe el paisaje castellano y refleja su amor por cripción de sus distintos elementos. En el poema C los adjetivos
Leonor. (verde, azul, serena) dan la sensación de paz y armonía, que le
producen los recuerdos de su vida con Leonor.
c) Siente tan real, tan verdaderas, la voz y la mano de Leonor que
ANÁLISIS Y COMENTARIO DE TEXTOS espera que la tierra se la devuelva. («¡Eran tu voz y tu mano, en
(pág. 214) sueños, tan verdaderas!...»).
d) Poema A: 7- 11A 7- 7a 11- 11A 11- 11A 11- 7a 11- 11A 11-
1. Comprensión 11A. Combinación de versos heptasílabos y endecasílabos y
rima asonante; es una silva asonantada.
a) En el poema A el poeta manifiesta un sentimiento de tristeza
por la tierra castellana y, al mismo tiempo, de identificación Poema B: 7- 7a 7- 11A 11- 11A 11- 7a 11- 7A 11- 11A 7- 11A.
con el paisaje de Castilla, lo que le lleva a sentir amor por él Combinación de versos heptasílabos y endecasílabos y rima
(«…en el fondo / del corazón, tristeza, / tristeza que es amor!», asonante; es una silva asonantada.
vv. 9-11). Poema C: 8- 8a 8- 8a... Son versos octosílabos, riman en aso-
En el poema B Machado recuerda en sueños su amor por el nancia los pares y los impares quedan libres; es un romance.
paisaje castellano. Cuando escribe ese texto, el poeta ya no resi- e) Exclamación retórica: Todo el poema A es una exclamación
de en Castilla y su esposa ha fallecido, por lo que el poema se retórica en la que poeta exalta la belleza del paisaje castellano. /
llena de melancolía, de angustia y soledad. vv. 43-44: «¡Quién sabe lo que se traga la tierra!». Deseo de en-
En el poema C se contraponen las ideas de sueño y realidad. El contrar de nuevo a Leonor tras su muerte. Metáfora: v. 4: «su
poeta expresa el sentimiento de proximidad con su mujer al so- curva de ballesta». Forma del río Duero a su paso por Soria. In-
ñar con ella. terrogación retórica: vv. 21-22: «¿No ves, Leonor, los álamos del
río / con sus ramajes yertos?». Se dirige a su esposa ya fallecida.
b) El paisaje castellano produce una inmensa emoción en el poeta, Enumeración: vv. 1 al 7. Descripción intimista del paisaje por
que describe de manera impresionista los colores y las formas medio de sustantivos que hacen referencia a los elementos
del paisaje. paisajísticos. Paralelismo: vv. 38-39: «como una campana nue-
c) El poeta sueña con su mujer y siente que está con ella. Esta va, / como una campana virgen». La repetición produce un
sensación le produce paz y armonía. efecto sonoro que acompaña la alegría al oír la voz de Leonor.

61
Unidad 9 >  El modernismo y la generación del 98

4.  Análisis del tema y de la estructura ¿Qué quiere recrear el poeta con esta contraposición? El poeta
recrea el rechazo a la realidad propia de los modernistas y su
a) Machado compuso el primer poema en Soria, y el segundo en refugio en ambientes exóticos y refinados.
Baeza, después de la muerte de Leonor. Se puede observar
cómo en el segundo el autor repite versos del primero, pero son  4. ¿Cómo se describe a la mujer? Describe a una mujer envuelta en
evocaciones de lo vivido, del pasado; el contraste con la reali- ropajes lujosos (abrigo de marta cibelina y falda de Alençon)
dad es duro y se refleja en los adjetivos utilizados. Destacan, que está descansando y adormilada, rodeada también por
sobre todo, los dos últimos versos: «voy caminando solo, / triste, objetos de lujo y exóticos (porcelana china, biombo del Ja-
cansado, pensativo y viejo.» pón). Se quiere transmitir un ambiente de sensualidad y exo-
tismo, de refinamiento, que tanto agrada a los artistas
b) El pasado y el presente, los recuerdos y la tristeza de su vida, el
modernistas. Para describir a la mujer se hace referencia a
deseo y la realidad se mezclan en el sueño del poeta. La resuel-
sus vestidos (abrigo, falda); al gato que está con ella, con su
ve con una llamada a la esperanza del encuentro.
rostro rosado y halagüeño, de mirar risueño; también se expli-
c) El poeta proyecta su tristeza en el paisaje: Poema A: «...tristeza / ca que descansa durmiendo.
tristeza que es amor...» (vv. 10-11). Poema B: «voy caminando
solo, / triste, cansado, pensativo y viejo» (vv. 13-14), «mi cora-  5. El poema consta de tres partes: la primera está formada por
zón está vagando en sueños» (v. 20). las dos primeras estrofas y el primer verso de la tercera. En
ellas se describe a Carolina y el ambiente de lujo en el que se
El recuerdo de Leonor predomina en los dos últimos poemas. Poema encuentra. La segunda parte la forman los dos versos finales
B: «¿No ves, Leonor, los álamos del río con sus ramajes yertos?» de la tercera estrofa y la cuarta estrofa en los que irrumpe el
(v. 7), «Dame tu mano y paseemos» (v. 10). Poema C: «Soñé poeta en el salón.
que tú me llevabas» (v. 1), «Sentí tu mano en la mía / tu mano
de compañera…» (vv. 7-9), «¡Eran tu voz y tu mano…» (v. 13).  6. Evasión: el salón lujoso y la chica refinada y somnolienta eva-
La realidad y el sueño están en conflicto en la vida del poeta. Poema den al poeta de la realidad gris y fría. Exotismo: el gato de ango-
B: «mi corazón está vagando en sueños» (v. 6). Poema C: «soñé ra blanco, las jarras de porcelana china, un biombo de seda del
que tu me llevabas» (v. 1), «¡Eran tu voz y tu mano, en sueños, Japón. Sensualidad: Carolina está envuelta con su abrigo de
tan verdaderas!...» (vv. 13-14). marta cibelina, el gato roza con su hocico la falda de Alençon,
el poeta va a besar su rostro, rosado y halagüeño como una
El pasado representa la felicidad y el presente, la tristeza y la me- rosa roja. Cosmopolitismo: cae la nieve del cielo de París.
lancolía: Poema B: «Mira el Moncayo azul y blanco; dame tu
mano y paseemos» (felicidad, vv. 9-10); «voy caminando solo /  7. Placidez: apelotonada (adjetivo), descansa (verbo), sillón
triste, cansado, pensativo y viejo» (tristeza, vv. 13-14). Poema (sustantivo), envuelta (adjetivo), abrigo (sustantivo), reclina
C: salvo los dos últimos versos, el poeta siente melancolía del (verbo), invade (verbo), sueño (sustantivo). / Delicadeza: bri-
pasado feliz junto a Leonor: «tu mano de compañera» (v. 12). lla (verbo), fino (adjetivo), angora (sustantivo), blanco (adjeti-
vo), porcelana (sustantivo), seda (sustantivo), sutiles
(adjetivo), dulce (adjetivo), flor (sustantivo).
5.  Comentario y opinión crítica
 8. Los recursos que utiliza son la aliteración («Medio apelotona-
Respuesta libre. da, descansa en el sillón…»), las imágenes («y en tanto cae la
nieve del cielo de París») y la cadencia rítmica que consigue
con la acentuación de la sílaba final aguda («rozando con su
EVALUACIÓN (pág. 217) hocico la falda de Alençon»).

 1. En un salón de exquisita elegancia y confort descansa Caroli-  9. Se trata de un soneto con versos alejandrinos con rimas con-
na. En la estancia entra sigilosamente un hombre, que se sonantes alternantes muy sonoras (Carolina, sillón, cibelina,
acerca a la chica para darle un beso. Ella abre los ojos y pare- salón…), en los que se combinan sílabas átonas y tónicas
ce como si se detuviera el tiempo, creando un contraste con para aportar musicalidad («El fino angora blanco junto a ella
lo que sucede fuera. se reclina…»).
El esquema métrico es: 14A 14B 14A 14B / 14A 14B 14A
 2. ¿Quién es la persona que entra en el salón? La persona que entra 14B / 14C 14D 14C / 14D 14C 14D
en el salón es el poeta.
¿Qué simboliza el abrigo gris? Simboliza la realidad que hay en 10. Aliteración: «Medio apelotonada, descansa en el sillón»; «El
el exterior, una ciudad fría y monótona. El poeta, al quitarse el fino angora blanco junto a ella se reclina»; «entro sin hacer
abrigo, se está despojando de toda la mediocridad del exte- ruido; dejo mi abrigo gris»; «Abre los ojos, mírame con su mirar
rior. risueño…». Metáfora: «con sus sutiles filtros la invade un dulce
sueño; que fuera flor de lis». Símil: «como una rosa roja que
fuera flor de lis». Pleonasmo: «mírame con su mirar risueño».
 3. ¿Cómo son los elementos que hay en el salón? El salón es un lugar
que rebosa elegancia y refinamiento. Los elementos que hay
11. La originalidad del libro Azul de Rubén Darío radica, entre
en él son lujosos y exquisitos: abrigo de marta cibelina, gato
otras cosas, en la influencia de elementos estéticos. Esto hace
de angora, jarras de porcelana china, biombo de seda del Ja-
que los poemas y cuentos estén enriquecidos con un amplio
pón…
vocabulario y variadas imágenes, lo que deriva en un particu-
¿Qué diferencias existen entre el salón y el exterior? El salón es lar estilo del texto.
un lugar acogedor, exótico, mientras que el exterior es frío y
mediocre. 12. Respuesta abierta.

62
10 # El novecentismo y el vanguardismo

liberal, las instituciones académicas y las fórmulas de ges-


EN CONTEXTO (pág. 219)
tión más modernas con la que querían equipararse. Se
a) En los años 20 París fue la ciudad preferida de los artistas y es- puede hablar, por tanto, de que son una generación euro-
critores de todo el mundo, atraídos por su hermosura y misterio. peísta ya que quieren trasladar a España las corrientes de
En el barrio de Montmatre había galerías, exposiciones y escue- pensamiento imperantes en Europa.
las de arte. Los nuevos artistas y movimientos artísticos, espe-
cialmente el vanguardista, encontraron en esta ciudad el  2. José Ortega y Gasset, catedrático de Filosofía, había estudiado
espacio ideal para relacionarse y desarrollar su creatividad. Po- en Alemania; Manuel Azaña, doctor en Derecho y funcionario
demos citar entre ellos a Picasso, Modigliani, Man Ray, Matisse del Estado, había estudiado en París; Fernando de los Ríos,
y Miró. Tras la Primera Guerra Mundial numerosos norteameri- había estudiado en la Institución Libre de Enseñanza, era doc-
canos también se dieron cita en la llamada Ciudad de la Luz, tor en Derecho y catedrático de universidad. También pertene-
donde encontraban mayor libertad. Uno de ellos fue la escritora cieron a esta generación los siguientes colaboradores de la
Gertrude Stein en cuya casa se reunían Hemingway, Ezra revista España dirigida por Ortega y Gasset: Ramón Pérez de
Pound y F. Scott Fitzgerald. Otros puntos de reunión de la inte- Ayala, Ricardo Gutiérrez Abascal, Margarita Nelke, Salvador
lectualidad eran los cafés de Le Dome y La Coupole, mientras de Madariaga, de profesión ingeniero, y Luis Araquistain. Ra-
que al Hotel Ritz acudía a inspirarse el ensayista y crítico fran- món Gómez de la Serna también formó parte de la generación
cés Marcel Proust. del 14, aunque de manera independiente, al círculo de Ortega.
En Barcelona, encontramos a Eugenio D’Ors y a Gabriel Miró.
b) El cuadro de Las señoritas de Avignon, de Pablo Picasso, modi-
También hubo un médico, Gregorio Marañón.
fica el modo de concebir la pintura por el tratamiento plano (bi-
dimensional) de las figuras y por el uso de puntos de vista
 3. En primer lugar, Ortega y Gasset nos dice que para vivir la vida
diferentes y simultáneos. Por ello se le considera un anteceden-
de manera verdadera y auténtica, cada persona debe hacer lo
te del cubismo.
que se espera de ella. Por eso, es necesario descubrir qué se
Gracias a la multiplicidad de planos, las figuras pueden verse espera de cada uno de nosotros y llevarlo a cabo; la circuns-
de frente, de perfil y de espaldas al mismo tiempo, rompien- tancia personal es la que nos ayuda a ver con más claridad el
do el sentido del volumen y la perspectiva tradicionales. Lo camino que debemos seguir.
vemos claramente en la mujer en cuclillas a la derecha del
a) Ortega y Gasset utiliza un símil extraído del ámbito de la litera-
cuadro.
tura para explicar el término circunstancia: es como un pie
Descompone la realidad en planos angulares sin fondo ni forzado (verso o rima fijados de antemano a los que debe
perspectiva espacial, con las formas destacadas por líneas atenerse una composición poética) que se le da a un poeta.
claroscuras.
b) Esta circunstancia no se puede cambiar ni evitar, es inelu-
c) Se refiere a su generación, la del 14, formada por José Ortega y dible. No es independiente de nuestro yo sino que va liga-
Gasset,  Eugenio d’Ors, Salvador de Madariaga, Ramón Pérez da a él y, por ello, es única y personal.
de Ayala, Juan Ramón Jiménez, Ramón y Cajal y él mismo.
Respuesta abierta. / La formación de un grupo se debe princi-  4. a) En el texto se exponen dos conceptos distintos de arte; uno,
palmente a que actúa como comunidad cronológica, es decir, del siglo xix, concebido como realista y popular; el otro, arte
que sus miembros tienen edades similares y están creativos en puro, que solo puede ser concebido por artistas.
unos años determinados. / Respuesta abierta.
b) El tema del fragmento es la contraposición entre el arte del
siglo xix y el de inicios de siglo xx.
ACTIVIDADES (págs. 227-229) Cabe destacar dos ideas principales: el arte del siglo xix redu-
ce al mínimo los elementos estrictamente estéticos, convierte
la obra en la ficción de realidades humanas y está destinado
El novecentismo o la generación del 14 (págs. 220-221) a la mayoría; el arte de principios del siglo xx es puro, o tiende
a la perfección, y se caracteriza porque elimina el contenido
 1. a) Los autores de la generación del 98 creían que para salir de humano de la obra y está destinado a minorías.
la depresión de la nación era necesario recuperar los au-
c) Respuesta libre.
ténticos valores del país, para ello, ahondan en la cultura y
las costumbres más españolas (casticismo).
b) Aunque las preocupaciones de los autores de la genera- La lírica novecentista: Juan Ramón Jiménez
ción del 98 son similares a los de la generación del 14 (págs. 222-223)
(deseo de regenerar España), estos últimos muestran una
actitud más optimista y vital, alejada del pesimismo y la
angustia que caracterizó a los noventayochistas.  5. Juan Ramón Jiménez da a entender que tiene una relación
muy estrecha con la poesía, pues plantea su relación con ella
c) Los novecentistas creían que Europa era la solución a los como si fueran dos amantes. De esta entrega en cuerpo y
problemas de España porque veían en ella la moderniza- alma a su vocación literaria, casi amorosa, es de donde nace
ción, la cultura y la investigación, así como la democracia la fuerza de su poesía.

63
Unidad 10 >  El novecentismo y el vanguardismo

 6. a)  Estrofa de versos octosílabos con rima asonante a-a en los ra de la vida como una película es propia del ultraísmo,
pares. Son coplillas de romance. de la fascinación técnica de este movimiento literario.
Además, el poeta refleja su gusto por el uso de la termi-
b) El tema es el sentimiento de melancolía ante su ausencia.
nología del cine (pantalla, reflectores). Poema C: perte-
Las ideas principales son: Él no volverá (1), no sabe si al-
nece al movimiento surrealista, sin métrica, ni rima. Las
guien se acordará de él (2), pero habrá otras cosas que
metáforas utilizadas proceden de la asociación libre de
seguirán estando presentes: las estrellas, las flores…
imágenes del poeta.
c) En Juan Ramón Jiménez, la influencia modernista se
b) El tema del poema A es la llamada a la creación de un
percibe en un léxico cuidado (suspiros, fragancias, pláci-
universo nuevo. Se trata de un texto creacionista por sus
da...) y una poesía de tono melancólico, delicado y sen-
referencias a la creación de un mundo nuevo: «levante-
sorial. Sin embargo, como Machado, Juan Ramón
mos de nuevo nuestros mundos», «En el principio
Jiménez desarrolla ya desde sus inicios un estilo muy
era…».
personal, que no puede encasillarse estrictamente en el
modernismo, aunque reciba influencias de este. El poe- c) El autor utiliza la poesía con una terminología técnica pro-
ma pertenece a su primera etapa, etapa sensitiva: poesía pia del cinematógrafo de la época. La vida aparece como
de forma sencilla con acento becqueriano, predominio de una representación.
octosílabos y asonancias...
d) Sucesión de imágenes surrealistas que no se pueden re-
ducir a términos racionales: «Para entrar en la sombra las
 7. «A mi alma» forma parte del libro Sonetos espirituales (1917)
lejanías se calzan las chinelas».
que pertenece a la etapa intelectual (1916-1936) del autor.
En ella, Juan Ramón Jiménez se da cuenta de que la verdad
que busca puede estar en su interior. En este poema el yo 11. Respuesta abierta.
poético hace interlocutora a su alma que, como para algunos
místicos, es vista como una entidad autónoma, no encadena- 12. La morcilla es un chorizo lúgubre: Término real: la morcilla;
da. Esta entidad sensible está siempre atenta a captar la per- término imagen: chorizo lúgubre; posible asociación: la mor-
fección, la belleza (Tu rosa será norma de las rosas, / tu oír de cilla tiene una forma parecida al chorizo, pero al ser de color
la armonía…) sobre la que el poeta no parece tener un domi- negro lo asocia al concepto «lúgubre». / El cometa es una es-
nio absoluto. Este poema es una muestra de que su segunda trella a la que se le ha deshecho el moño. Término real: come-
etapa tiene un carácter más metafísico, y que ha abandonado ta; término imagen: moño; posible asociación: la cola de un
el sentimentalismo. cometa se asemeja a un moño deshecho. / Las cucharillas
son los peces de las pequeñas peceras de los vasos. Término real:
 8. a) El poema describe las distintas etapas por las que pasa su las cucharillas; término imagen: los peces; asociación: cuan-
poesía. Así, podemos distinguir: vv 1-3: poesía inocente, do se remueven, las cucharillas en los vasos se parecen a
sencilla; vv 4-9: poesía vestida o modernista; vv 10-14: los peces en las peceras.
poesía que se va desnudando o depurando progresivamen-
te y vv 15-18: poesía desnuda. 13. El pez más difícil de pescar es el jabón dentro de la bañera. Térmi-
no real: el jabón; término imagen: el pez; asociación: el jabón
b) Combinación de versos de arte menor (heptasílabos) y arte
dentro de la bañera es difícil de coger y de ahí la asociación
mayor (de nueve y once).
con un pez. / En la noche acústica se oyen lejanos trenes que
c) El lenguaje es sobrio (supresión de todo ornamento, con- pasan diciendo: «que-te-cojo, que-te-cojo, que-te-cojo…». La últi-
centración conceptual, adjetivación matizada…). ma expresión recoge el traqueteo de los trenes en medio del
silencio de la noche. / Las gaviotas nacieron de los pañuelos que
d) Respuesta orientativa. Sí que podríamos decir que se trata,
dicen adiós en los puertos. Término real: las gaviotas. Término
en cierto sentido, de un poema amoroso, debido a la estre-
imagen: los pañuelos; posible asociación: la silueta de las ga-
cha relación que mantiene el poeta con su obra. En el
viotas se asemeja a la de los pañuelos agitados en las despe-
poema, Juan Ramón Jiménez se dirige a la poesía como si
didas en el puerto. / Monólogo significa el mono que habla solo.
fuera su amada, y unas veces le despierta amor: «y la amé
Juego de palabras por semejanza de sonidos.
como un niño», «creí de nuevo en ella», «pasión de mi
vida»; y otras, todo lo contrario: «y la fui odiando».

 9. a) El poeta se dirige a dios. Al dios creado en él y para él mismo. ANÁLISIS Y COMENTARIO DE TEXTOS
(pág. 230)
b) El poeta se muestra en un estado de total plenitud.
c) «sólito en lo uno mío».
1. Comprensión
El vanguardismo (págs. 224-225) a) El poeta observa una chica negra que lleva una rosa blan-
ca en la mano y describe la escena, así como los pensa-
10. a) Poema A: En el aspecto formal, se observa una ruptura mientos que le sugiere. A continuación explica cómo
con las formas poéticas anteriores, ya que carece de mé- reaccionan los pasajeros del ferrocarril subterráneo de
trica y rima. Desde el punto de vista del contenido, el Nueva York a los efectos y efluvios de la rosa portada por la
poeta propone la creación de un mundo nuevo («Haga- chica que duerme, pero la trata con exquisito cariño, como
mos Nuestro Génesis», v. 7). Poema B: Ya desde la dis- si fuera la madre de esa rosa.
posición tipográfica, rompe con la tradición poética. La
medida y la disposición de los versos no responde a es- b) Se refiere al ferrocarril metropolitano o metro de Nueva York.
tructuras clásicas, sino a la libertad del poeta. La metáfo- c) La chica con la rosa blanca simboliza un mundo ideal posible.

64
Unidad 10 >  El novecentismo y el vanguardismo

2. Contextualización madre del alma…», «cual si fuese de oro», «como en


una pesadilla de cansancio y de tristeza…», «como la
a) El texto pertenece a la etapa intelectual de la trayectoria conciencia del subterráneo».
poética del autor (1916-1936).
b) En esta etapa Juan Ramón Jiménez observa el mundo con —— Apóstrofe de admiración: «¡Cómo la lleva!».
una nueva mirada, yendo «más allá» de las cosas, como
vemos en esta prosa poética del Diario de un poeta recién
casado. Los símbolos (la muchacha negra con la rosa) ya
4.  Análisis del tema y de la estructura
no se limitan a describir la realidad, sino que adquieren un a) El contenido del texto tiene dos partes:
nuevo significado, cuyo mensaje hay que descifrar (un
mundo ideal posible): «La negra va dormida, con una rosa —— Los dos primeros párrafos muestran la escena en la
blanca en la mano…». Sus composiciones se van depu- que se describe a la chica negra con la rosa blanca en
rando de todo artificio superfluo (ornamentación, senti- la mano.
mentalismo, color, musicalidad…) «presencia inmaterial», —— El tercer párrafo es una reflexión. La mirada del poeta se
«eternidad», «huele un punto a rosa blanca». extiende a todo el vagón y se observan los efectos trans-
c) El mundo subterráneo significa la suciedad, la oscuridad, formadores de la rosa sobre los pasajeros. En esta parte
la contaminación y la violencia de la ciudad de Nueva predomina una atmósfera poética y transcendente.
York, que se oponen a la belleza y a la armonía del mundo.
b) Los textos en cursiva ayudan al poeta a establecer una di-
El poeta siente añoranza del mundo natural y de su belle-
ferencia entre la parte descriptiva y otra parte emocional.
za, y reconoce que en el mundo subterráneo y hostil pue-
de existir la belleza. c) Todo el mundo de perfección de la rosa blanca alude a
una perfección eterna.

3. Expresión d) Respuesta abierta.


a) La técnica de Juan Ramón Jiménez depura la poesía de
todo artificio superfluo, eliminando todo lo circunstancial
5.  Comentario y opinión crítica
para hacerla más breve y concisa. Todo es más sencillo y
elemental, más espontáneo. Por eso se la llama poesía a) Respuesta abierta.
pura o desnuda.
b) En este poema en prosa los colores negro y blanco son
b) El texto puede inscribirse en el género literario de la prosa símbolos de belleza, pues reflejan el deseo de armonía
poética o poema en prosa. entre opuestos aparentes.
c) Describe de forma poética una escena cotidiana. Los diarios
c) Respuesta abierta.
y libros de viajes recogen, por tanto, las vivencias e impre-
siones diarias del viajero o de la persona que lleva un diario. d) El proceso interior empleado por Juan Ramón Jiménez es el
d) El contraste es esencial en todo el poema. La realidad invi- de despertar la propia conciencia de manera que nos permita
sible anda por todo el subterráneo. El contraste está pre- ver la esencia que se esconde tras la forma aparente de las
sente entre todo lo que sugiere la rosa blanca, que huele cosas; esta esencia es bella de por sí y gracias a ella podemos
también a primavera mejor y a eternidad, y el ambiente del transformar toda la fealdad que nos rodea. El poeta sueña
subterráneo: estrepitoso, rechinante, negro, sucio y cálido, una realidad nueva muy alejada de la fealdad aparente.
con el hierro, el carbón, los periódicos. También se esta-
blece un contraste entre la rosa y el sueño y el triste atavío
de la muchacha. EVALUACIÓN (pág. 233)
e) El lenguaje empleado es sencillo. Predominan los adjetivos
cromáticos (negra, blanca, rosa, verde, oro, moradas), así  1. La originalidad en la distribución del texto, que crea la imagen
como los morales o valorativos (májica, triste, indefensa, de una campana en movimiento. / La ausencia de signos de
inconciente, pobre, invisible, atento, delicada, bella, inma- puntuación para que el lector vaya situándolos. / El contenido
terial). Predominan los nombres, que se utilizan para alu- poético, que corresponde al imaginario creacionista del autor
dir a las cosas con las que el alma del poeta toma contacto y que el lector deberá recrear.
(rosa, sueño, superposición, atavío, seguridad, delicadeza,
madre, alma, pecho, hombro, honor…).  2. El ángelus está sentado en la campana, como sobre un co-
lumpio, esperando que el sacristán lo despierte con el toque
Alguno de los recursos empleados son: de campanas.
—— Enumeración de términos: «una bella presencia inma-
terial que se va adueñando de todo, hasta que el hie-  3. Aquí se trata del toque de campana del atardecer, cuando se
rro, el carbón, los periódicos, todo, huele un punto a pone el sol. Esto viene sugerido por las siguientes expresiones
rosa blanca, a primavera mejor, a eternidad…». del texto: «Enmudecen los astros y los frutos», que indica esa
—— Sinestesias: «todo huele a eternidad», «el silencio atento». hora del día en que todo está en silencio en el firmamento y la
naturaleza; «Con su guadaña ensangrentada un segador can-
—— Personificaciones: «La rosa y el sueño apartan…», «Inde- tando se alejaba», porque ya han finalizado las faenas del
fensa con el sueño, se sonríe, la rosa blanca…», «Una campo y los trabajadores van camino del hogar.
realidad invisible anda por todo el subterráneo…»).
—— Comparaciones: «como si esta mañana la hubiera  4. Surtidores: que los trabajadores regresan sudorosos. Delfines
dado ella a luz», «como si ella se sintiera, en sueños, líricos: «Delfines» porque van mojados y «líricos» porque van

65
Unidad 10 >  El novecentismo y el vanguardismo

cantando. Ríos al hombro. El hombre lleva a sus espaldas del sonido «i» («fin», «jardín», «olvido») y la rima en «a» («en-
(«hombro») su vida («ríos»). El cielo es un jardín. Se emplea sangrentada», «alejada») nos evocan el sonar de las campa-
el recurso del tópico literario del locus amoenus para repre- nas (poema musical). Todo ello converge con el título y queda
sentar aquí el cielo como un lugar ideal para el descanso feliz. recreado en el poema.
Guadaña ensangrentada. La guadaña, herramienta empleada
en el campo para segar a ras de tierra, también se ha emplea-  9. El ángelus dormita encaramado en el columpio a la espera de
do para representar a la muerte. Aquí, la palabra «ensangren- que llegue el momento de la oración y el sacristán toque la
tada» se podría interpretar como que la guadaña está sucia campana al atardecer. El sol se pone, van apareciendo las
porque ha estado segando las mieses todo el día. sombras de la noche. Todo queda en silencio en el firmamen-
to cósmico y en la naturaleza. Los hombres sudorosos, heri-
 5. Se dirige a los hombres, segadores, campesinos, que trabajan dos por los rayos del sol, dejan el duro trabajo mientras van
en el campo, y, en general, a todas las gentes del pueblo, invi- cantando. Otros siguen trabajando con su vida pesarosa sobre
tándolos a rezar. los hombros sin descansar. La vida va transcurriendo como si
no tuviera fin. Los días van pasando, seguimos trabajando, sin
 6. La vida de las personas es única. Cada persona va creando su pensar en que un día todo llegará a su fin y nos espera un
vida día a día, igual que hace la naturaleza con los árboles. paraíso, un jardín de felicidad.
 7. Esta palabra representa el eco (cierre) del último son de la Terminado el toque de campanas, también termina la ora-
campana. La oración ha acabado y empieza un tiempo de si- ción. Silencio, todo se olvida. Pero en medio de ese silencio
lencio y de descanso. se escucha, aún, la canción de un segador que ha pasado el
día segando (cortando las mieses de trigo) y se dirige a su
 8. El autor emplea diferentes recursos para reforzar el significa- hogar.
do del poema: la llamada a la oración del Ángelus.
La disposición tipográfica de los versos dibuja una campana, 10. Tanto en el poema de Gerardo Diego como en el cuadro de
que ayuda significativamente a comprender el poema. Ello Millet se representa una escena campesina: el rezo del Ánge-
produce un doble efecto: el toque de la campana nos evoca lus. En ambos casos los campesinos oyen las campanas y
su sonido. Los sonidos vocálicos de las palabras: repetición dejan el trabajo que estaban realizando para rezar.

66
11 # La lírica del 27

lizan el pasado son: la arboleda perdida, que se repite a lo largo


EN CONTEXTO (pág. 235)
de la obra del poeta y simboliza su propia vida; la expresión Dejé
a) Además de los autores que participaban desde el principio en la un temblor, dejé una sacudida, un resplandor de fuegos no
publicación de la revista (Federico García Lorca, José Berga- apagados (vv. 5 y 6), que simboliza la juventud del poeta; y pa-
mín, Jorge Guillén, Gerardo Diego) en la edición conmemorativa lomas tristes junto al río podrían ser una metáfora de la juventud
del tercer centenario de la muerte de Luis de Góngora, también perdida y de las vivencias y amistades que dejó (v. 9).
participaron Manuel de Falla, Juan Gris, Pablo Picasso, Salva- Analiza las formas verbales y explica… El poema está escrito en
dor Dalí, Benjamín Palencia, Bores, Joaquín Peinado, Gregorio pretérito perfecto simple. El poeta utiliza la anáfora dejé para
Prieto, Cossio... subrayar tantas cosas que tuvo que abandonar. En el verso
b) Varios de los autores de la generación del 27 se exiliaron tras 13, utiliza el imperativo (Dame tú): el poeta pide a la ciudad
finalizar la guerra civil: Rafael Alberti, Emilio Prados y Luis Cer- de Roma que durante su exilio allí le ayude a no sentir tanto
nuda. Otros permanecieron en España: Vicente Aleixandre, Dá- dolor por estar lejos de su país.
maso Alonso y Gerardo Diego. Federico García Lorca, sin
En qué etapa… En la etapa del exilio.
embargo, murió fusilado.
 5. Muestra qué temas tratan… Poema A: el amor, un amor malo-
grado. Poema B: la desolación humana. Poema C: tiene como
ACTIVIDADES (págs. 248-251) tema el paso del tiempo y la nostalgia.
El poema A, una composición de pentasílabos y octosílabos,
La generación poética del 27  (págs. 236-238) tiene influencia de la tradición lírica popular española. Es un
poema de arte menor con rima consonante en todos los versos,
 1. Las composiciones A y C utilizan formas tradicionales. El poe- menos en el tercero y en el cuarto, que tienen rima asonante.
ma A tiene versos heptasílabos, con rima asonante en los El poema B recibe la influencia de la lírica culta tradicional, por
versos pares. El poema C es un soneto: 11A 11B 11B 11A el uso de versos endecasílabos y heptasílabos. El poema C no
11A 11B 11B 11A 11C 11D 11E 11C 11D 11E. respeta ninguna forma de la métrica tradicional.
Solo el poema B no presenta una estructura métrica clásica.
No tiene rima, y la medida de sus versos es irregular. Este Poetas principales  (pags. 239-245)
poema se inscribe en las formas vanguardistas utilizadas por
la generación del 27.  6. a) «Quítate ya los trajes, / las señas, los retratos; / ya no te
quiero así, / disfrazada de otra, / hija siempre de algo» (vv.
 2. Explica con tus palabras… Un canto a la plenitud del día. 5-9), «Te quiero pura, libre» (v. 10), «solo tú serás tú» (v.
Extrae palabras y expresiones que muestren… «¡Todo ya pleno!» 15), «enterraré los nombres, los rótulos, la historia» (vv.
(v. 1), «quien lo veía todo completo para un dios» (vv. 17-18), 19-20), «lo que encima me echaron» (v. 22), «al anónimo
«¡Las doce en el reloj!» (vv. 19-20). eterno del desnudo» (v. 25).
b) La sencillez del poema se encuentra en el uso de palabras
 3. Indica B… Desde el inicio vemos que el
por qué en el texto
y expresiones inteligibles para cualquier lector: «Sé que
poeta habla de su vida, recuerda su pasado y reflexiona sobre cuando te llame / entre todas las gentes» (vv. 12-13); «Iré
su existencia. rompiendo todo / lo que encima me echaron» (vv. 21-
¿En qué consiste vivir… Buscar el amor, ir detrás de él hasta 22)…; también se aprecia una intensidad expresiva: «yo
encontrarlo. ¿Qué no es vivir? «dormir», «suspirar o presentir no te quiero así» (v. 7); «Te quiero pura, libre, / irreducti-
palabras que aún nos vivan» (v. 3). ble: tú» (vv. 10-11), «enterraré los nombres, / los rótulos, la
historia» (vv. 19-20), «Yo te quiero, soy yo», (v. 28)…; otro
¿Con quién habla exactamente? El poeta habla consigo mismo, rasgo importante es el uso de imágenes insólitas: «vivir en
con su pasado y su presente. los pronombres» (v. 4); «lo que encima me echaron / des-
Analiza el estilo del poema e indica… El poeta utiliza un léxico de antes de nacer» (vv. 22-23), «anónimo / eterno del
sencillo, con libertad en el ritmo y en el verso, pero con imáge- desnudo» (vv. 24-25).
nes simbólicas que pertenecen a un universo propio. Ese esti- c) El poema está compuesto por versos heptasílabos, en los
lo corresponde a su último período creativo, a la llamada que hay alguna asonancia, excepto en los dos versos cor-
«etapa de meditación». tos, el 14 (trisílabo) y el 27 (tetrasílabo), que rematan las
dos partes fundamentales del poema. Salinas da un cierto
 4. Señala cuál es el tema del poema C… El texto refleja la nostalgia ritmo al poema con la disposición de los pronombres a lo
que el poeta, en el exilio, siente de su país. Como los poetas largo del texto.
de la generación del 27, Alberti evoca su vida anterior, de la
que se ve apartado a causa de su exilio político y pide ser re-  7. a) Jorge Guillén da testimonio en el poema «Los intranquilos»
compensado por la belleza de Roma. del desorden, la inseguridad y la vaciedad de la vida: «Así
Indica de qué siente nostalgia… Alberti siente nostalgia de la vida vivimos sin saber / Si el aire es nuestro» (vv. 13-14), «Qui-
en su Andalucía natal. En este poema, las metáforas que simbo- zá muramos en la calle / Quizá en el lecho» (vv. 15-16), y

67
Unidad 11 >  La lírica del 27

clama contra el mal y la guerra: «El mañana asoma entre ojos», «lento se deshace», «Movido suavemente», «desvaído
nubes / De un cielo turbio» (vv. 5-6), «Estamos siempre a sueño», «las hojas van cayendo».
la merced / De una cruzada», (vv. 9-10). Al final, sin em-
c) La senectud del poeta se expresa mediante la imagen de
bargo, abre una ventana a la felicidad que dan las cosas
las hojas doradas otoñales que van cayendo al suelo: «sue-
sencillas aunque sea una postura vacía ante la vida:
ños de oros vivos».
«Somos entre tanto felices. […] ¡Televisión!» (vv. 17-20).
d) El viento simboliza la realidad intangible de la vida que va
¿Qué diferencias encuentras…? La diferencia de tono es
deshaciéndose.
evidente. Mientras que en el poema perteneciente a Cla-
mor predomina la queja, el malestar y la vaciedad; los ver-
 9. a) La amada dormida provoca una desazón o desasosiego, ya
sos de Cántico muestran entusiasmo hacia el mundo y la
que al estar dormida no puede comunicarse y se ha aleja-
vida, disfrutando de la belleza de la naturaleza.
do de él.
b) malestar (vv. 1, 4, 5-6, 11-12 ), inseguridad (vv. 9, 13-16),
b) El sueño de la amada le hace sentirse como en una cárcel
felicidad (vv. 17-20).
de la que no puede salir.
c) El tema del poema es el malestar y la inseguridad en la
El último terceto cierra el poema… Frustración del poeta
vida, amenazada por las guerras y el mal. Para expresar
porque no puede entrar de ningún modo en el sueño de la
esta sensación el poeta emplea imágenes como «el maña-
amada.
na que asoma entre un cielo turbio» (vv. 5-8), «la cruza-
da», evocación de las guerras, que amenazan (vv. 9-10), c) El poema es un soneto que consta de dos cuartetos y dos
no sabemos si el aire es nuestro (vv. 13-14) y no sabemos tercetos de versos endecasílabos con rima consonante
si moriremos en la calle o en la cama (vv. 15-16). 11A 11B 11B 11A; 11A 11B 11B 11A; 11C 11C 11D; 11E
11D 11E.
d) El poeta logra superar esa situación evadiéndose, distra-
yéndose en los bares y viendo la televisión, aunque puede
10. a) La composición solo presenta rima interna en la 1.ª estrofa
intuirse la insatisfacción que le produce todo esto.
(«muerto»…«perros»), y la métrica no corresponde a una
e) Al usar la primera persona del plural, Jorge Guillén universa- composición clásica: 11 («müertos») - 11- 11- 11-; 14-
liza sus sentimientos y los hace comunes al de todos los 14- 14- 14- 14- 14-14- 14- 14. La segunda y la tercera
hombres. De este modo rehúye el tono personal que hubie- estrofa están compuestas por versos alejandrinos sin rima.
ra conseguido con el uso de la primera persona del singular.
b) El poeta entiende la muerte como algo definitivo (para
¿A quién se está refiriendo? Se refiere, por tanto, a la socie- siempre) que significa olvido (como todos los muertos que
dad entera en la que está inmerso. se olvidan). Para evitar el olvido, rememora la figura del
torero (yo canto) y utiliza la poesía como eterno testimonio
f) En el poema de Jorge Guillén se advierte una influencia de
de su vida.
la poesía de Juan Ramón Jiménez en la ausencia de orna-
mentación, el lenguaje al servicio de la idea (poesía pura), c) El sentimiento de duelo por la muerte de un personaje no-
la emoción contenida, los metros cortos y la falta de rima. table, o público, o un ser querido; en este caso, el torero
muerto, amigo del poeta.
g) El poema está compuesto por estrofas de cuatro versos en
los que riman los pares. La presencia de ritmo se produce
11. a) «ángel de luz, ardiendo» (v. 1), «ángel de las tinieblas» (v.
a través de la aparición en diferentes momentos del poema
4), «chispas múltiples» (v. 6), «alas sin plumas» (v. 8),
de las formas verbales en la primera persona del plural.
«Luzbel de las canteras sin auroras» (v. 13), «pozos sin
h) Los rasgos vanguardistas se observan en la mención de agua, simas sin sueño» (vv. 13-14), «sol, luna» (v. 17).
elementos futuristas o modernos (arcángeles-átomos, tele-
b) La acumulación de imágenes visionarias.
visión) y en el uso de una frase en inglés: Seven-o’clock.
12. a) Bécquer dice en los versos 13-16 de la rima LXVI: «en
 8. a) Este poema pertenece a la etapa de meditación. El poeta,
donde esté una piedra solitaria / sin inscripción alguna, /
en la vejez, se comunica consigo mismo en poemas filosó-
donde habite el olvido, / allí estará mi tumba», muy pareci-
ficos en los que emplea imágenes y símbolos de su etapa
do a lo que escribe Cernuda: «Donde habite el olvido […] /
surrealista. Por ejemplo: imágenes oníricas: «Las
Memoria de una piedra sepultada entre ortigas / […] don-
sorprendidas ánimas te miran / cuando no pasas» (vv. 2-3)
de mi nombre deje». La influencia, por tanto, es notoria.
o «fenecidas hojas caen y vuelven / a caer» (vv. 5-6).
b) Los rasgos surrealistas que se observan en el poema son el
También aparecen referencias a la vejez, simbolizada por
uso de imágenes oníricas (procedentes de los sueños); por
las hojas otoñales que caen al suelo: «mientras la sobria
ejemplo: «vastos jardines sin aurora» (v. 2), «En esa gran
tierra las espera, / abierta» (vv. 7-8), «un desvaído sueño
región donde el amor, ángel terrible, / No esconda como
de oros vivos / se esparce» (vv.12-13).
acero / En mi pecho su ala» (vv. 9-11). La falta de rima y de
b) El poeta se dirige a sí mismo, pero mantiene un distancia- regularidad métrica son propias del surrealismo poético,
m i e n t o e m p l e a n d o l a s e g u n d a p e r s o n a : « Tu aunque al final puede tener una intención de rima asonante.
pensamiento…» (v. 4). Excepto en el verso inicial que em-
c) Quiere sumirse en el olvido para librarse del deseo amoro-
plea la tercera persona: «Nació y no supo…» (v. 1).
so y sus consecuencias, y del amor imposible.
¿Qué tono emplea? El poeta emplea un tono de resignación
d) Son versos libres, es decir, no hay métrica regular y se ob-
callada, de observación impasible ante la llegada de la vejez
serva cierta intención de rima al final. Los elementos tradi-
y de cierta frustración: «el viento», «a solas cae despacio»,
cionales que se observan son el uso de la anáfora
«las fenecidas hojas caen», «Callado el corazón, mudos los

68
Unidad 11 >  La lírica del 27

(«Donde…»), una anadiplosis («Disuelto en niebla, ausen- mina la poesía amorosa, y donde Hernández añadió este
cia, / Ausencia leve como carne de niño…»), y las estrofas poema, compuesto al morir su entrañable amigo Ramón Sijé.
son tercetos, quintetos y dos versos sueltos, que desarrolla
cada uno una idea. b) En dicha etapa los temas que predominan son la experiencia amo-
rosa del poeta –sentida como un anhelo insatisfecho–, la angustia
existencial. En las composiciones, Miguel Hernández emplea pre-
Miguel Hernández: un epígono del 27  (pág. 246) ferentemente dos imágenes que expresan su estado anímico: el
toro y el cuchillo como presentimiento trágico de la muerte.
13. a) El niño que se amamanta con sangre de cebolla, la mujer
que se derrama hilo a hilo en la cuna, el niño que se traga c) Ramón Sijé y Miguel Hernández eran amigos. Ambos vivían en
la luna, la doble luna del pecho. Orihuela y compartían su afición por la literatura. Ramón Sije
admiraba la capacidad creativa de su amigo y le ayudó a intro-
b) El poeta clama contra las penurias que pasan su mujer y
ducirse en los círculos intelectuales de Madrid.
su hijo, que en plena posguerra solo se alimentan de cebo-
llas. La sonrisa de su hijo, sin embargo, es el antídoto a d) Miguel Hernández estaba muy unido a la naturaleza y la cono-
tanta tristeza y soledad que vive en la cárcel y por eso se cía perfectamente. De ahí las continuas alusiones a elementos
agarra a ello como tabla de salvación. que forman parte de ella: «lluvias», «caracolas», «amapolas»,
«rastrojos», «escarbar». En el poema, Miguel Hernández utiliza
14. a) Gobiernas: veletas; bronces: poéticamente significa cam- la naturaleza como una plataforma para establecer un vínculo
pana; ronces: manifestaciones de cariño; bridas: freno de con su amigo, ya que él se cree el hortelano del huerto en el que
las riendas y el correaje del caballo. está enterrado Ramón Sijé.
b) La influencia del conceptismo barroco se observa en las me-
táforas que utiliza. Las lunas son metáforas de las gitanas, 3.  Expresión
que van de un lado a otro, pues son nómadas («Como gobier-
nas, […]» (v. 1); «siempre en mudanza, siempre dando a) La elegía es una composición lírica en que se lamenta la muerte
vueltas.», «las veo desfilar, libres…» (v. 2); «Domesticando de una persona o cualquier otro infortunio.
van mimbres» (v. 5); «estas lunas que esgrimen, […] / las
dentaduras» (vv. 7-8). b) Miguel Hernández utiliza en esta composición tercetos encade-
nados, es decir, estrofas de tres versos endecasílabos con rima
c) Se trata de una octava real, propia de la lírica culta del consonante encadenada. La estrofa que cierra el poema es un
Barroco, concretamente de Luis de Góngora. La octava serventesio.
real consta de ocho versos endecasílabos con rima conso-
nante distribuida 11A 11B 11A 11B 11A 11B 11C 11C. c) El hortelano de la tierra que su amigo enterrado abona (vv. 1-5);
el corazón del amigo es alimento de las amapolas (vv. 6-7); la
muerte es como un manotazo, un golpe helado, un hachazo, un
ANÁLISIS Y COMENTARIO DE TEXTO empujón brutal (vv. 10-12).
(pág. 252)
d) Con la metáfora «terciopelo ajado» se refiere al corazón sin vida
de su amigo.
1. Comprensión
e) Usa la hipérbole para expresar su dolor cuando dice «por doler
a) El dolor que causa en el poeta la muerte inesperada de un ami- me duele hasta el aliento» (v. 9); «No hay extensión más grande
go y el requerimiento de encontrarse con él. que mi herida» (v. 13); «no perdono ni a la tierra ni a la nada»
b) Aunque el deseo del poeta es transmitir el dolor que siente por (v. 24); «Quiero escarbar la tierra con los dientes» (v. 28).
la muerte de Ramón Sijé, este sentimiento se va modulando a lo
largo del poema del siguiente modo:
—— Sentimiento desgarrador del dolor representado en el horte-
4.  Análisis del tema y de la estructura
lano: «Yo quiero ser llorando» (v. 1), «Tanto dolor» (v. 8), a) El poema está compuesto por cinco partes:
«lloro mi desventura» (v. 14), «ni consuelo» (v. 17).
—— Imprecación a la muerte: «No perdono a la muerte» (v. 22), —— Hasta el verso 9: el poeta quiere ser un hortelano para estar
«En mis manos levanto una tormenta» (v. 25), «Quiero es- en contacto con su amigo.
carbar la tierra con los dientes» (v. 28).
—— Del verso 10 al 18: sentimiento de dolor por la pérdida de su
—— Añoranza y requerimiento de encontrarse con su amigo: amigo Ramón Sijé.
«Volverás a mi huerto y a mi higuera» (v. 34), «Alegrarás la
sombra de mis cejas» (v. 40), «que tenemos que hablar de —— Del verso 19 al 27: imprecación que Miguel Hernández le
muchas cosas» (v. 48). hace a la muerte temprana de su amigo.
c) El deseo final del autor es que se produzca un futuro reencuen- —— Del verso 34 al 45: esperanza de que su amigo vuelva a la
tro con su amigo en «las aladas almas de las rosas del almen- naturaleza.
dro» porque todo ha brotado y allí se encuentra con él.
—— Del verso 46 al 49: cierre del poema con el requerimiento
de encontrarse con su amigo.
2. Contextualización
a) Esta elegía pertenece a su etapa de plenitud poética, concreta- b) La vida: «y siento más tu muerte que mi vida» (v. 15); «no per-
mente al libro El rayo que no cesa (1934-1935) en el que predo- dono a la vida desatenta» (v. 23). El amor: «Volverás al arrullo

69
Unidad 11 >  La lírica del 27

de las rejas / de los enamorados labradores» (vv. 38-39); «y tu  6. En el poema pueden distinguirse dos tipos de errores:
sangre se irán a cada lado / disputando tu novia y las abejas»
(vv. 41-42). La muerte: «Temprano levantó la muerta al vuelo» —— Confusiones entre dos cosas que se parecen: «trigo» y
(v. 19); «No perdono a la muerte enamorada» (v. 22); «y besarte «agua», «mar» y «cielo», «estrellas» y «rocío».
la noble calavera» (v. 32). —— Errores totales, que contraponen términos inconfundi-
bles: «norte» y «sur», «noche» y «mañana», «calor» y
c) El poeta impreca a la muerte de los versos 19-27: «Temprano «nevada».
levantó la muerte el vuelo» hasta «no perdono a la tierra ni a la
nada».  7. El contenido político-social se asocia a la situación del
d) El momento de máxima intensidad del poema se produce de mundo en pleno siglo xx, inmerso en graves conflictos. Di-
la estrofa octava a la onceava (vv. 22-33) cuando el poeta ex- cha confusión también le afecta a él, exiliado tras la guerra
presa su mayor rabia diciendo que no perdona y acto seguido civil.
se lanza a desenterrar el cadáver de su amigo. Los términos
que ayudan a lograr esta tensión son las estrofas de estructura  8. Paloma: simbólicamente puede representar la paz, mensajera
anafórica encabezadas por «No perdono» (vv. 22-24) y la des- de la paz, donde aparece transportando una ramita de olivo
cripción de la furia concentrada en sus manos y sus dientes en el pico; Norte y Sur: el Norte puede asociarse con el frío
(vv. 25-30), furia que culmina con el beso a la calavera de y la oscuridad, mientras que el sur simboliza la luz y el calor;
Ramón Sijé (v. 32). mar y cielo: el mar es un abismo, el caos, mientras que el
cielo es el lugar de lo divino y lo sagrado; corazón y casa: el
corazón se relaciona con los sentimientos y las pasiones, y la
5.  Comentario y opinión crítica casa representa el hogar, la familia.
a) Respuesta abierta.
 9. Los versos segundo y tercero de los trípticos riman en asonan-
b) Respuesta abierta. te. Igualmente, el segundo verso del díptico inicial y final rima
con el segundo y tercero de los trípticos. El metro corto y la
asonancia recuerda la poesía popular.
EVALUACIÓN (pág. 255)
10. epífora o repetición de las mismas palabras al final de frases o
 1. Rafael Alberti nos habla de una paloma que se equivoca versos: «Se equivocaba» (vv. 2, 5, 8, 11, 14); antítesis (vv. 3,
constantemente en sus acciones, interpretando todas las co- 6, 7, 9, 10, 12, 13) y anáfora: «Se…, se…» (vv. 1-2); «Que…,
sas al revés. que…» (vv. 12-13).

 2. La primera parte corresponde a la primera estrofa, de dos Comenta la función que realizan en el texto: todas las figuras
versos, donde se presenta el asunto del poema: la paloma se literarias reiteran la idea de errores cometidos por la paloma.
equivocaba.
11. Explica la evolución…: La trayectoria poética de Rafael Alberti
La segunda parte está constituida por las cuatro siguientes pasa por las siguientes etapas:
estrofas, todas de tres versos, en las que se explican cuáles
fueron los errores de la paloma. —— Poesía neopopular y neotradicional de sus primeras obras,
en las que recrea las formas y los temas de la tradición
La tercera parte lo ocupa la última estrofa, también de dos española, y se acerca también a las vanguardias.
versos, en que, a modo de conclusión, se muestran las conse-
cuencias de su error. —— Poesía culta y vanguardista, que le permite expresar todas
sus contradicciones internas.
 3. La paloma, tal vez, puede representar a la humanidad, a la —— Poesía comprometida. Abandono de los temas de carácter
patria, desconcertadas en un momento de entreguerras; a la personal para tratar asuntos político-sociales de la guerra
paz, que no se sabe hacia dónde va, incluso también podría civil con un tono combativo y de protesta.
referirse a la amada, etc.
—— Poesía del exilio, última etapa en que son frecuentes la
 4. Al tratarse de esos signos externos, parece que se está refi- añoranza de la patria perdida y el rechazo del fascismo.
riendo a una mujer, quizás a su amada. Con los años, Alberti fue cicatrizando las heridas de la
guerra y cultiva una poesía personalizada en la que la in-
 5. «Por ir al Norte, fue al Sur. / Creyó que el trigo era agua» (vv. 3-4); fluencia vanguardista, la poesía político-social y el humor
«Creyó que el mar era el cielo; / que la noche, la mañana» (vv. están vigentes. En ella combina formas tradicionales con
6-7); «Que las estrellas, rocío; que el calor, la nevada» (vv. 9-10). otras innovadoras.

70
12 # El teatro de principios de siglo xx

des de las clases medias-altas. En esta obra, Benavente,


EN CONTEXTO (pág. 257)
valiéndose de personajes arquetípicos de la comedia del arte
a) ¿Qué tipo de teatro critica? Federico García Lorca critica el italiana, realiza una sátira universal sobre el poder del dinero.
teatro mal orientado, que achabacana y adormece al público.
 3. Serafín y Joaquín Álvarez Quintero: El traje de luces (1989) y El
¿Cuál debe ser el papel social de las obras dramáticas? El pa- patio (1900). / Carlos Arniches: La señorita de Trevélez (1916).
pel social de las obras dramáticas debe ser el de recoger el / Pedro Muñoz Seca: La venganza de don Mendo (1918).
latido social e histórico del pueblo, el drama de la gente y
el color genuino de su paisaje y de su espíritu.  4. a) Salvador, señor de Montoya, quiere iniciar una relación amoro-
¿Qué medio de comunicación cumpliría hoy en día esa función? sa con Araceli, pero ella, a pesar de su insistencia, lo rechaza.
El medio de comunicación que hoy cumpliría esa función b) Son versos octosílabos entrelazados. La rima es asonante
es la televisión. en los versos pares.
b) Al entrar en la librería de Zaratustra, Max Estrella dice:
«Mal Polonia recibe a un extranjero». Es una adaptación El teatro renovador  (págs. 260-263)
del verso «Mal, Polonia, recibes a un extranjero», que pro-
nuncia Rosaura en la escena II de La vida es sueño, de  5. Bertolt Brecht (teatro épico): La ópera de los tres centavos,
Calderón de la Barca. Madre Coraje y sus hijos, El resistible ascenso de Arturo Ui, El
círculo de tiza caucasiano.
Otras referencias a la literatura en la escena II son: Don
Gay, al entrar en la librería, exclama: «Salutemplúriman», Luigi Pirandello (teatro de reflexión): El placer de la honradez,
que procede de la fórmula romana de saludo «Salutemplu- Seis personajes en busca de autor, Cada uno a su manera,
rimamdicit». Durante la conversación, don Gay menciona Así es (si así os parece), Enrique IV.
el Palmerín de Constantinopla, una obra ficticia que hace Antonin Artaud (teatro de la crueldad): Los Cenci, Heliogábalo
referencia a la serie de libros de caballerías conocida como
Los Palmerines. Al final de la escena, una niña pregunta a o el anarquista coronado.
Zaratustra si ha salido la entrega semanal de El hijo de la
 6. En la mitología griega, Fedra era una princesa cretense, hija
difunta, y si sabe si se ha casado Alfredo. La conversación
de Minos y de Pasífae, y hermana de Ariadna.
entre Zaratustra y la niña pone de relieve la existencia de
los folletines, que se vendían por entregas. Zaratustra le Teseo, mítico rey de Atenas, la raptó para casarse con ella.
responde: «Es un secreto lo que hacen los personajes de Durante la boda fueron atacados por las amazonas, pues Te-
las novelas. Sobre todo en punto de muertes y casamien- seo había abandonado a la amazona Antíope, con quien ha-
tos. […] Estaría bueno que se divulgase el misterio. Pues bía tenido un hijo llamado Hipólito.
no habría novela».
Años más tarde Fedra se enamoró de su hijastro, Hipólito,
pero este rechazó sus insinuaciones. Entonces Fedra, despe-
chada, y temiendo ser delatada por Hipólito, tramó un plan: lo
ACTIVIDADES (págs. 265-266) acusó ante su padre de haber intentado forzarla. Irritado, Te-
seo entregó a su hijo a la furia de Poseidón para que lo casti-
El teatro tradicional  (págs. 258-259) gara. Fedra, al enterarse de la muerte de Hipólito, quedó
destrozada por la culpa y se ahorcó.
 1. a) El objetivo del teatro tradicional es distraer al público, sin Artistas que se han inspirado en el mito de Fedra: Eurípides, Sé-
un predominio de la finalidad crítica o moral.
neca, Jean Racine, Miguel de Unamuno, André Gide, Salva-
b) Muy pocos empresarios se arriesgan a introducir inno- dor Espriu (escritores) Nicolas Poussin, Alexandre Cabanel.
vaciones. (pintores) y Hans W. Henze (compositor), entre otros.

c) Siguen el mismo estilo teatral de épocas anteriores: len-  7. a) Ser de igual opinión o carácter.
guaje asequible, ambientes conocidos, personajes de la
misma clase social que el público espectador... b) Expresiones cultas: «indigno atropello», «violación de los
derechos individuales». Expresiones populares: «de su cuer-
 2. A principios del siglo xvii, los pícaros Leandro y Crispín llegan da», «el cotarro», «¡Válgame un santo de palo!», «estar
a una ciudad italiana disfrazados de amo y criado, respectiva- curda», «una copa de más». Palabras de la jerga propia de
mente. Crispín, gracias a su labia, hace creer a todo el mundo una profesión: «columnas», «la política del periódico».

que Leandro es un noble rico, generoso y culto, y viven lujosa-


mente sin gastar dinero. Por medio de este engaño Leandro  8. En España se hizo un teatro de vanguardia a la altura del de
intenta enamorar a Silvia, la hija del rico Polichinela, para así otros países.
acceder a sus riquezas. El problema surge cuando Silvia y A Ramón Gómez de la Serna es difícil adscribirlo a alguna
Leandro se enamoran de verdad. corriente, es preferible hablar de «ramonismo». Sus obras
Los intereses creados se inscribe dentro de la comedia bur- más destacadas son Teatro muerto, por su arriesgada pro-
guesa porque aborda de forma amable los defectos y las virtu- puesta, y Los medios seres.

71
Unidad 12 >  El teatro de principios de siglo xx

El teatro de Federico García Lorca se inscribe en las vanguar- ¿Podemos deducir algún rasgo… La Suegra muestra pudor cuan-
dias. Sus obras son difíciles e irrepresentables, pero valiosas. do la chica describe la decoración de las medias y también es
Destacan El público, metateatro, y Así que pasen cinco años, entrometida, pues señala con intención el tiempo que Leonardo
de gran audacia formal. fue novio de su prima.
La Mujer es complaciente con su marido y su suegra, e insegura
 9. ¿Qué asuntos exponen? Exponen la lucha de unos personajes
porque no sabe si le invitarán a la boda de su prima.
contra el orden social para conseguir la libertad. Es un enfren-
tamiento entre lo posible y lo deseado. Leonardo se muestra malhumorado y misterioso, al igual que su
madre, porque no cuenta qué le ocurre.
¿Qué finalidad poseen? Su finalidad es didáctica. Se ponen en
evidencia morales antiguas o equívocas. La Muchacha muestra ingenuidad y no le afecta el ambiente
¿En qué ambiente se desarrollan? Se desarrollan en ambientes tenso en que se desarrolla la conversación.
rurales.
¿Quiénes y cómo son los personajes? Los personajes son tipos 2. Contextualización
representativos de la sociedad del momento y encarnan dife-
a) El fragmento pertenece al acto I, cuadro segundo. En esta esce-
rentes posturas.
na se comenta que la petición de mano será al día siguiente, y
¿Cómo es el lenguaje que emplean los personajes? El lenguaje es se hacen comentarios sobre los regalos que el novio acaba de
coloquial. Lorca utiliza muchas imágenes poéticas y elemen- comprar.
tos líricos.
b) Las familias del novio y la novia gozan de una buena posición
social y económica, superior a la de Leonardo, la Mujer y la
10. Yerma, condenada a la infertilidad, habla sola tratando de es-
Suegra. La Muchacha es la criada de estos últimos. A ambas
tablecer un diálogo imposible con el hijo, tan deseado, que
familias las separan viejos odios y enfrentamientos.
nunca tendrá.
Localiza en el texto... «En el patio ladra el perro»: el ladrido es c) Leonardo es el antiguo novio de la protagonista. En la obra deci-
un símbolo del desasosiego del personaje. de abandonar a su mujer y a su hijo, y rapta a la novia.

«¡Cómo me duele esta cintura donde tendrás primera cuna!»: Posteriormente acaba con la vida del novio, y muere también él
el dolor en la cintura es símbolo de la frustración por la inferti- mismo.
lidad, por el vacío. Es un personaje complejo, que acaba aceptando sus sentimien-
tos y luchando por ellos, aunque la historia concluye en tragedia.
11. a) El respeto al luto, la sumisión de las hijas, la tradición, la
condición de las mujeres, la autoridad materna. Justifica su actitud en esta escena.... Leonardo se pone nervioso
porque se entera de que su antigua novia va a casarse y se
b) Bernarda Alba: autoritaria, tradicional, con una concep- pone de manifiesto que aún alberga sentimientos hacia ella.
ción machista del papel de la mujer. Magdalena: rebelde,
rechaza la clausura y las limitaciones que conllevaba ser ¿Qué papel crees que desempeña… Respuesta abierta.
mujer. La prima de la mujer es la Novia, que desencadena la tragedia
al escapar con su antiguo novio. Ella quiere casarse y alejarse
12. Respuesta abierta. de Leonardo, pero finalmente sus sentimientos se imponen.
¿Por qué crees que Leonardo…? Para enfatizar que es el protago-
ANÁLISIS Y COMENTARIO DE TEXTOS nista de la obra.
(págs. 267-268)
3. Expresión
1. Comprensión a) La parte versificada es la comprendida entre «Las patas heri-
das» y el final del texto. Se trata de 21 versos correspondientes
a) Leonardo acaba de montar a caballo: verdadera. / La prima de la a una canción entonada por la Suegra y la Mujer. Es una com-
Mujer va a contraer matrimonio a la mañana siguiente: falsa. / La posición de arte menor, pues se trata de versos de 6 sílabas. Y
Mujer defiende a su prima: verdadera. / La Muchacha ha ido a com- presenta rima asonante. El esquema métrico es: 6a 6b 6c 6b 6d
prar unas medias con el novio: falsa. / La Suegra y la Mujer cantan 6b 6e 6b 6f 10f 6g 10g 6b 6h 6b 6i 6b 6i 6b 6i 6b. Se trata por
una nana al niño y lloran porque no se duerme: falsa. tanto de un romancillo.
b) «Piden a mi prima»: pedir a la familia de una mujer el consenti- b) Las exclamaciones tienen una función expresiva, y los persona-
miento para casarse con ella. / «Es de cuidado»: no se compor- jes adquieren un tono dramático, premonitorio de lo que va a
ta conforme a lo establecido. / «Se van a juntar dos buenos ocurrir.
capitales»: se van a unir mediante matrimonio dos familias que
gozan de buena posición económica. Las interrogaciones dinamizan el texto, ya que los diálogos son
cortos y directos.
c) «Está abajo tendido [...] como si llegara del fin del mundo»: hi-
pérbole o exageración. / «Duérmete clavel»: personificación. / Tanto las exclamaciones como las interrogaciones proporcionan
«Nana, niño, nana»: epanadiplosis. al texto una entonación más intensa. Se expresa con más fuerza
el sentimiento del personaje. El resultado es un diálogo dinámico.
d) Suegra: práctica, visceral, curiosa. / Mujer: sensible, resuelta,
emotiva. / Leonardo: rudo, retraído, impetuoso. / Muchacha: in- c) Elementos relacionados con la naturaleza: rosa, rosal, clavel, mon-
discreta, vivaracha, impresionable. taña, caballo. / Elementos relacionados con la sangre y el agua:

72
Unidad 12 >  El teatro de principios de siglo xx

agua, río, sangre, nieve. / Elementos relacionados con los metales: Hace que el novio despechado sea quien alcance a los fugitivos
puñal de plata. y también muera.
Explica el significado…: La rosa simboliza la pasión amorosa y En la realidad, quienes dieron muerte al amante fueron unos pa-
también la belleza y juventud de la mujer. Aparece en el poema rientes del novio, que no murieron, sino que fueron encarcelados.
y también en el dibujo de las medias que compra el Novio a la
Las relaciones de parentesco entre los personajes también son
Novia. / El rosal, con su espinoso tallo, simboliza el dolor y la
diferentes a las del suceso real.
dificultad. / El clavel simboliza la pasión del hombre. / La mon-
taña es el lugar al que van los amantes: la muerte. / El caballo
representa el deseo, el instinto. / El agua y el río simbolizan la
vida. / La sangre representa la muerte. En este poema premoni-
EVALUACIÓN (pág. 271)
torio, la muerte corre más rápida que la vida: «la sangre corría
más fuerte que el agua». / La nieve es el frío que quema; repre-
 1. Respuesta libre.
senta los sentimientos contradictorios del amor y su intensi-
dad. / El puñal de plata representa la muerte.
 2. Justifica…: Este texto periodístico pertenece a la tipología de
noticia cultural.
d) Reproducir las acciones de los personajes: «Saliendo»; «Se sien-
Tiene algún rasgo propio del reportaje, pues incluye opiniones
ta»; «Pausa»; «Le aparta bruscamente las manos de la cara»;
de algunos de los protagonistas del hecho noticiado, además
«Entran abrazados»; «Aparece la muchacha»; «Entran co-
de que el lenguaje es libre y creativo.
rriendo»; «La imita»; «Señala el tobillo»; «Señala las pantorri-
llas»; «Señala el muslo»; «Aparecen Leonardo y su mujer»; ¿Dónde podrías encontrarlo… Se podría encontrar en un diario
«Se va llorando»; «Se levanta»; «Sale Leonardo»; «Entra y impreso. En este caso procede de la web de RTVE, del aparta-
vuelve a salir con el niño en brazos»; «La mujer ha permaneci- do Noticias.
do en pie».
Expresar la forma de actuar o de hablar de los personajes: «Agrio»;
 3. El lenguaje utilizado por Jon Bandrés es claro, como es habitual
en las noticias, aunque el autor le ha dado un tono de cierta
«Tímida»; «Con intención»; «Alegre»; «Fuerte»; «Enérgica»;
espontaneidad y naturalidad, usando expresiones coloquiales
«Inmóvil».
como «dar un empujón» o «hacer hincapié», o empleando
Indicar a quién deben dirigirse los personajes: «A su mujer»; «A construcciones metafóricas como «detrás de los focos».
Leonardo»; «A su hija».
e) En la nana aparece un caballo con las patas heridas y dentro de
 4. Los actores van a actuar sin cobrar, para ayudar a poner en
marcha el proyecto.
los ojos un puñal de plata. Es el anuncio de la huida dramática
de los amantes y de la muerte, que llegará en el acto tercero.
Este presagio de muerte se repite cuando se habla de la sangre
 5. ensayando, escenificarán, tablas, actores, escena, localida-
des, función, elenco, obra, repertorio, papel, focos, montaje,
que corre por el río.
local de ensayos.
Los versos «¡Ay, caballo grande / que no quiso el agua!» simbo-
lizan que la pasión amorosa (el caballo) elige la muerte y recha-  6. El argumento de…: El amor entre don Mendo y una joven noble,
za la vida (el agua). En el acto tercero los amantes huyen a para quien su padre tenía otros planes. Don Mendo va a la cárcel
caballo, y camino del arroyo los encuentra el Novio. para no deshonrar a la dama, y al final sale para vengarse.
La finalidad con la que fue escrita: La finalidad de esta obra es
4.  Análisis del tema y de la estructura entretener y hacer reír al público mediante situaciones dis-
paratadas.
a) El tipo de lenguaje: Es un lenguaje sencillo pero lleno de fuerza
y lirismo. La ambientación: Ambientes rústicos y sencillos. La  7. La venganza de don Mendo es una parodia.
inclusión de la nana en la trama: En sus obras, Lorca suele intro-
ducir canciones populares, generalmente con una función  8. La venganza de don Mendo es una obra de Pedro Muñoz
simbólica. Seca del año 1918. Pertenece al teatro costumbrista, y más
concretamente pertenece al subgénero del astracán, que de-
forma los rasgos cómicos con el único fin de hacer reír.
5.  Comentario y opinión crítica
Se trata de una parodia de un drama histórico.
a) En Bodas de sangre, Lorca introdujo algunas variaciones con
respecto al suceso real que le inspiró.  9. Respuesta libre.

73
La narrativa. De la posguerra
13 # a la actualidad
b) Adjetivación: «cuadrada piedra gris», «hojas lacias y amari-
EN CONTEXTO (pág. 273)
llentas», «lenta lluvia», «días inútiles», «historias turbias»,
a) Respuesta abierta. «podrido olor». / Repetición: «¡Cuántos días sin importancia!
Los días sin importancia...»; «me pesaban... me pesaban»;
b) Respuesta abierta. «...llenos de historias... historias completas»; «su olor, que
era el podrido olor...» / Comparación: «Me pesaban como
una cuadrada piedra gris en el cerebro».
ACTIVIDADES (págs. 290-293)

La novela de posguerra (1940-1970)  (págs. 277-286)  3. a) La colmena pertenece al realismo social. El autor no se limi-
ta a describir los hechos sino que pretende denunciar la
 1. a) El protagonista siente consuelo al contar las barbaridades desigualdad que existe entre las clases sociales. En este
que ha hecho. Justifica tu respuesta… «Don Santiago me dijo caso, con la irónica descripción de los lavabos critica los
que lo hiciese si me traía consuelo, y como me lo trae…»; lujos superfluos y desmesurados de quien pueda permitir-
«como para compensar, momentos hay también en que con se comprar ese tipo de lavabos.
ello gozo»; «al contarlo tan alejado me encuentre de todo lo
pasado como si lo contase de oídas»; «¡Buena diferencia hay b) El narrador de La colmena es omnisciente ya que a través
entre el pasado y lo que yo procuraría que pasara si pudiese del personaje presenta los hechos emitiendo un juicio.
volver a comenzar». c) Adjetivación: «grifos relucientes, lozas tersas y sus nítidos,
b) Sí, está muy arrepentido y le gustaría que no hubiera pasa- purísimos espejos»; «lavabos blancos, lavabos verdes…»;
do, aunque reconoce que no puede evitar cometer esos «lujosos retretes»; «elegantes cisternas». Comparación: «La
crímenes y que solo estar encerrado puede evitárselo. tienda luce como una joyería o como la peluquería de un
gran hotel»; «bidés con el cuadro de mandos como el de
c) Afronta la situación con resignación y siguiendo los conse-
un automóvil».
jos de Don Santiago, quién le aconseja que lo haga para
expurgar sus culpas. ¿Qué significado crees que tiene…? Satirizar y ridiculizar los
d) El fragmento pertenece a la última parte de la novela, usos y las costumbres de una burguesía adinerada, que
cuando Pascual Duarte permanece encarcelado por los puede permitirse una serie de lujos cuando otras personas
crímenes que ha cometido. están pasando necesidad.
e) Verbos: creo, hiciese, veo, duele, gozo, encuentre, contase, d) Uno de los motivos por los que la guerra podría tener sen-
procuraría, pudiese, evito, quiero. Pronombres: uno, me, tido sería para eliminar las enormes desigualdades econó-
mi, yo. micas que existían en la sociedad española de posguerra.
f) Algunos de los rasgos de la novela existencial que apare-
cen en el fragmento de La familia de Pascual Duarte es el  4. a) El autor reproduce los diálogos de los personajes de forma
destino trágico al que se ve abocado el protagonista como realista. El narrador no interviene, y el lector conoce las
consecuencia de la incomunicación y del ambiente fami- situaciones a través de las acciones de los personajes. El
liar del que procede. El personaje vive situaciones límite de estilo directo de este fragmento imprime agilidad a la lec-
violencia y desesperación y es consciente de sus errores. tura: —Dígame, ¿están ahí todavía? / —Sí, sí que están
En el fragmento predomina el narrador en primera perso- — contestaba Mauricio—. ¿Sucede algo? / Una desgracia.
na, que recurre con frecuencia al monólogo para expresar
b) Se trata de un narrador omnisciente que no juzga, solo
sus pensamientos. En el texto se emplea un lenguaje colo-
narra lo que ve, y se expresa en tercera persona.
quial, adecuado a la condición social del personaje.
g) El texto pertenece a una novela tremendista porque confie- Entre sus intervenciones distinguimos la conducta de los
sa los errores cometidos, así como la difícil situación social personajes a través de lo que dice el autor. En el primer
en que vive este y su familia. Pascual Duarte es el típico caso: «miró hacia la puerta…», «contestaba Mauricio...»;
personaje de este tipo de novelas en las que el protagonis- en el segundo caso: «Ya salía de la mesa. Daniel lo cogía
ta siempre recurre a la violencia para solucionar los proble- por un brazo…».
mas que se le van presentado a lo largo de la vida.
c) Algunos de los rasgos de la novela social que pueden ob-
 2. a) Algunas de las expresiones que reflejan el vacío y la angus- servarse en el texto son:
tia vital de la protagonista son: «¡Cuántos días sin importan- —— Aparición del protagonista colectivo. Los personajes
cia! Los días sin importancia... me pesaban encima»; «me que aparecen en la novela pertenecen a diferentes co-
envolvía la tristeza. Tenía ganas de... volver la espalda a lectivos y actúan: grupo de amigos, personas que tra-
todo»; «¡Cuántos días inútiles!». bajan en la venta, domingueros…
La temática de este fragmento puede relacionarse con la no-
—— Todo sucede en un tiempo y un espacio concreto: 16
vela existencial. La protagonista se lamenta de la monotonía y
horas a orillas del río Jarama.
el vacío con que transcurren los días. Ha perdido la ilusión
con la que llegó y le gustaría aislarse, mantenerse al margen —— El punto de vista es el de un narrador testigo que narra
de la opresión que le produce el ambiente de la casa. en tercera persona lo que sucede.

74
Unidad 13 >  La narrativa. De la posguerra a la actualidad

d) La primera parte del fragmento es un diálogo en el que hablar, y, lógicamente, ese paso debe ser solemne, inclusi-
predomina un estilo directo que reproduce el habla colo- ve, si me apuras, ajustado a unas palabras rituales, acuér-
quial. La segunda parte sería la descripción donde se pre- date de lo que decía la pobre mamá, que en paz
senta el ambiente de forma realista y con gran detalle. descanse.»; «que tú, como un niño maleducado, mira que
eres». Todo el pasado está intercalado por algo que está
 5. a) En la novela experimental reaparece la exposición de la recordando en el presente. El pasado es el diálogo entre
problemática existencial de los personajes. Este fragmento Mario y Menchu el día en que él se le declaró:
describe al Muecas, y para ello el autor combina el monólo- Marío. —¿Quieres ser mi novia?
go interior («En aquella pieza yo les haría la suya talmente,
con chimenea y todo, por los tres mil reales…») y la voz del Menchu. —¿A qué ton?
narrador («Especulador chabolero enriquecido con ventas Mario. — Pues por que sí.
de comestibles en dosis mínimas»). Los registros lingüísti-
cos varían para dar veracidad a las descripciones y a los Menchu. —¿De modo que porque sí se hacen novios dos
personajes: el narrador utiliza un estilo culto y cuidado, personas?
frente al personaje que monologa reproduciendo el habla
Mario. —Me gusta estar contigo.
coloquial de la calle («No, lo que es esas piedras no, que yo
ya las tengo reservadas por si se deciden y por decirlo de Menchu. —Si te gusta estar conmigo será por algo, ¿no?
una vez, son mías. Nadie me las va a disputar a mí»).
Mario. —Porque te quiero.
b) Se describe al Muecas, personaje de origen humilde que
Menchu. —Eso yo es otra cosa.
proporciona animales para el laboratorio del protagonista,
y se refleja la compleja sociedad donde sobrevive: «Espe-
culador chabolero enriquecido con ventas de comestibles La novela actual  (págs. 287-288)
en dosis mínimas, decía que los tiempos eran malos y
que las compras de azúcar de diez en diez céntimos solo  7. a) El tema es la incertidumbre de la persona y de la existen-
llevaban al vicio.» cia. Las ideas principales: la relación de Bárbara con Chi-
c) El autor emplea un tono sarcástico para describir la posi- cho, la relación de Bárbara y el protagonista; la dificultad
ción social del Muecas; mediante una hipérbole, le asigna de entendimiento y las conclusiones que saca de todo ello.
la profesión de arquitecto aparejador, para referirse a su b) La expresión «en las redes de las mujeres» significa ena-
actividad como albañil: «Arquitecto aparejador-contratista morarse. ¿Qué figura retórica…? Una metáfora.
de chabolas, en feliz ignorancia de planes de ordenación y
normas municipales, construía como pocos pueden ya, al c) Respuesta orientativa. La lengua y el estilo de este texto repre-
libre albedrío de su instinto artístico y de acuerdo con la sentan un retorno a la sencillez y a la naturalidad del lenguaje.
naturaleza de sus materiales.»
 8. a) El documento está escrito como un atestado de la policía.
d) Todos los golpes del subconsciente van brotando sin nin- El uso del relativo que delante de cada oración supone que
guna lógica, a medida que van apareciendo. Es un estilo hay un verbo principal que en este caso sería «declara».
indirecto libre, en el que el protagonista expresa lo que se Los hechos narrados son la muerte del periodista Domingo
le ocurre: brotar de la conciencia. Pajarito de Soto, los detalles de las pesquisas policiales y
e) La novela denuncia las injusticias sociales, como la novela los resultados obtenidos.
social, pero también aborda los conflictos existenciales de Escribe de nuevo el texto… Respuesta abierta.
los personajes. El estilo es más elaborado, combina distin-
tos registros lingüísticos, figuras retóricas, cambios de b) Se trata de un narrador-testigo que declara lo que ha visto.
perspectiva del narrador y rupturas temporales.
c) La obra se inscribe en el género de la novela de intriga y
policíaca. El fragmento muestra cómo se intenta esclarecer
 6. a) Referencia a la segunda persona: tú vengas, te pones, fuiste, la muerte del periodista, que llevaba a cabo una investiga-
ya ves, me diste; uso de vocativos: Mario, ya ves tú, querido; ción sobre la empresa Savolta.
uso de imperativos:
perdóname, fíjate, acuérdate.
b) El autor ha alterado la puntuación del texto: verdadera («Ma-  9. La tendencia narrativa a la que pertenece La voz dormida, de
rio, que a lo mejor me pongo inclusive pesada, pero no es Dulce Chacón, es la novela histórica. La obra está ambientada
una bagatela eso, que para mí…»). Predomina un estilo…: en la posguerra española (1939-1963). La autora quiere de-
falsa («buena trabajina me diste…; que acabaste por pasar nunciar las consecuencias de la guerra y la importancia de las
por el aro, zascandil…»). La sintaxis es correcta: falsa («que mujeres en un momento tan difícil de la historia.
tú, como un niño maleducado, mira que eres…»).
c) Es una expresión que dirige el amado a la amada. ANÁLISIS Y COMENTARIO DE TEXTOS
d) El presente se refiere al monólogo de Menchu ante el (pág. 294)
muerto: «y perdóname que insista, Mario, que a lo mejor
me pongo inclusive pesada, pero no es una bagatela eso,
que para mí, la declaración de amor, fundamental, impres- 1. Comprensión
cindible, fíjate, por más que tú vengas con que son tonte- a) Respuesta abierta.
rías. Pues no lo son, no son tonterías, ya ves tú, que te
pones a ver, y el noviazgo es el paso más importante en la b) Una mujer de dos saca cuatro: una mujer que administra eficaz-
vida de un hombre y de una mujer, que no es hablar por mente el dinero / Ya estás cumplido: ya has hecho suficiente / No

75
Unidad 13 >  La narrativa. De la posguerra a la actualidad

vivías en el mundo: no eras una persona práctica / Tú erre que y te has dejado querer, Mario, que así qué cómodo, que te
erre: tú empeñado en seguir con lo mismo / Por un oído me entra crees que con un broche de dos reales o un detallito por mi
y por otro me sale: no presto atención, no hago caso de los con- santo ya estás cumplido.» «A la gente le importan un comino
sejos / Por darse pote: por darse importancia / Porque hace bien: las tesis y los impactos, créeme, que a ti, querido, te echaron
porque queda bien. a perder los de la tertulia, el Aróstegui y el Moyano, ese de las
barbas, que son unos inadaptados.» / Palabras y expresiones
c) Carmen reprocha a su marido difunto los pocos detalles que
comodín «coletillas» propias de la expresión oral: «Ya, vas a de-
tuvo con ella y critica la ideología y el empleo de su marido, que
cirme que tú tenías tus libros»; «a ti, querido, te echaron a
tantos problemas les acarreó y no les permitió disfrutar de una
perder los de la tertulia»; «bueno, pues tú erre que erre». /
posición económica más desahogada.
Metáforas e hipérboles coloquiales: «por un oído me entra y por
otro me sale»; «me será muy difícil perdonarte, cariño, por mil
2. Contextualización años que viva.» / Repeticiones enfáticas: «que así qué cómodo,
que te crees que con un broche de dos reales o un detallito
a) La obra, publicada en 1966, se adscribe a la corriente de la por mi santo ya estás cumplido, y ni hablar, borrico, que me he
narrativa existencial contemporánea. Evoca la existencia de hartado de decirte que no vivías en el mundo, pero tú, que si
Carmen junto a su marido, Mario, de cuerpo presente. Se trata quieres.» / Exclamaciones e interrogaciones retóricas: «¿sabes lo
de un monólogo interior en el que la mujer reprocha al esposo que es, Mario? ¿te acuerdas?» / Usos de vocativos y de imperati-
muerto su actitud hacia ella y su propia vida. vos…: «vas a decirme», «no me vengas ahora, hijo,» «Vamos a

Explica la importancia de Miguel Delibes… Delibes es uno de los ver», «ya lo oyes».
grandes autores de la narrativa española contemporánea, y
cuenta con obras importantes en todos los períodos. Trata todos
los temas y experimenta con todas las técnicas propias de la
4.  Análisis del tema y de la estructura
novela contemporánea. a) Primera parte: Desde el principio hasta «hoy nadie se confor-
ma con un empleo». Carmen recrimina a Mario no haber ac-
b) La obra pertenece a la corriente de novela experimental, por el
cedido a sus caprichos (coche, regalos) y no haberle ofrecido
punto de vista del narrador, el argumento y su técnica narrativa.
una mejor posición social. / Segunda parte: desde «Ya, vas a
Se trata de un monólogo interior, aunque el autor le confiere un
decirme…» hasta «…y luego se enamoró de la hijastra». Re-
carácter «dialogado», ya que el discurso de Menchu se dirige a
proche por su inquietud intelectual, su afición a la literatura y
su marido. Para ello utiliza imperativos y vocativos en segunda
la filosofía y la publicación de escritos que ella no entiende y
persona.
que en su opinión solo servían para causarle problemas. / Ter-
cera parte: desde «Bueno, pues tú a cuento…» hasta el final.
3. Expresión Le reprocha que haya mantenido su actitud vital, que a ella le
parece absurda e incomprensible, lo mismo que sus escritos,
a) Uno de los rasgos de la obra de Miguel Delibes es su perspecti- desoyendo todos los consejos y críticas.
va irónica frente a la hipocresía y la pequeña burguesía. En este
texto, la ironía destila del discurso de la viuda de Mario, en par- b) Frustración: «…tú has tenido la suerte de dar con una mujer
tes como: «Egoísmo puro, para que te enteres […], que hoy en de su casa, una mujer que de dos saca cuatro y te has dejado
día nadie se conforma con un empleo», que muestran el egoís- querer»; «Bueno, pues tú a cuento, por un oído me entra y por
mo de la propia Menchu, o sus comentarios acerca de los libros otro me sale.» / Afán de aparentar: «Nunca lo entenderás, pero
que escribía su marido muerto: «…pero el caso es que no se a una mujer, no sé cómo decirte, le humilla que todas sus ami-
entendía una jota del libro, porque si los generales ven a sus gas vayan en coche y ella a patita.» / Superficialidad: «…que te
soldados […] y con toda la razón, además». crees que con un broche de dos reales o un detallito por mi
santo ya estás cumplido, y ni hablar, borrico...» / Desprecio hacia
b) Es un narrador protagonista, Menchu, quien se dirige a su mari- la cultura: «Ya, vas a decirme que tú tenías tus libros y El Co-
do muerto, al que describe y hace continuos reproches mien- rreo, pero si yo te digo que tus libros y tu periodicucho no nos
tras transcurren las horas del velatorio. han dado más que disgustos...» / Clasismo y prejuicios sociales:
c) Variedad de registros lingüísticos: Registro culto: «Los que «...hoy un Seiscientos lo tiene todo el mundo, Mario, hasta las
quieren enriquecerse caen en tentaciones, en lazos y en mu- porteras…»; «…¿quién iba a leer esas cosas tristes de gentes
chas codicias locas y perniciosas que hunden a los hombres muertas de hambre que se revuelcan en el barro como puer-
en la perdición y en la ruina, porque la raíz de todos los males cos?».
es la avaricia». Registro coloquial: «y por eso mismo me será c) Mario era catedrático de instituto, culto y desinteresado, cola-
muy difícil perdonarte, cariño, por mil años que viva, el que boraba en un periódico y en una tertulia y había publicado dos
me quitases el capricho de un coche». Ausencia de elementos de libros. / Carácter: a Mario le interesaba la cultura y no el dine-
relación: En la expresión del monólogo no aparecen elementos ro, tenía inquietudes literarias y filosóficas. A Menchu le intere-
de relación. Alteración o ausencia de puntuación: «Ya, vas a de- san las cuestiones materiales (coche, regalos, etc.) y las
cirme que tú tenías tus libros y El Correo, pero si yo te digo que apariencias sociales. / Amigos y tipos de conversación: los amigos
tus libros y tu periodicucho no nos han dado más que disgus- de Mario eran los de la tertulia, en opinión de Menchu, unos
tos, a ver si miento, no me vengas ahora, hijo, líos con la cen- inadaptados. Las amigas de Menchu, que se reúnen para to-
sura, líos con la gente y, en sustancia, dos pesetas» (ausencia mar el té, viajan en coche y no entienden los libros de Mario,
de puntos). hasta el punto de reírse de él. / Cultura y educación: Mario tenía
d) Lenguaje coloquial: «por un oído me entra y por otro me sale»; una sólida formación intelectual y sus preocupaciones filosófi-
«por darse pote.» / Sintaxis fragmentada y desorden de ideas: cas escapaban a su mujer, que no entendía sus obras y propo-
«Aunque me esté mal el decirlo, tú has tenido la suerte de dar nía a su marido que escribiera sobre intrigas amorosas. /
con una mujer de su casa, una mujer que de dos saca cuatro Ideología: Mario escribía en un periódico, hecho que le causa-

76
Unidad 13 >  La narrativa. De la posguerra a la actualidad

ba problemas con la censura y con la gente. De eso se deduce expresando su opinión personal. La extensión del artículo, fir-
su inclinación hacia el pensamiento de izquierdas. Menchu se mado por el autor, es breve.
queja porque no entiende que lo hiciera por tan poco dinero,
en lugar de buscar un pluriempleo para mejorar su posición  6. El autor se expresa con un lenguaje elaborado, en el que se
social. incluye algún término especializado de la literatura (tradición,
vanguardia, fragmentarismo, argumento) y las nuevas tecno-
logías (mundo virtual, Google Earth, Internet). Localiza algún
5.  Comentario y opinión crítica tecnicismo … Poliédrica: de diversas facetas; anacrónicas:

Respuesta libre. trasnochadas.

 7. Algunos autores de la generación del 27 (Rafael Alberti,


EVALUACIÓN (pág. 297) Vicente Aleixandre, Pedro Salinas) empleaban el tradicio-
nalismo y la vanguardia según las necesidades expresivas.
 1. El texto trata sobre la crisis de la novela tradicional a causa de En la poesía de Federico García Lorca ambas tendencias se
las nuevas tecnologías. mezclaban y las diferencias entre ellas quedaban difumi-
nadas.
 2. Mutante: variable, inestable; global: general, universal; ende-
ble: delicado, frágil.  8. Respuesta libre.

 3. Nuevos ingredientes de la novela: las innovaciones en la novela;  9. Como se dice en el texto, Agustín Fernández Mallo (1967)
realidad poliédrica:
diferentes aspectos que puede presentar la y Eloy Fernández Porta (1974) son unos «promotores» de
realidad; estructuras sociales anacrónicas: contexto social anti- la nueva manera de concebir la literatura. Ambos autores
cuado; conquistas tecnológicas: descubrimientos de la tecnolo- pertenecen a una generación de escritores (Generación
gía moderna. Nocilla) que formaron una corriente literaria al margen de
la tradición. Las obras de estos autores se caracterizan por
 4. La novela actual… debe tener en cuenta nuevas formas de la combinación de diferentes técnicas expresivas (collage,
expresión. Aunque no desaparezca la novela tradicional, hay música, performance, blogs), el fragmentarismo, la inter-
que tener en cuenta las nuevas tecnologías ya que es una disciplinariedad.
evolución inevitable de la sociedad actual.
10. Respuesta libre.
 5. El autor es un periodista que opina sobre un tema de actuali-
dad, la crisis de la novela tradicional, de manera sencilla y 11. Respuesta libre.

77
La lírica. De la posguerra
14 # a la actualidad
¿Por qué crees que Dámaso Alonso prescinde… Dámaso Alon-
EN CONTEXTO (pág. 299)
so utiliza el verso libre para dar mayor expresividad al poe-
a) Berta es un personaje elíptico porque no aparece en la película, ma, lo que supone una ruptura tanto en el contenido como
solo se habla de ella, representa el Londres, la Inglaterra que no en la forma.
se ve. b) vacío, abismo, frío, miedo.
b) ¿Cuál debe ser la preocupación de un poeta…? La preocupación c) Dámaso Alonso le reprocha que haya permitido la guerra y
del poeta debe ser mantener los ojos y los oídos bien abiertos, las muertes y el dolor consiguientes (en sentido simbólico,
registrar, procesar, reflejar —a su manera, y con el corazón en habla de un millón de cadáveres para referirse a quienes
una mano y la tradición que, personalmente, se ha forjado y ha viven en Madrid). Extrae palabras relacionadas…: cadáve-
aprendido a respetar, en la otra— la realidad y el mundo, tal y res, revuelvo, nicho, pudro, gemir, gimiendo, enfurecien-
como él los concibe. do, pudre, pudren, podredumbre, letales.
¿En qué consiste la libertad creativa? La libertad creativa consiste d) Ambos poemas expresan el vacío de la existencia humana,
en no tener que amoldarse a una estética dominante sino elegir motivado por la situación en que se encontraba España.
libremente aquellas que se derivan de los autores que el poeta
lee y admira.  4. a) Se refiere a aquellos escritores que tienen una concepción
más esteticista y refinada de la poesía. En esa época, estos
¿Qué factores crees que se oponen a la libertad del artista? Los ideales los encarnaban los poetas de la corriente garcilasis-
factores que se oponen a la libertad del artista son las consig- ta. Anteponían la belleza estética al sufrimiento humano,
nas, las estéticas dominantes y las influencias excesivas de al- en una época en la que la poesía no podía dar la espalda a
gunas literaturas nacionales. la terrible realidad social.
Investiga de dónde está tomada la expresión..: La expresión pro-
ACTIVIDADES (págs. 309-310)
cede de Juan Ramón Jiménez, quien dedica su Segunda
Antología «a una inmensa minoría». El poeta escribe para
una inmensa mayoría, y por ello utiliza un estilo claro y sen-
La poesía de la posguerra  (págs. 300-304) cillo: se trata de una poesía social, que denuncia la situación
que vive el país.
 1. Analiza la métrica y la rima…: Se trata de versos endecasílabos,
11A 11B 11B 11A 11A 11B 11B 11A 11C 11D 11E. La acen-  5. La poesía social antepone el contenido del mensaje al cuida-
tuación recae en la sexta y la décima sílabas, lo que corres- do de la forma poética: «porque vivimos a golpes, porque
ponde a una gran elaboración formal. apenas si nos dejan / decir que somos quienes somos, / nues-
tros cantares no pueden ser sin pecado un adorno.» // El len-
a) El poeta expresa su visión del mundo mediante contrarios, guaje de esta poesía es claro y sencillo, para que pueda
como el contraste entre llenarse de amor y vaciarse («sé entenderse por la gran mayoría de los lectores: «Hago mías
que de amor me lleno dulcemente» / y en voz a borbotones las faltas. Siento en mí a cuantos sufren / y canto respirando.»
me derramo»), o la mezcla de angustia y júbilo («Suben y // Los poemas emplean un tono coloquial y prosaico que huye
ya no sé donde coincide / mi angustia con mi júbilo, orde- de la artificiosidad: «poesía para el pobre, poesía necesaria /
nando»). A pesar de estas oposiciones, la razón acaba or- como el pan de cada día.» // Los poetas sociales denuncian
denando el mundo («ordenando esta razón...»). las injusticias que sufren los oprimidos y la falta de libertades:
b) La búsqueda de la musicalidad, el uso de formas métricas «hago mías las faltas. Siento en mí a cuantos sufren / y canto
clásicas y una concepción del mundo armónica son carac- respirando.» // La poesía social rechaza la poesía preciosista
terísticas de la corriente poética arraigada, que tiende a que se escribe de espaldas a la realidad: «Maldigo la poesía
una poesía formalista de corte clásico. concebida como un lujo / cultural por los neutrales / que, la-
vándose las manos, se desentienden y evaden. / Maldigo la
 2. En la última estrofa..: Al poeta le preocupan el destino y la so- poesía de quien no toma partido hasta mancharse».
ledad, y se siente arraigado al amor por su hijo.
 6. «La partida»…: poesía arraigada. / «A la inmensa mayoría»…:
Justifica que este poema…: El poema muestra una cuidada for- poesía social. / «España en marcha»…: poesía social.
ma estrófica y plantea problemas existenciales y religiosos,
desde el punto de vista de la fe («Desde mi vieja orilla, desde  7. Respuesta abierta.
la fe que siento») y la serenidad («me arrastras de la mano...
Y en tu ignorancia fío»).  8. El poema presenta una técnica conversacional, en la que el
autor habla consigo mismo («me pregunto...»). La
 3. a) Los poemas emplean versos alejandrinos, endecasílabos y composición muestra las sensaciones íntimas y personales
versos libres, respectivamente. del poeta, que intenta interpretar el mundo que tiene ante sí.
El lenguaje es sencillo, pero sin embargo el poema presenta
Crees que el poema de Blas de Otero… Sí, se trata de un sone- una forma cuidada, con imágenes, contrapuntos, puntuación
to alejandrino: 11A 11B 11B 11A 11A 11B 11B 11A 11C expresiva... Trata el tema del amor, como se evidencia en el
11D 11E 11C 11D 11E (forma habitual en la tradición lírica último verso. Este tema aparece con frecuencia en los poetas
española). de la Promoción de los años sesenta.

78
Unidad 14 >  La lírica. De la posguerra a la actualidad

 9. «Palabras para Julia»…: Goytisolo dedica este poema a su hija ba las hojas y despeinaba a las niñas, como si se tratara de un
recién nacida, a la que anima a enfrentarse al futuro, a la juego. En la última estrofa reconoce cómo las neurosis de su
existencia, que, pese a las dificultades, vale la pena vivir. / «El madre también se han apoderado de él y describe con dolor las
adiós»…: despedida del marido a su mujer fallecida. / «Alto sensaciones que le producen el ruido del viento y de los rayos.
jornal»…: la satisfacción del trabajo.
d) Con el paso del tiempo, todos los recuerdos que le producía el
viento se convierten en realidad: aunque digan que está lejos,
es pequeño y distante.
La poesía actual  (págs. 305-307)
10. a) El uso de versículos, es decir, la ausencia de métrica y de 2. Contextualización
rima.
a) El tema de la infancia es uno de los asuntos recurrentes en la
b) El poeta prescinde de la puntuación. Este recurso tiene poesía de la Promoción de los años sesenta. El tono intimista y
como objetivo crear un poema en el que lo más importan- autobiográfico en la manera de abordar el amor a su madre y el
te es el ritmo interno de la composición, pues se antepo- recuerdo de esta encaja dentro de esa corriente.
nen las imágenes y las sensaciones antes que la sintaxis
y la lógica. b) La poesía de la promoción de la década de 1960 deja de ser un
medio de comunicación y se convierte en un vehículo de auto-
c) La música de saxofón y el cantante Nat King Cole. conocimiento, en el que el poeta expresa sus sentimientos más
d) De la muerte. íntimos: angustia, tristeza, pena…

e) Es un poema de los novísimos por el afán de innovación, el


uso del verso libre, las técnicas de collage y las referencias 3. Expresión
a la cultura contemporánea.
a) El poeta utiliza la anáfora para dar musicalidad a la composición.
El empleo de signos ortográficos (guiones, paréntesis y puntos
11. a) El autor señala que en la poesía actual no se tratan los te-
suspensivos) permite distintos cambios de ritmo. El léxico es
mas que preocupan a la sociedad.
sencillo pero evocador, y crea imágenes que proporcionan plas-
b) Se identifica con la poesía de la diferencia. Esta corriente ticidad al poema («cubriendo de cadáveres mínimos distantes
intenta reintroducir en la poesía un compromiso con la territorios»).
actualidad.
b) Métafora: «las huellas de la sangre.» / Personificación: «el vien-
c) Respuesta abierta. to... era como un alegre barrendero.» / Anáfora: «Recuerdo...
Recuerdo».
12. El poema «Madre» de Luis García Montero está escrito en c) El poeta se refiere a la guerra como un temor de la madre por su
versos libres, es decir, no se ajusta a ninguna pauta de rima experiencia vivida, como un hecho terrible que pasó y que aho-
o metro. Es un lenguaje sencillo y coloquial, con un léxico ra se ve de lejos, con un tono mucho más íntimo.
que se nutre del habla cotidiana. El tema es la abnegación
de su madre, que ha vivido más para los demás que para sí d) El poeta emplea la técnica autobiográfica para confesar sus vi-
misma. vencias y temores.

Por qué se puede incluir…: Se puede incluir en la poesía de la


experiencia porque muestra su predilección por la cotidianei- 4.  Análisis del tema y de la estructura
dad, con asuntos realistas y verosímiles que afectan al indivi-
duo moderno: el sentimiento del tiempo, lo íntimo, lo a) El poeta evoca su infancia, la memoria de la guerra y, en con-
individual. El lenguaje, a su vez, se caracteriza por la naturali- creto, el recuerdo de su madre y las sensaciones negativas que
dad, mientras que la métrica de esta corriente poética apues- despierta.
ta ora por las estrofas clásicas, ora por el verso libre como en b) El poema puede dividirse en dos partes. La primera se refiere a
este poema de Luis García Montero. los recuerdos del poeta cuando era niño. El autor describe la
imagen que tiene de su madre y los temores que tenía. A conti-
13. Respuesta abierta. nuación, habla de él cuando era niño y de las consecuencias de
la guerra. En la segunda parte, habla el yo poético desde el
presente. El poeta asume la carga de miedo y dolor, heredada
ANÁLISIS Y COMENTARIO DE TEXTOS de la madre.
(pág. 312)

5.  Comentario y opinión crítica


1. Comprensión a) Respuesta abierta.
a) El recuerdo de la guerra y el miedo que le provoca. b) Respuesta abierta.
b) Sí, al recordar a la madre revive el miedo de la guerra que dejó
tantas víctimas.
EVALUACIÓN (pág. 315)
c) El autor mantiene dos actitudes diferentes a los largo del poe-
ma. Primero evoca los recuerdos de cuando era niño y no cono-  1. El autor sostiene que la poesía actual carece de valor, y que
cía la gravedad de la guerra. El viento, que tanto miedo le los temas que aborda no tienen suficiente profundidad como
producía a su madre, para él solo era un fenómeno que levanta- para atraer a los lectores. Esta literatura, en opinión del autor,

79
Unidad 14 >  La lírica. De la posguerra a la actualidad

no tiene interés para su difusión internacional. El autor espera versal: autores que han sido traducidos a varias lenguas y
que se produzca un cambio y que la literatura refleje en un apreciados en distintos países. / Estado moral de una sociedad:
futuro la realidad. pauta de pensamiento y comportamiento de la población en
un contexto concreto.
 2. El desprestigio de la lírica actual.
 7. Simplicidad filosófica aterradora: hipérbole. / Oriente y occidente
 3. Valía: valor, aprecio de algo / Manido: dicho de un asunto o un debaten: personificación / Fértil decadencia: oxímoron.
tema, muy trillado / P roeza : hazaña, gesta / H egemonía :
supremacía / Reiterativa: que denota repetición.  8. 1. Escasa relevancia de temas abordados, sin relación con las
cuestiones importantes de la actualidad. 2. Limitada relevan-
 4. Cultivadores: seguidores, poetas / Desgañiten: voceen, griten / cia internacional. Los editores no se interesan por estas obras,
Aterradora: terrorífica, pavorosa / Aprecio: estima, valor / Dig- pero sí muestran interés por otros poetas en lengua española
nas: meritorias, plausibles / Vanguardia: modelo, novedad. que siguen siendo traducidos a varios idiomas. 3. La abun-
Propón cinco adjetivos…: Respuesta abierta. Se ofrece la siguien- dancia de ediciones no significa que la poesía sea de calidad.
te: insustancial, irrelevante, banal, pobre, intrascendente...
 9. El texto corresponde a la tipología del ensayo periodístico.
 5. Respuesta abierta. Muestra las siguientes características: voluntad didáctica,
un estilo sencillo y comunicativo, pero con un propósito
 6. Respuesta orientativa. Se ofrece la siguiente orientación: Dis- estético. Por su corta extensión y concisión de ideas pode-
cursos personales:que los poetas actuales no reflejan un uni- mos decir que se trata de un ensayo publicado en prensa,
verso propio, singular o íntimo, sino un pensamiento aunque la referencia de título y obra no nos indica el medio
uniformado que no difiere entre unos y otros. / Pensamientos de en que se ha editado.
altura: pensamientos trascendentes, sobre temas de relevan-
cia para el ser humano y el contexto cultural. / Dimensión uni- 10. Respuesta abierta.

80
El teatro.
15 # De la posguerra a la actualidad
abuelo, con la vida. La humanidad, ya que se exalta la bondad
EN CONTEXTO (pág. 317)
y la dignidad del ser humano, encarnado por el abuelo; y el
a) Las reflexiones del autor se pueden relacionar con el teatro poé- lirismo, con un lenguaje poético: metáforas («tus manos de
tico o de ensueño. hielo», «Quién me ligó con dulces hilos que no había sentido
nunca?», selección de las palabras por su valor rítmico…
b) En el tráiler se anuncia la obra Tres sombreros de copa, de Mi-
guel Mihura. Se destaca que de joven se embarcó en la gira de a) El abuelo representa la realidad y la Peregrina la ilusión.
la compañía de varietés del cómico Alady, hecho que segura-
mente le influyó a la hora de escribir Tres sombreros de copa y,  4. En qué situación se produce… Se produce en una situación
en general, en su obra teatral de tono humorístico. absurda pues las protagonistas, Clotilde y Mariana, han ido a
un cine sin saber qué hacían en él y, sin embargo, han dejado
c) Respuesta abierta. de ir a un concierto que sí les interesaba.
a) La característica de los personajes que apreciamos es que
ACTIVIDADES (págs. 330-333) tienen tics y manías externos: Mariana de pequeña se co-
mía las flores y una temporada le dio por andar hacia atrás,
mientras que la abuela les hacía vestiditos y sombreritos a
El teatro de posguerra  (págs. 318-326) todas las cerillas que caían en sus manos; y el abuelo se
pasó los últimos años de su vida pelando guisantes.
 1. Este fragmento muestra rasgos de humor ingenioso, pues se
b) «Por lo que afecta a tu hermana, corramos un velo; y con
dice que si participas en cacerías de la alta sociedad y eres
respecto a mí, bajemos un telón metálico.»; «también los
buen cazador, puedes aspirar a un ministerio. El teatro bur-
Ojedas son para un Noticiario»; «A mí es ya muy difícil que
gués era un teatro amable e ingenioso que pretendía divertir
nadie me parezca raro, hija mía, acostumbrada como es-
al público. Tenía un tono elegante y moralizador en su crítica
toy a los de casa».
de las costumbres, tal como se ve aquí: se critica veladamen-
te la manera de elegir los ministros en aquella época. Otro c) Sí: primero Mariana habla de que establecerá una amis-
rasgo a destacar es que el diálogo está cuidadosamente ela- tad duradera con el acomodador, del cual añade que si
borado, como corresponde a autores de prestigio, y Jaoquín se quitara el bigote para hablar ganaría mucho. Luego
Calvo Sotelo lo era. Se puede deducir que estos personajes resulta que no saben qué van a representar en ese salón,
pertenecen a las clases media o alta: burgueses acomodados pero, sin embargo, han dejado de asistir a un concierto
o aristócratas que viven sin problemas económicos, y que que sí les interesaba. A continuación explica los tics de
acuden a cacerías de postín. los abuelos y la locura sin curar del tío Cecilio. Finalmen-
te repasa los tics de Mariana en su niñez.
 2. a) Francisco Rabal destaca del gaditano José M.ª Pemán «su
gracia andaluza y su sal», rasgos que, efectivamente, en-  5. a) Esta escena se sitúa en la parte final de la obra, el desen-
contramos en sus obras pertenecientes al grupo de la farsa lace (en el tercer acto), cuando Paula se entera de que
castiza, comedias ligeras de costumbres de ambiente an- Dionisio no es malabarista y además se tiene que casar
daluz, donde predominan el sentido del humor y la habili- con su novia.
dad en los enredos. Sobresalen dentro de esta temática:
b) Paulina y Dionisio sienten frustración y abatimiento porque
Los tres etcéteras de Don Simón y La viudita naviera.
constatan que la felicidad alcanzada juntos se esfuma por
b) Hacia la mitad del poema, Rabal menciona que Pemán las circunstancias que los rodean.
tuvo una buena relación con el periodista y ensayista
c) Paula representa a los personajes del espectáculo de va-
Paco Umbral y con el también dramaturgo Alfonso Sas-
riedades, que son el símbolo de una sociedad que no se
tre, uno de los autores más destacados del drama social
ajusta a unos moldes ni a una moral convencional, sino a
o realista de los años 50 y 60.
una vida errante, alegre y libre, en apariencia. Dionisio,
junto con don Sacramento (padre de su prometida) y Mar-
 3. Alguno de los rasgos del teatro poético o de ensueño de pos-
garita (su novia) reflejan los valores de una sociedad bur-
guerra que aparecen en este fragmento de La dama del alba
guesa que se rige por la cursilería, el dinero y los
son la presencia del amor y la comprensión para transformar
convencionalismos sociales.
los aspectos negativos de la realidad humana, en este caso la
muerte, tan ligada a nuestra existencia (el diálogo lo protago- d) En este fragmento subyace la crítica a los matrimonios im-
nizan la Peregrina, que simboliza la muerte, y el abuelo, que puestos socialmente. Esta crítica es característica de la
defiende a sus nietos). La universalidad, ya que se plantean obra teatral de Mihura porque en ella se ataca a los con-
problemas humanos y comunes a todos, como la realidad vencionalismos o imposiciones sociales que ahogan la li-
inexorable de la muerte. La figura del abuelo representa a los bertad de la persona y, por tanto, su felicidad.
débiles, tal como desvela Peregrina cuando afirma «Soy bue-
e) Lenguaje absurdo: Dionisio. —Sí. Me caso, pero poco...
na amiga de los pobres y de los hombres de conciencia lim-
pia. ¿Por qué no hemos de hablarnos lealmente». El idealismo, Situaciones ridículas: Dionisio. —[…] Pero ya no me caso,
transmitido a través de elementos simbólicos de ilusión y de Paula... ¡Yo no puedo tomar huevos fritos a las seis y media
realidad, está representado en la muerte, que dialoga con el de la mañana...!

81
Unidad 15 >  El teatro. De la posguerra a la actualidad

Carácter de los personajes: Dionisio. —Porque esta habita- f) Este tipo de teatro fue bautizado como esperpento.
ción es más bonita. Desde el balcón se ve el puerto... […]
g) Respuesta abierta.
Yo no sabía nada de nada. Yo solo sabía pasear silbando
junto al quiosco de la música...
El nuevo teatro  (pág. 327)
 6. a) Fernando es directo, efusivo y sincero: «Pero nosotros no nos
dejaremos vencer por este ambiente. ¡No! Porque nos mar-  8. a) La visión que se da de la justicia es muy negativa, dando a
charemos de aquí. Nos apoyaremos el uno en el otro. Me entender que es una farsa o comedia en que los mismos
ayudarás a subir, a dejar para siempre esta casa misera- jueces eluden cumplir las leyes por la que se rige dicha
ble, estas broncas constantes, estas estrecheces. Me ayu- justicia; también se dice que el sistema penitenciario solo
darás, ¿verdad? Dime que sí, por favor. ¡Dímelo!» / Carmina busca la venganza. Estás de acuerdo con ella. Respuesta
es tímida y retraída: «¡Fernando!» / Fernando es un soñador: abierta.
«Primero me haré aparejador. ¡No es difícil! En unos años
me haré un buen aparejador. Ganaré mucho dinero y me b) Sí que se puede calificar de «teatro experimental» porque,
solicitarán todas las empresas constructoras. Para enton- por una parte, los personajes sufren una destrucción inter-
ces ya estaremos casados... Tendremos nuestro hogar, na y pasan a ser personajes signos portadores de un men-
alegre y limpio..., lejos de aquí. Pero no dejaré de estudiar saje crítico; es lo que le sucede al Emperador. Por otra
por eso. ¡No, no, Carmina! Entonces me haré ingeniero. parte, el desarrollo de la escena muestra que los temas se
Seré el mejor ingeniero del país y tú serás mi adorada mu- esconden bajo parábolas y el espectador ha de descifrar
jercita...» / Carmina siente admiración por Fernando y cree su significado; dichos temas plantean los problemas que
ciegamente en sus palabras: «¡Fernando! ¡Qué felicidad...!
acucian al ser humano contemporáneo, como, por ejem-
¡Qué felicidad!». plo, el ejercicio del poder y la administración de la justicia,
que vemos en este texto.
b) Descripción personajes: Carmina, la madre, sale de su casa
con expresión de inquieta y los divisa, entre disgustada y
angustiada. Gestos personajes: Quedan un momento abra-
El teatro actual  (pág. 328)
zados. Después, él la lleva al primer escalón y la sienta
 9. a) El tema es el hambre que sufre una familia porque las ra-
junto a la pared, sentándose a su lado. Se cogen las manos
ciones no son suficientes.
[…] / Carmina, la madre, sale de su casa […] / Fernando,
el padre, que sube la escalera, se detiene, […] Actitudes de b) Uno de los ejemplos que nos ayudan a localizar la época
los personajes: se miran arrobados […] / sale de su casa en que pasan los hechos es la aparición de la cartilla de
con expresión de inquieta y los divisa, entre disgustada y racionamiento (Don Luis. —Pero hace meses que la ración
angustiada. / Fernando, el padre, que […] se detiene, es- que dan en la cartilla es casi la misma.) y el hambre que
tupefacto, […]. pasa la mayoría de la gente (Doña Dolores. —Que como su
madre entra y sale constantemente en casa, yo no sé si la
c) El fragmento refleja la situación de penuria y el deterioro
pobre mujer, que está, como todos, muerta de hambre,
social propios de la España de posguerra. En los años cua-
[…]).
renta del siglo xx, la población sufre los efectos de la crisis
económica (pobreza, racionamiento de alimentos y otros c) El estilo de Fernando Fernán Gómez es directo, y a veces
productos básicos…). Historia de una escalera se estrena dramático, cercano al realismo, en el que prevalece el
en plena dictadura franquista y supone una vuelta al teatro contenido sobre la forma. Es propio del teatro que al llegar
social y realista. En esta obra, Buero Vallejo denuncia la la democracia retornó a la línea tradicional intentando re-
precaria situación y la falta de futuro en la sociedad espa- flejar situaciones cotidianas y problemas sociales de clases
ñola de posguerra. marginales.

7. a) En este diálogo se plantea un conflicto laboral en una ofici-


na, donde uno de los trabajadores (Crok) es rechazado por COMENTARIO DE TEXTOS (pág. 334)
el resto, debido a que es diferente a ellos y no sigue la co-
rriente.
1. Comprensión
b) Podemos adivinar una crítica de tipo social y político, pues
el autor da a entender que la mayor parte de la sociedad a) Respuesta libre.
española de la época carece de criterio propio y vive aletar- b) Don Sacramento acude al dormitorio de Dionisio para pedirle
gada, distraída con el fútbol y sin plantearse las cosas en explicaciones de por qué no ha respondido al teléfono, hecho
profundidad. que ha afligido a su hija. Las razones de Dionisio no satisfacen a
c) La temática de El tintero se puede relacionar perfectamente don Sacramento, que le considera un bohemio y le explica que,
con la biografía de Carlos Muñiz, pues él era funcionario del en adelante, después de casarse, habrá de llevar una vida orde-
Ministerio de Hacienda por oposición y siempre manifestó su nada en su casa.
malestar ante la administración, de la cual formaba parte.
d) Llama la atención sobre todo el uso de las expresiones po- 2. Contextualización
pulares (es la oveja negra, pena debía darle, A la porra…) a) Tres sombreros de copa fue escrita por Miguel Mihura en 1932,
y coloquiales (mis hijos cagan tan ricamente…). aunque no se estrenó hasta después de la guerra civil, en 1952.
e) Las intervenciones de los personajes recuerdan a los de Se inscribe en la etapa de renovación del teatro español de pre-
las obras de Ramón del Valle-Inclán, pues están marcadas guerra, con autores como Valle-Inclán o García Lorca. Sin em-
por el absurdo. bargo, Mihura no se dio a conocer como dramaturgo hasta la

82
Unidad 15 >  El teatro. De la posguerra a la actualidad

posguerra. En su primer período (1932-1946) las piezas teatra-


les se acercan al teatro del absurdo, siempre con una intención EVALUACIÓN (pág. 337)
crítica. La obra más representativa de esta época es Tres som-
breros de copa.  1. El matrimonio Tepán visita a su hijo Zapo en el frente de gue-
rra. Allí lo encuentran junto a un enemigo hecho prisionero,
Zepo, con quien mantienen una amistosa conversación.
3. Expresión Mientras tanto, los camilleros buscan alguien a quien llevar en
su camilla.
a) Lenguaje absurdo: «¡Mi niña está triste! Mi niña cien veces
llamó por teléfono, sin que usted contestase a sus llamadas.
La niña está triste y la niña llora. La niña pensó que usted se  2. El tema es la crítica de las guerras. Otras ideas que aparecen
había muerto. La niña está pálida... ¿Por qué martiriza usted es lo absurdo.
a mi pobre niña?». Situaciones ridículas: «¡Las personas de-
centes deben llevar siempre patatas en los bolsillos, caballe-  3. En el teatro experimental el texto no es prioritario, ya que se
ro!», «Toda mi familia ha tomado siempre huevos fritos para trata de un espectáculo que exige algo más que literatura. De
desayunar... Solo los bohemios toman café con leche y pan ahí que, en este caso, la ironía y el humor absurdo sean los
con manteca.» C arácter de los personajes : «¡Las personas elementos que acercan la obra al espectador. El autor ironiza
decentes no salen por la noche a pasear bajo la lluvia...! ¡Us- sobre el paralelismo existente entre la situación de los solda-
ted es un bohemio, caballero!» «¡Usted vivirá en mi casa y mi dos de ambos bandos. Esto le sirve para mostrar la absurdi-
casa es una casa honrada!» dad de las guerras.

b) La comicidad reside en la alusión al refinamiento del modernis-  4. La situación real a la que se refiere el texto es la guerra civil
mo literario y de su lenguaje remilgado. La frase recuerda un española. Se muestra a partir de una serie de situaciones có-
célebre verso de Rubén Darío: La princesa está triste…, ¿qué micas y disparatadas.
tendrá la princesa?
c) Don Sacramento representa los valores tradicionales y la rigidez  5. Dichos hechos inverosímiles producen, al mismo tiempo, la
de costumbres, en este caso adscritas a las convenciones ma- sonrisa y la reflexión del espectador.
trimoniales de la época. Su nombre es alegórico (igual que el de
Doña Rosario) y hace referencia al sacramento del matrimonio  6. Ambos simbolizan los dos bandos enfrentados, los soldados
religioso. que nada saben y nada tienen que ver con el conflicto.

 7. Los dos están de acuerdo en lo absurdo y lo cruel de la gue-


4.  Análisis del tema y de la estructura rra. Arrabal no se podía identificar con los padres de Zapo
porque, por edad y estado civil (matrimonio convencional) di-
a) Nos acercamos al desenlace y Dionisio se deja llevar por la pre-
fícilmente pueden ser un reflejo de las ideas del autor.
sión social y opta por seguir el camino que le marcan, es decir,
casarse con su novia y olvidar a Paula y su mundo, mucho más
 8. Las reacciones de los personajes, tanto de los padres como
libre que el que conocía.
de los soldados y camilleros, son incoherentes en la situación
b) Los dos están muy nerviosos. Don Sacramento es el futuro sue- en que se encuentran, una trinchera en el frente de guerra. La
gro de Dionisio. ruptura con la lógica en el lenguaje se ve, por ejemplo, cuan-
do preguntan a Zepo si está herido y él responde «Yo tampo-
c) Que se adapte a las normas socialmente aceptadas, así como al
co. Nunca he tenido suerte…» o cuando la Sra. Tepán le
orden y a las costumbres de su familia. Para Dionisio representa
pregunta a Zepo desde cuándo es enemigo: «Es de nacimien-
la aceptación de una vida convencional que no desea.
to, o se hizo usted enemigo más tarde?». Ejemplos de comici-
d) La vida burguesa es la que Dionisio acepta al casarse con su dad en los personajes los tenemos cuando la Sra. Tepán
prometida, una vida decente y ordenada, basada en las apa- interviene en la conversación sobre las heridas de guerra di-
riencias, tal como le exige don Sacramento. La vida bohemia es ciendo que aquella mañana se había cortado pelando cebo-
la alternativa que esa noche conoce en el hotel, que representa llas y entonces el marido pide al camillero que se la lleve.
la libertad.
e) Paula representa el amor prohibido del personaje, sus verdade-
 9. El teatro de Fernando Arrabal se vincula, en general, a la ten-
dencia experimental. Pic-Nic pertenece a su primera época
ros deseos. Al ocultarlo está reprimiéndolos y, por tanto, mani-
como dramaturgo, y en este período sus obras presentan
fiesta su falta de libertad.
universos cerrados en los que los personajes reaccionan de
f) Se critica la sociedad burguesa, sobre todo sus formalismos y la forma incoherente; una actitud que refleja su dificultad para
necesidad de guardar las apariencias. adaptarse a la realidad cotidiana.

10. Pic-Nic muestra las influencias vanguardistas que la drama-


5.  Comentario y opinión crítica turgia extranjera tuvo en los autores del Nuevo Teatro. Concre-
a) Respuesta abierta. tamente en esta obra se adivinan elementos surrealistas,
vanguardia, el surrealismo, que siempre ha atraído mucho a
b) Respuesta abierta. Arrabal.

83
La literatura hispanoamericana
16 # desde el siglo xx
poema, se utiliza la metáfora «gran río de aire», «gran río
EN CONTEXTO (pág. 339)
de huesos», «gran río de sangre», y la repetición de algu-
a) ¿Cómo crees que es la literatura hispanoamericana? Respuesta nos términos (sangre, río) y la epifora «gran río».
orientativa. La literatura hispanoamericana busca el equilibrio d) Respuesta libre.
entre sus raíces coloniales y la búsqueda de su propia identidad
americana para lograr autonomía y originalidad, independiente-  3. a) Deseo del poeta de la pervivencia de su amor más allá de
mente de las claras influencias extranjeras. ¿Y el realismo la muerte.
mágico? El realismo mágico parte de una alteración de la realidad
singular, inesperada e inverosímil. b) El poeta emplea un tono optimista en los primeros versos
cuando se dirige a su amada con fuerza y energía para que
b) Respuesta abierta. su amor sobreviva a su muerte y ella pueda seguir siendo
feliz a pesar de su ausencia. Sin embargo, a partir del pri-
mer terceto, el poeta expone el motivo por el que no debe
ACTIVIDADES (págs. 355-358) estar triste, porque él estará presente en su ausencia.
c) Pablo Neruda desea que su mujer siga feliz y no sufra su
1.  La poesía hispanoamericana  (págs. 343-346) ausencia porque tras ella hay una presencia.
d) El lenguaje es claro y sencillo, con una gran profundidad y
 1. a) Analiza la métrica… El poema no presenta una estrofa fija. emotividad sentimental. Caracteriza el poema la construc-
Se trata de un romance de verso irregular (versos de 8, 10
ción paralelística («de sur a sur levanta tus ojos indele-
y 14 sílabas), con rima asonante en i-a en los pares.
bles, / de sol a sol que suene tu boca de guitarra» (vv. 2 y
b) Indica la función…Son elementos que tienen una función 3), «Es una casa tan grande la ausencia… / Es una casa
expresiva. Las exclamaciones intensifican el sentido del tan transparente la ausencia» (vv. 9 y 12) y el contraste:
verso, y las interrogaciones son preguntas retóricas que se entre vivir / morir, que concluye con la paradoja final, hi-
dirigen al amado ausente. pérbole del amor, de los últimos versos «que yo sin vida te
veré vivir y si sufres, mi amor, me moriré otra vez». Tam-
c) Señala las imágenes… «A abrir mis venas y mi pecho, / a bién presenta un contraste el ímpetu inicial de los prime-
mondarme en granada viva, / y a romper la caoba roja / de ros versos con la ternura y melancolía de los últimos. La
mis huesos que te querían.» (vv. 23-26). «¡Cómo duele, anáfora del segundo cuarteto «No quiero que vacilen tu
cómo cuesta, / cómo eran las cosas divinas, / y no quieren risa ni tus pasos, / no quiero que se muera mi herencia de
morir, y se quejan muriendo, / y abren sus entrañas vívi- alegría, / no llames a mi pecho, estoy ausente» es una in-
das!» (vv. 41-44). tensificación de todo lo que el poeta no quiere que pase.
d) Indica a qué tendencia… Pertenece a la poesía intimista. Su Estas ideas contrastan con la estrofa anterior, en la que el
lenguaje sencillo y el tema amoroso son algunas de las poeta expresa lo que sí quiere que suceda.
características de esta corriente. Imagen de la ausencia como una casa transparente. El
poeta quiere que su amada viva feliz en esa casa para que
 2. a) Analiza la métrica… En ambos casos se trata de dos poe-
pueda verla siempre, aunque no esté con ella.
mas con rima libre.
e) Podemos dividir el poema en dos unidades. La primera
b) Escribe un título alternativo… Respuesta abierta. abarca los dos primeros cuartetos y es la voluntad del poe-
c) ¿A qué tendencia… Ambos poemas se inscriben en la poe- ta de que su amada no sufra si él muere. La segunda es la
sía social. reflexión melancólica de lo que significa la ausencia y la
petición del poeta de que la amada viva en la ausencia.
El tipo de lenguaje que se utiliza: Ambos utilizan un lenguaje
bastante claro y sencillo. El uso de la persona verbal y la in-
tención del autor. El poema de Mario Benedetti describe
 4. a) Señala las imágenes oníricas y surrealistas que aparecen:
las miserias de la pobreza y las injusticias del poder. El «Yo sueño, sobrellevando mis vestigios morales. (...) /
poeta se dirige a la humanidad, que se queda impasible. Estoy solo entre materias desvencijadas, / la lluvia cae
Se emplea la tercera persona. sobre mí, y se me parece, / se me parece con su desva-
río, solitaria en el mundo muerto, / rechazada al caer, y
En el poema de Nicolás Guillén, el autor se dirige al lector sin forma obstinada.»
increpándole su actitud indiferente e incitándole a que
salga y vea el dolor que se padece en la calle. Es un poema ¿Qué pretende transmitir el poeta con ellas? El autor presenta
más punzante y directo que el anterior, como puede com- una visión del mundo a través de diversas imágenes visio-
probarse por el uso del imperativo. narias con un tono de deterioro: «materias desvencijadas,
mundo muerto».
Los recursos estilísticos: En el primer poema, al comienzo
aparece la metáfora del «purgatorio del hambre y de la b) Pertenece al libro Residencia en la Tierra, del período su-
sed», y hacia el final, la epífora de «atroz» (epífora: repeti- rrealista del poeta, en el que realiza una reflexión metafísi-
ción de una palabra al final de una frase). En el segundo ca apasionada sobre el sentido de la vida; aquí nos
muestra, con imágenes oníricas, un mundo desolado.

84
Unidad 16 >  La literatura hispanoamericana desde el siglo xx

  5. a) El sueño como la inconsistencia, como muerte, como oca- b) ¿Qué significado crees que tiene… Este hecho marca un cam-
so y como triste oro. La poesía como aurora y también bio de etapa en su experiencia vital: deja de ser una niña.
como ocaso. El arte debe ser como un espejo. Se refiere a
elementos que se desvanecen. c) Relaciona el estilo y el tipo… En esta obra se presenta otra
realidad social, la de una familia de buena posición, aun-
b) El ultraísmo. que no se abandona el tono de realismo mágico. Con esta
obra, Isabel Allende inicia una corriente de literatura sobre
mujeres; en este caso, relata la vida de Clara con una vi-
2.  La novela hispanoamericana  (págs. 347-351) sión marcada por la sensibilidad femenina.

 6. a) Escribe un título alternativo… Respuesta abierta.


3. El cuento  (págs. 352-353)
b) ¿Qué tono utiliza en el fragmento… El autor utiliza un tono
humorístico, con matices absurdos. Justifica que el texto
pertenece a la corriente… El fragmento se inscribe dentro 10. Justificaque el texto… Presenta el elemento fantástico («un

de la corriente artística que experimenta nuevas formas, libro con una página en blanco») e inquietante («En un pue-
en este caso a partir de la burla y la ironía. blo de Escocia»), y el desenlace introduce el desconcierto
existencial («Si un lector […] muere»). Por otra parte, este
 7. a) Relaciona el estilo y las técnicas narrativas… En el texto apa- relato presenta un solo plano narrativo y una estructura lógi-
recen distintos planos de la narración superpuestos: el ca: presentación, nudo y desenlace. Esto no sucede en mu-
narrador, el diálogo de los niños y los ladridos de los pe- chos cuentos hispanoamericanos, que suelen presentarse in
rros. Estos planos se entremezclan gracias a una puntua- medias res.
ción escasa. Este estilo es fruto de la experimentación que
caracteriza la literatura del boom. 11. Temática: En los dos poemas se trata el tiempo: en el de Bor-
ges es un tiempo cíclico, las historias se van repitiendo a lo
b) Chingolo: Pucha diablo. Si se escapa un día... / Mañuco: Si largo del tiempo, mientras que Cortázar, a partir del reloj que
se escapa, hay que quedarse quietos, los daneses solo le regalan, va elucubrando sobre la angustia que le produce el
muerden cuando huelen que les tienes miedo. / Chingolo: paso del tiempo. / Estructura : En el texto de Borges se repro-
¿Quién te lo dijo?/ Mañuco: Mi viejo. / Choto: Yo treparía al duce la estructura de una argumentación, pero aplicando una
arco, así no me alcanzaría... lógica irreal, desconcertante. Se plantea una tesis: Al destino
le agradan las repeticiones…, y se defiende comparando dos
 8. a) Explica los siguientes aspectos… La ambientación: la acción se situaciones de distintas épocas.
sitúa en Macondo, un lugar inventado por el autor. / Tipo de
narrador y punto de vista: Narrador heterodiegético, como En el texto de Cortázar se empieza introduciendo un elemento
narrador testigo. / Elementos maravillosos e hipérboles: En la desestabilizador, el reloj, que desencadena una espiral de
novela del boom se mezcla la descripción de hechos rea- angustia con un desenlace sorprendente. / Estilo : Borges
les con otros inventados; en este caso, los elementos ma- trata un tema literario, con un estilo cuidado (descubre las
ravillosos son el efecto que causan los pájaros en toda la caras y los aceros… Le agradan las repeticiones, las variantes,
aldea y el cambio de espíritu de José Arcadio Buendía por las simetrías), aunque también introduce americanismos
la influencia de elementos extraordinarios y sus deseos de («¡Pero, che!»). En el caso de Cortázar, se trata de un estilo
conocer las maravillas del mundo. / Las hipérboles que sencillo, marcado por la repetición rítmica de «te regalan
aparecen son el hecho de que Úrsula se tuviera que tapar un…» o «no te regalan un…».
los oídos con cera de abejas para no perder el sentido de la
realidad y que Úrsula utilizara un cuchillo de cocina para
cortar la barba salvaje de su marido. / Recursos del cuento EVALUACIÓN (pág. 363)
popular y el mito: Introduce una pequeña narración con
introducción, desarrollo y conclusión con elementos mara-  1. Escribe un título…: Respuesta abierta.
villosos, que recuerda a un cuento popular.

b) Gabriel García Márquez recrea en Macondo… El nombre  2. Analiza el estilo y la estructura del texto… El estilo es directo,
de Macondo se quedó en la memoria de Gabriel García basado en preguntas y respuestas. Las preguntas son claras y
Márquez desde niño, desde que lo leyó en el letrero de una directas, con el objetivo de conocer la opinión de la entrevista-
finca cuando viajaba en tren desde Aracataca. En su libro da. Las respuestas son igualmente concisas.
de memorias Vivir para contarla, explica: «Esta palabra me
había llamado la atención desde los primeros viajes con mi  3. Señala la respuesta adecuada… No pertenece a un círculo de
abuelo, pero solo de adulto descubrí que me gustaba su escritores.
resonancia poética. Nunca se lo escuché a nadie ni pre-
gunté siquiera qué significaba».  4. Explica qué quiere decir Isabel Allende… «Tenemos que dar el
doble de batallas para ganar la mitad de reconocimiento»:
A las
 9. a) Indica si hay algún elemento fantástico… «Los objetos tenían mujeres les cuesta mucho más que a los hombres obtener el
vida propia; los aparecidos se sentaban en la mesa y ha- reconocimiento como escritoras. / «La literatura misma tiene
blaban con los humanos; el pasado y el futuro eran parte como única materia prima la palabra »: El escritor solo debe

de la misma cosa», … En la casa convivía el pasado con el preocuparse de escribir bien y entonces es seguro que le lee-
presente; Clara vivía sin tiempo, sin salir de la casa ni con- rán. / «Unos se apoyan en los otros, se inflan los unos a los
tacto con el exterior; Clara no habló desde los diez hasta otros»: Los escritores de un mismo grupo establecen relacio-

los diecinueve años. nes de mutuo apoyo.

85
Unidad 16 >  Consecuencias sociales de la Revolución Industrial

 5. ¿Por qué no debe hablarse de literatura femenina… La escritora mitad del siglo xx. Recibió el Premio Nobel de Literatura en
no cree que deba existir una diferenciación de la literatura crea- 1945. Los temas de su poesía son la docencia, el amor a los
da por las mujeres, para que no se las aparte, aunque reconoce niños, el paisaje chileno y el dolor por la muerte de los seres
que la edad, el sexo y el contexto social influyen en la literatura. queridos. Su poesía reúne las tradiciones latinoamericanas y
las tendencias de vanguardia.
 6. Explica la importancia que tiene el mercado editorial… Los edito-
res publican libros de escritoras porque las mujeres leen cada
 8. Explica por qué suele clasificarse a Isabel Allende… Esta primera
vez más. ¿Qué provocó el auge de este mercado… El llamado
novela de la escritora tuvo un gran éxito, lo que significó un
boom latinoamericano.
impulso para la literatura escrita por mujeres.
 7. Indica qué género cultivó Gabriela Mistral… La chilena Gabriela
Mistral (1889-1957) cultivó la poesía intimista en la primera  9. Redacta un texto… Respuesta abierta.

86
LENGUA Y LITERATURA 2 BACHILLERATO
SEGUNDO CURSO

Recursos didácticos. Orientaciones y solucionario


Modalidad de Humanidades y Ciencias Sociales

Proyecto y edición: grupo edebé

Dirección general: Antonio Garrido González


Dirección del área de productos educativos: Esteban Lorenzo Domínguez
Dirección del área de Humanidades y Ciencias Sociales: Natalia Corretjé Carné
Dirección de Pedagogía: Santiago Centelles Cervera
Dirección de Producción: Juan López Navarro

Equipo de edición de edebé:


Edición:  Sara Gómez Marino y María Lorente Torres
Asesoría:  José Arlegui Suescun y Ángel María Oronoz Irigoyen
Pedagogía:  Santi Centelles Cervera
Corrección:  María José Gracia Bona
Diseño gráfico y cubierta: Luis Vilardell Panicot

Colaboradores:
Texto: Marina Arrufat Pradera, Noelia Nogales Merino, Cristina Illamola i Gómez y Marc Granó Plaza
Preimpresión: Copydrana

Cualquier forma de reproducción, distribución, comunicación pública o transformación de esta obra solo puede ser realizada con la autorización de
sus titulares, salvo excepción prevista por la ley. Diríjase a CEDRO (Centro Español de Derechos Reprográficos) si necesita fotocopiar o escanear algún
fragmento de esta obra (www.conlicencia.com; 91 702 19 70 / 93 272 04 45).

El libro incluye una cuidada selección de enlaces de páginas web que el grupo edebé considera que pueden ser de interés. No obstante, estas páginas
no le pertenecen. Por tanto, el grupo edebé no puede garantizar la permanencia ni la variación de sus contenidos y tampoco se hace responsable de
los posibles daños que puedan derivarse del acceso o del uso de las páginas.

Los editores han hecho todo lo posible para localizar a los titulares de los materiales que pudieran aparecer en la obra. Si involuntariamente alguno
ha sido omitido, los editores subsanarán el error cuando sea posible.

Es propiedad de grupo edebé


© grupo edebé, 2014
Paseo San Juan Bosco, 62
08017 Barcelona
www.edebe.com

ISBN 978-84-683-1755-7
Depósito Legal. B. 14934-2016
Impreso en España
Printed in Spain
EGS - Rosario, 2 - Barcelona

You might also like